89
Enlace Químico y Estructura de la Materia Enlace Químico y Estructura de la Materia Licenciatura en Química Departamento de Química Inorgánica DE ALCALÁ UNIVERSIDAD

Enlace Quimico Y Estructura de Materia

Embed Size (px)

Citation preview

Page 1: Enlace Quimico Y Estructura de Materia

7/22/2019 Enlace Quimico Y Estructura de Materia

http://slidepdf.com/reader/full/enlace-quimico-y-estructura-de-materia 1/89

Enlace Químicoy Estructura de

la Materia

Enlace Químicoy Estructura de

la Materia

Licenciatura en Química

Departamento de Química Inorgánica DE ALCALÁUNIVERSIDAD

Page 2: Enlace Quimico Y Estructura de Materia

7/22/2019 Enlace Quimico Y Estructura de Materia

http://slidepdf.com/reader/full/enlace-quimico-y-estructura-de-materia 2/89

Índice

Tablas 1

1 La estructura electrónica de los átomos 51.1 La estructura del átomo1.2 La luz: frecuencia, cuantos y efecto fotoeléctrico1.3 El espectro del hidrógeno atómico. El modelo de Bohr y sus ampliaciones1.4 El modelo mecano-cuántico. La ecuación de Schrödinger1.5 El átomo de hidrógeno en el modelo mecano-cuántico. Forma de los orbitales atómicos

1.6 Los átomos polielectrónicos1.7 Configuraciones electrónicas de los átomos polielectrónicos en su estado fundamental1.8 Estados atómicos de energía. Términos de Russell–Saunders1.9 Periodicidad de algunas propiedades físicas

Bibliografía / Seminarios / Problemas / Soluciones

2 El enlace en las moléculas 362.1 La formación de pares iónicos2.2 El enlace covalente: el enlace del par de electrones2.3 Parámetros de los enlaces covalentes2.4 El carácter iónico de los enlaces covalentes y el carácter covalente de los enlaces iónicos2.5 La teoría de la repulsión de los pares electrónicos de la capa de valencia (VSEPR)

2.6 Las moléculas polaresBibliografía / Seminarios / Problemas / Soluciones

3 Orbitales y enlaces químicos 543.1 La descripción mecano-cuántica del enlace químico3.2 La teoría del enlace de valencia (TEV)3.3 La teoría de los orbitales moleculares (TOM)3.4 Ácidos y bases de Lewis

Bibliografía / Seminarios / Soluciones

4 Sólidos 654.1 Comparación entre las propiedades de sólidos, líquidos y gases4.2 Fuerzas entre moléculas y iones

4.3 Estructuras de los cristales4.4 Bandas de orbitales moleculares4.5 Sólidos metálicos4.6 Sólidos iónicos: enlace y estructura4.7 La formación de sólidos iónicos: entalpía de red4.8 Sólidos moleculares4.9 Sólidos covalentes

4.10 Comparación entre las propiedades de los distintos tipos de sólidosBibliografía / Seminarios / Problemas / Soluciones

5 Gases y líquidos 875.1 La teoría cinético-molecular de los gases

5.2 Predicciones de la teoría cinético-molecular5.3 Los gases reales: ecuación de Van der Waals5.4 Propiedades de los líquidos

Bibliografía / Seminarios / Problemas / Solucionesç

________________________________________________________________________________________________

©©©© 2222000000003, EEEErrrrnnnneeeessssttttoooo ddddeeee JJJJeeeessssúúúússss AA A Allllccccaaaaññññiiiizzzz

Page 3: Enlace Quimico Y Estructura de Materia

7/22/2019 Enlace Quimico Y Estructura de Materia

http://slidepdf.com/reader/full/enlace-quimico-y-estructura-de-materia 3/89

_________________________________________________________________________________________

1 La estructura electrónica de los átomos1.1 La estructura del átomo

1.2 La luz: frecuencia, cuantos y efecto fotoeléctrico

1.3 El espectro del hidrógeno atómico. El modelo de Bohr y sus ampliaciones

1.4 El modelo mecano-cuántico. La ecuación de Schrödinger

1.5 El átomo de hidrógeno en el modelo mecano-cuántico. Forma de los orbitales atómicos

1.6 Los átomos polielectrónicos

1.7 Configuraciones electrónicas de los átomos polielectrónicos en su estado fundamental

1.8 Estados atómicos de energía. Términos de Russell–Saunders

1.9 Periodicidad de algunas propiedades físicas_________________________________________________________________________________________

1.1 La estructura del átomo

 Las partículas subatómicas. Durante el siglo XIX, el descubrimiento de la electrolisis del agua (1800,

Nicolson y Carlisle) y de las leyes de Faraday (1832) sugieren que las unidades de carga eléctrica están

asociadas con los átomos. Pero no es hasta finales del siglo XIX y principios del XX que se identifican

las partículas cargadas que componen el átomo. El protón y el electrón fueron identificados estudiando laconductividad de los gases a bajas presiones por Goldstein en 1886 y por Thomson en 1897,

respectivamente. Estudiando el comportamiento de las partículas frente a campos eléctricos y

magnéticos, se determinó que el protón es una partícula de carga positiva con una relación masa/carga de

1,04 10 –8 kg/C, mientras que el electrón es una partícula de carga negativa con una relación masa/carga

de 5,69 10 –12 kg/C. En 1909, Millikan estudia la carga adquirida por una gota de aceite al ser irradiada

con rayos X y propone que la unidad elemental de carga asociada al electrón y al protón vale 1,60 10 –19

C. A partir de este valor de carga y de sus relaciones masa/carga, se determinaron las masas del electrón

y la del protón, siendo ésta 1835 mayor que aquella (tabla 1.1). Al bombardear berilio con partículas α de

alta energía, Chadwick demostró en 1932 que los átomos están constituidos también por partículas sincarga llamadas neutrones, de masa similar a la del protón.Tabla 1.1. Las partículas subatómicas

 Nombre Símbolo Masa Carga Descubrimiento

protón p 1,673 10 –27 kg +1,602 10 –19 C Goldstein, 1886electrón e 9,109 10 –31 kg –1,602 10 –19 C Thomson, 1897neutrón n 1,675 10 –27 kg Chadwick, 1932

 El modelo atómico de Rutherford . En 1911, Rutherford, Geiger y Marsden estudiaron las trayectorias

de las partículas α disparadas contra láminas de diferentes materiales (figura 1.1). De acuerdo con los

resultados obtenidos, Rutherford propuso un modelo de átomo (figura 1.2) caracterizado por la existenciade un núcleo central con una carga positiva idéntica a la negativa de los electrones, que están fuera del

núcleo, y que contiene el 99,9% de la masa total del átomo en sólo el 0,01% de su diámetro (d ≈ 1 106

toneladas/cm3). El tamaño de un átomo es del orden de 10 –10 m (= 1 Å) mientras que el del núcleo es del

orden de 10 –15 m.

Figura 1.1. Dispositivo experimental de Rutherford para lamedida de la dispersión de partículas α, mediante láminasmetálicas muy delgadas. La fuente de partículas α es elpolonio radiactivo colocado en el interior de un bloque de

plomo, que sirve para proteger de las radiaciones y paraseleccionar un haz de partículas. La lámina de oro que seutilizó tenía un espesor de 6 10 –5 cm. La mayoría de laspartículas pasaban con poca o ninguna desviación, a. Unaspocas se desviaban ángulos grandes, b, y, ocasionalmente,alguna partícula rebotaba en la lámina, c.

Pantalla de

centelleo

Lámina

de oro

Haz de

partículas α

Fuente

a

b

b

c

Page 4: Enlace Quimico Y Estructura de Materia

7/22/2019 Enlace Quimico Y Estructura de Materia

http://slidepdf.com/reader/full/enlace-quimico-y-estructura-de-materia 4/89

Figura 1.2. Interpretación del experimento de Rutherford. La mayor partedel espacio de un átomo está casi “vacío” ya que sólo está ocupado porlivianos electrones. Toda la carga positiva del átomo y casi toda su masa seencuentra en su centro, en un núcleo muy denso y pequeño. La mayoría delas partículas α con carga positiva (a) atraviesan el átomo por el espaciodesocupado sin experimentar desviaciones. Algunas (b) se acercan a losnúcleos y se desvían al ser repelidas por su carga positiva. Sólo unas pocasllegan a acertar (c) en un núcleo y salen despedidas hacia atrás.

a

a

a

b

c

La representaciónno es a escala. Silos núcleos fuerantan grandes comolos puntos negrosque losrepresentan, eltamaño del átomo

debería ser de unasdecenas de metros.

 Nucleones. El núcleo atómico está constituido por protones y neutrones, que por ello se llaman nucleo-

nes. El número atómico ( Z ) de un átomo es el número de protones, que es igual al de electrones en el áto-

mo neutro, e identifica a un elemento. El número neutrónico ( N ) es el número de neutrones. El número

másico ( A) de un átomo es el número de nucleones, A = Z + N . Un elemento puede contener átomos de

diferente número másico, es decir, puede contener diferentes isótopos de distinta masa atómica (tabla

1.2). En un elemento natural, la abundancia relativa de sus isótopos en la naturaleza suele ser casi cons-

tante y recibe el nombre de abundancia isotópica natural. La denominada masa atómica de un elemento

es una media de las masas de sus isótopos naturales ponderada de acuerdo a su abundancia relativa.Tabla 1.2. Algunos elementos con varios isótopos

 Nombre Símbolo Número

atómico (Z) Número

neutrónico (N) Número

másico (A) Masa Abundancia

natural, %

Hidrógeno–1 1H 1 0 1 1,674 10 –24 g, 1,008 u 99,985Deuterio 2H o D 1 1 2 3,344 10 –24 g, 2,014 u 0,015Tritio 3H o T 1 2 3 1,674 10 –24 g, 3,016 u muy inestableCarbono–12 12C 6 6 12 1,9926 10 –23 g, 12 u exactas 98,90

Carbono–13 13C 6 7 13 2,159 10 –23 g, 13,00 u 1,10Cloro–35 35Cl 17 18 35 5,807 10 –23 g, 34,97 u 75,77Cloro–37 37Cl 17 20 37 6,138 10 –23 g, 36,97 u 24,23

Uranio–235 235U 92 143 235 3,902 10 –22 g, 235,0 u 0,72Uranio–238 238U 92 146 238 3,953 10 –22 g, 238,05 u 99,27

 Modelo electrónico. ¿Por qué los electrones no caen sobre el núcleo que los atrae? La situación recuerda

a la Tierra que no cae sobre el Sol porque gira en una órbita estacionaria, en la que fuerza centrípeta y

fuerza de atracción gravitatoria se igualan. Sin embargo, el modelo planetario no es aplicable a partículas

cargadas, ya que según las leyes clásicas del electromagnetismo, un electrón con un movimiento

acelerado, como el circular, radia energía electromagnética, lo que debería producir su progresiva caída

sobre el núcleo. Un modelo electrónico coherente se encuentra en las leyes de la mecánica cuántica.

1.2 La luz: frecuencia, cuantos y efecto fotoeléctrico Naturaleza ondulatoria de la luz. La luz es una radiación electromagnética, es decir, una onda de cam-

pos eléctricos y magnéticos. Se caracteriza, como cualquier onda, por su  frecuencia (ν ), que se define

como el número de ciclos por unidad de tiempo y cuya unidad en el SI es el hertz o hercio (Hz),

equivalente a 1 (ciclo) s –1. La longitud de onda (λ ) es la distancia entre picos y su relación con la

frecuencia es λ = c/ν , donde c es la velocidad de la luz. La frecuencia de una luz determina su color ,

aunque sólo es visible una parte del espectro electromagnético (figura 1.3).

 La luz como un haz de partículas. En 1900, Max Planck estudia la radiación emitida por un cuerpo

negro y observa que es como si ésta fuera emitida en porciones E = hν , donde h es la constante de Planck

(6,63 10 –34 J s). En 1906, Einstein estudia el efecto fotoeléctrico (figura 1.4) y, para explicarlo, proponeque la luz puede ser considerada como un haz de partículas o como una onda, con una relación E = hν 

entre la energía cinética de cada partícula y la frecuencia de la onda (recuadro 1.1). Una partícula de luz

recibe el nombre de fotón y la energía de cada fotón es un cuanto de energía.

6 | Enlace químico y estructura de la materia Licenciatura en Química

Page 5: Enlace Quimico Y Estructura de Materia

7/22/2019 Enlace Quimico Y Estructura de Materia

http://slidepdf.com/reader/full/enlace-quimico-y-estructura-de-materia 5/89

Rejillacolimadora

Películaa)

Luzblanca Prisma

Figura 1.3. a) La luz blanca es una mezclade radiaciones de todas las longitudes deonda de la luz visible. Si un rayo estrechose pasa a través de un prisma, se separa enun espectro continuo de todas laslongitudes de onda que componen la luzvisible. El color de una luz depende de su

frecuencia o longitud de onda.b) La luz visible es sólo una porción delespectro electromagnético.

b)

5,0 10 6,0 10 7,0 10

10 –210 –610 –7 10 –510 –8 10 –310 –410 –910 –1010 –11

Ondas

de radioMicroondasInfrarrojo

   V   i  s   i   b   l  e

Ultra-

violetaRayos XRayos γγγγ  

3 1019ν (s ) 3 1015 3 1011

λ (m)

RojoVioleta

7,5 1014

4,0 106,0 10 14 5,0 1014 4,3 1014

 –7 –7 –7 –7

 –1

ν (s ) –1

 E (kJ/mol) 107 103 10 –1

 E (kJ/mol)

λ (m)

3,0 102 2,4 10 2 2,0 102 1,7 102

ehν  1mv2

2 E c =

+

Placametálica

Tubo avacío

a)

e

ν 

   M  e  t  a   l   A

   M  e  t  a   l    B

 E c

ν 0B

b)

ν 0A

Figura 1.4. a) Efecto fotoeléctrico. Cuando una radiación electromagnética de frecuencia suficiente choca contra la superficiede un metal (electrodo negativo) en el interior de un tubo a vacío, se desprenden electrones del metal que crean una corrienteeléctrica. La intensidad de la corriente crece con la intensidad de la radiación.b) La gráfica relaciona la energía cinética de salida de los electrones con la frecuencia de la luz. Para cada metal, hay unafrecuencia umbral ν 0 por debajo de la cual la luz no es capaz de arrancar electrones de la placa metálica. Por encima de lamisma, la energía cinética aumenta linealmente con la frecuencia. La pendiente de la recta es la misma para todos los metales.

Recuadro 1.1. El efecto fotoeléctrico

Las observaciones experimentales señaladas en la figura 1.4 pueden explicarse suponiendo que la luz está compuesta defotones de energía hν , de forma que cada fotón puede transmitir su energía a un sólo electrón durante la colisión.• Una luz es incapaz de arrancar electrones de un metal por muy intensa que sea, si no tiene una frecuencia míni-

ma. Esta frecuencia umbral es diferente para cada metal. Un electrón se encuentra atrapado en un metal por unaenergía de la que debe disponer como mínimo un fotón para poder arrancarlo. Cuanto más intensa sea una luz, mayores el número de fotones del rayo, pero la energía de cada fotón únicamente depende de la frecuencia de la radiación.Si E 0

A es la energía que atrapa a un electrón en el metal A, la frecuencia mínima ν 0A que debe tener un fotón es

 E 0A = hν 0A

Cada metal tiene una energía E 0, y por tanto una frecuencia umbralν 0, característica.• La energía cinética de los electrones depende linealmente de la frecuencia de la luz, siendo la pendiente igual

para todos los metales. Por choque, el fotón transmite toda su energía al electrón, el cual usa parte en salir del metal y

el resto la almacena en forma de energía cinética. La energía cinética de los electrones arrancados debe ser igual a E c = E  fotón – E 0 = hν  – hν 0 = h(ν  –ν 0)

lo que explica la línea recta de la figura 1.4b en la que la pendiente, igual para todos los metales, debe ser igual a h.• La intensidad de la corriente crece con la intensidad de la radiación. Un rayo intenso contiene un mayo múmero

de fotones, por lo que es capaz de arrancar un mayo número de electrones.

Universidad de Alcalá Tema 1: La estructura electrónica de los átomos | 7

Page 6: Enlace Quimico Y Estructura de Materia

7/22/2019 Enlace Quimico Y Estructura de Materia

http://slidepdf.com/reader/full/enlace-quimico-y-estructura-de-materia 6/89

1.3 El espectro del hidrógeno atómico. El modelo de Bohr y sus ampliaciones

 El espectro del hidrógeno. Cuando se pasa una corriente eléctrica a través de un gas en un tubo a presión

muy baja, se emite una luz cuyo espectro no es continuo sino a líneas (figura 1.5).

λ (m) 5,0 10 6,0 10 7,0 104,0 10 –7 –7 –7 –7

Figura 1.5. La luz que emite un tubo dedescarga relleno de un gas estácompuesta por frecuencias discretas y daun espectro de líneas.

Cada gas da un espectro característico. Al estudiar el espectro del hidrógeno, se encontró una relación

matemática entre las frecuencias de sus líneas llamada ecuación de Rydberg:

ν = ℜ −

1 1

1

2

2

2n n

Serie de Lyman n1 = 1 n2 = 2, 3, 4, 5… Serie de Brackett n1 = 4 n2 = 5, 6, 7, 8…Serie de Balmer n1 = 2 n2 = 3, 4, 5, 6… Serie de Pfund n1 = 5 n2 = 6, 7, 8, 9…Serie de Pashen n1 = 3 n2 = 4, 5, 6, 7… ℜ = 3,29 1015 s –1 (constante de Rydberg)

¿Cómo se origina un espectro de emisión? En un tubo de descarga, la corriente eléctrica aumenta la tem-

peratura generando suficiente agitación térmica como para que los átomos rompan sus enlaces y los elec-

trones salten a estados de energía superior. Los espectros de emisión los producen los átomos al retornar

sus electrones a estados de menor energía, por lo que informan sobre los estados electrónicos del átomo.

 El modelo de Bohr. Con esta información, Bohr propuso en 1913 su modelo de electrones que giran en

torno al núcleo en órbitas circulares para las que no es aplicable el concepto clásico según el cual una

carga acelerada emite radiaciones continuamente. Bohr postuló que “un electrón siempre absorbe o emite

energía electromagnética en cuantos completos de hν ” (primer postulado). Como la luz emitida en el

espectro de un átomo no contiene todas las frecuencias, Bohr sugirió que no todas las órbitas son posibles

y, en concreto, propuso que “las únicas órbitas en que se puede mover un electrón son aquellas en las que

su momento angular es nh/2π, siendo n un número entero. Cuando un electrón se encuentra en estas

órbitas, no emite energía” (segundo postulado). El número n se llama número cuántico principal.A partir de estos postulados (recuadro 1.2), Bohr calculó la energía (figura 1.6) y el radio (figura

1.7) de las órbitas de un electrón en el átomo de hidrógeno. Además, dedujo la ecuación de Rydberg y el

valor de su constante (recuadro 1.3).Recuadro 1.2. Modelo de Bohr: Deducción del radio y energía de las órbitas

La fuerza centrípeta asociada al giro de un electrón en una órbita circular vale

 F centrípeta = mev2

r  (me = masa del electrón, v = velocidad, r = radio de la órbita)

La fuerza de atracción electrostática entre el electrón y el núcleo vale

 F elect . = 14πε 0

 Ze2

r 2( Z  = número atómico, 1 para H, e = carga elemental)

Si el electrón gira estacionariamente, ambas fuerzas tienen que ser iguales, mev2

r  = 14πε 0

 Ze2

r 2

Despejando la velocidad, v = 14πε 0

 Ze2

mer Velocidad que, según la mecánica clásica, debe tener un electrónpara moverse estacionariamente en una órbita de radio r .

Para que en su giro el electrón no emita ninguna radiación electromagnética, el segundo postulado de Bohr señala que sumomento angular (mevr ) debe de ser igual a nh/2π

 mevr = me1

4πε 0

 Ze2

mer r = nh2π

donde n = 1, 2, 3...

Despejando r , r = h2ε 0

πmee2

n2

 Z = a0

n2

 Z Radio de las únicas órbitas admitidas por el 2º postulado de Bohr.La constante a0 recibe el nombre de radio de Bohr y vale 0,529 Å.

La energía total de cada órbita viene dada por:

  E = E cinética + E  potencial = 12

mev2 ± 14πε 0

 Ze2

r  = 12

me1

4πε 0 Ze2

mer ± 1

4πε 0 Ze2

r  = ± 14πε 0

 Ze2

2r 

Sustituyendo r , por la expresión obtenida anteriormente:  E = ± 14πε 0

 Ze2

2r = ±

mee4

8h2ε 0

2

 Z 2

n2= ± 21,8 10±19 Z 2

n2julios

8 | Enlace químico y estructura de la materia Licenciatura en Química

 Ze+

v

r  e  – 

Page 7: Enlace Quimico Y Estructura de Materia

7/22/2019 Enlace Quimico Y Estructura de Materia

http://slidepdf.com/reader/full/enlace-quimico-y-estructura-de-materia 7/89

n = 6

 –21,8

 –2,7

 –10,9

 –5,4

0,0

1 2 3 4 5 6n

   E  n  e  r  g   í  a   (  e  n   1   0 –   1   9   J  u   l   i  o  s   /   á   t  o  m  o   )

n = 1

n = 2

n = 3n = 4n = 5

Serie deLyman

Serie deBalmer

Serie dePaschen

Infrarrojo

Ultra-violeta

λ(m)

5,00 10 –7

6,00 10 –7

7,00 10 –7

4,00 10 –7

Rojo

Verde

AzulVioleta

Figura 1.6. a) Gráfica de la energía de los orbitales atómicos del hidrógeno en función de n. Obsérvese que al aumentar n, ladiferencia de energía entre estados de energía sucesivos es cada vez más pequeña.b) Algunas transiciones electrónicas que pueden ocurrir en un átomo excitado de hidrógeno. Las transiciones hasta n = 1 sonlas de mayor energía y dan las rayas de menor longitud de onda (las de la serie de Lyman en el ultravioleta). Las líneas de laserie de Balmer aparecen en el visible; la de mayor longitud de onda (la roja) se debe a una transición desde n = 3 a n = 2.

n = 1a0

4 a0

9 a0

n = 2 n = 3Figura 1.7. Distancia relativa entre las tres primeras órbitasatómicas del hidrógeno, según el modelo de Bohr. Obsérveseque la diferencia de distancia entre órbitas sucesivas es cadavez mayor al aumentar n. La distancia entre la primera órbita y

el núcleo (a0) es igual a 0,529 Å y se llama radio de Bohr .Recuadro 1.3. Modelo de Bohr: Deducción de la ecuación de Rydberg

Un electrón absorbe o emite energía al saltar de una órbita a otra del hidrógeno ( Z = 1):

  E 2 ± E 1 =

mee4

8h2ε 02

1n1

2± 1

n22

Energía emitida o absorbida por un electrón al pasar de unaórbita 1 a una órbita 2, en el átomo de hidrógeno.

El primer postulado de Bohr propone que cada electrón en una transición emite o absorbe un único cuanto de luz, por loque la frecuencia de la radiación emitida o absorbida vendrá dada por la relación de Planck:

  E  fotón = E 2 ± E 1 = hν; hν =

mee4

8h2ε 0

2

1n1

2± 1

n22

; ν =mee4

8h3ε 0

2

1n1

2± 1

n22

El valor de la expresiónmee4

8h3ε 02coincide con el valor experimental de la constante de Rydberg (diferencia < 0,00001%).

Otros números cuánticos. El espectro de emisión del hidrógeno se complica en presencia de un campo

magnético (efecto Zeeman). Para explicarlo, Sommerfeld sugirió la existencia también de órbitas elec-

trónicas elípticas cuya excentricidad vendría dada por un nuevo número cuántico llamado azimutal (l,

tabla 1.3) y su orientación por otro llamado magnético orbital (ml,). Los números cuánticos de espín (s y

ms) se propusieron para justificar la presencia de dobletes en el espectro de, entre otros, el hidrógeno

(efecto Zeeman anormal). A veces se mencionan sólo cuatro números cuánticos (n, l, mly ms), y se omite

el de espín s porque su valor para el electrón es constante (1/2).

A pesar del avance fundamental que representó el modelo de Bohr, pronto se vieron sus problemas:

hechos experimentales no explicados (momento magnético nulo del orbital s, átomos polielectrónicos…),ideas que se mostrarían como incorrectas (trayectorias y momentos definidos) y debilidades metodológi-

cas (era necesario un modelo donde el momento angular y los números cuánticos fueran un resultado de

la teoría misma y no un ajuste empírico en función de parámetros experimentales).

Universidad de Alcalá Tema 1: La estructura electrónica de los átomos | 9

Page 8: Enlace Quimico Y Estructura de Materia

7/22/2019 Enlace Quimico Y Estructura de Materia

http://slidepdf.com/reader/full/enlace-quimico-y-estructura-de-materia 8/89

Tabla 1.3. Números cuánticos del átomo de hidrógeno Nombre Símbolo Valores Nombre de conjunto Sinónimos Describeprincipal n 1, 2, 3,… nivel o capa K (n = 1), L (2), M (3), N (4), … tamaño y energía orbital

azimutal* l  0,…,.n –1 subnivel o subcapa s (l = 0), p (1), d (2), f (3), … forma del orbital

magnético orbital ml  +l,…,–l orbital** de una subcapa orientación del orbital

de espín s

1

/2 espín del electrónmagnético de espín ms ±1/2 orientación del espín

*También llamado momento angular orbital; **órbita en el lenguaje de Bohr, orbital en el de la mecánica cuántica.

1.4 El modelo mecano-cuántico. La ecuación de Schrödinger

 La dualidad partícula–onda. En 1924, de Broglie propone que cualquier partícula (por ejemplo, un elec-

trón) puede considerarse como una onda con λ = h/mv (tabla 1.4). En 1927, Davisson y Germer observan

la difracción, propiedad característica de las ondas, de un haz de electrones en una lámina metálica.Tabla 1.4. Longitudes de onda de partículas

 Partícula Masa (kg) Velocidad (m s –1 ) Longitud de onda (Å)Electrón libre gaseoso (300 K) 9,11 10 –31 1,17 105 63Electrón libre gaseoso (200 10 3 K) 9,11 10 –31 3,00 106 2,4Electrón del átomo de H (n = 1) 9,11 10 –31 2,19 106 3,3

Átomo de He gaseoso (300 K) 6,64 10 –27 1370 0,73Pelota de Béisbol 0,10 20 3,3 10 –24

 Principio de incertidumbre. La experiencia demuestra que en un experimento dado materia y radiación

exhiben un comportamiento de onda o un comportamiento de partícula, pero nunca los dos a la vez. Las

propiedades de onda y de partícula son complementarias en el sentido de que no se pueden conocer am-

bas con total precisión. Así lo recoge el  principio de incertidumbre, formulado en 1927 por Heisenberg,

que afirma que el producto de las imprecisiones de dos magnitudes complementarias A y  B no puede sernunca menor de una cantidad pequeña del orden de h. La posición y el momento en una misma dirección

son magnitudes complementarias entre sí y sus precisiones máximas están limitadas por la expresión:

 ∆ x ⋅ ∆(mv x ) ≥h

donde ∆ x y ∆(mv x ) son, respectivamente, las imprecisiones en la posición y la cantidad de movimiento.

 Ejemplo. Imprecisión mínima en la posición de partículas cuya imprecisión en la velocidad es del 1%

 Partícula Masa (kg) Velocidad (m s –1 ) Imprecisión mínima en la posición (m)Automóvil 1000 28 (100 km/h) 10 –37

Electrón del átomo de H (n = 1) 9,11 10 –31 2,19 106 26 10 –10 (26 Å, el radio de Bohr es 0,529 Å)La consecuencia de la incertidumbre posición–momento es que no se puede especificar la trayectoria de

una partícula, pues ello implicaría conocer en un instante dado con total precisión su posición y su mo-

mento. La mecánica cuántica no trata la trayectoria en términos de determinación sino de probabilidad.

 La ecuación de Schrödinger. Las ideas de De Broglie sugirieron la posibilidad de caracterizar a una

partícula como el electrón mediante una función de onda (ψ ). El electrón en el átomo está aprisionado

por el campo eléctrico producido por el núcleo. En 1926, Schrödinger propuso que las ondas de los

electrones en tal situación eran estacionarias (figura 1.8 y recuadro 1.4). Además propuso describir el

comportamiento del electrón mediante una ecuación de ondas, resultado de combinar las ecuaciones

clásicas de una onda y de una partícula mediante la ecuación de De Broglie (recuadro 1.5).t 

(b)

1

t 2

t 1 2

(a)

Figura 1.8. (a) Una onda no estacio-naria se ”desplaza“ con el tiempo. (b)Una cuerda de una guitarra vibrandoes un ejemplo de onda estacionaria.

10 | Enlace químico y estructura de la materia Licenciatura en Química

Page 9: Enlace Quimico Y Estructura de Materia

7/22/2019 Enlace Quimico Y Estructura de Materia

http://slidepdf.com/reader/full/enlace-quimico-y-estructura-de-materia 9/89

Recuadro 1.4. Función de una onda. Ecuación de ondas.

 La función de una onda. Una onda se describe mediante una función de ondasΨ ( x , t ) que da la altura de la onda encada punto e instante. En una onda estacionaria, como la representada en la figura 1.8b, esta función se puede expresarcomo el producto de una función dependiente de  x , que da la envolvente o amplitud máxima en cada punto, y otradependiente de t que indica en qué punto del recorrido se encuentra en un momento determinado:

Ψ ( x , t ) = ψ ( x ) ψ (t )

En nuestro desarrollo, sólo nos interesa la amplitud de la onda ψ ( x ), que para una onda como la de la figura 1.8b, sepuede describir mediante una función como la siguiente:ψ ( x ) = A sen (2π x /λ ) (1)

 La ecuación de ondas. Una función que represente a una onda cumple la ecuación de ondas (2):

d 2ψ 

dx 2= ± 4π2

λ 2ψ  (2)

 Ejercicio Demuestra que la función (1) cumple la ecuación (2).

Recuadro 1.5. Origen de la ecuación de Schrödinger

La ecuación de Schrödinger es el resultado de combinar la ecuación que caracteriza a una onda estacionaria (1) con laque caracteriza a una partícula (2), mediante la relación de De Broglie (3).

d 2

ψ dx 2

= ± 4π

2

λ 2ψ 

  E = E c + V  = 12

mev2+ V 

 λ = hmev

  E ± V =h

2

2me

1

λ 2

 d 2ψ 

dx 2= ±

8π2me

h2( E ± V ) ψ  ⇒

±h2

8π2me

d 2ψ 

dx 2+ V ψ = E ψ 

(1)

(2)

(3)

(4)

 E = energía total, E c = energía cinética, V = energía potencial

La ecuación (4) es la ecuación deSchrödinger en un espaciomonodimensional que en un espaciotridimensional se convierte en (5), quese suele abreviar como (6) o (7).

 ±h2

8π2me

∂2ψ 

∂ x 2+

∂2ψ 

∂ y2+

∂2ψ 

∂ z2+ V ψ = E ψ  (5)

 ±h2

8π2me

∇2ψ + V ψ = E ψ  (6) ∇2 = ∂

2

∂ x 2+ ∂

2

∂ y2 + ∂

2

∂ z2

 H ψ = E ψ  (7) H  = ±h2

8π2m

∂2

∂ x 2+∂

2

∂ y2+∂

2

∂ z2+ V 

Sentido físico de la función de onda (interpretación de Born). La función de onda ψ no tiene sentido

físico directo. Sin embargo, el valor de su cuadrado ψ 2 en un punto representa la densidad de

 probabilidad  en dicho punto y el producto ψ 2dV , la  probabilidad de encontrar el electrón dentro del

volumen infinitesimal dV . La probabilidad de encontrarlo en un volumen V es

  ψ 2dV 

V  Las condiciones de frontera y los números cuánticos. Una ecuación como la de Schrödinger tiene un

número infinito de soluciones, pero todas ellas son funciones continuas y sin puntas, pues tienen que

tener derivadas primera y segunda. Además, el sentido físico de ψ 2 exige que ψ sea finita y con un único

valor en cada uno de sus puntos. También, para que ψ 2dV dé directamente la probabilidad, es necesario

normalizar la función (multiplicándola por el valor adecuado) para que se cumpla que:

  ψ 2dV 

todo el espacio= 1

Este tipo de limitaciones se llaman condiciones de frontera y justifican la cuantización. Por ejemplo,

exigen que la amplitud de un onda asociada a una partícula encerrada en un espacio finito sea nula en losextremos. Por ello (figura 1.9), en un espacio de una dimensión, cada onda puede caracterizarse mediante

un número entero. La relación postulada por Bohr entre el número cuántico y el momento angular sededuce de la consideración de un espacio de una dimensión circular (recuadro 1.6). Evidentemente, unelectrón que se mueva en un espacio de tres dimensiones debe poseer tres números cuánticos.

Universidad de Alcalá Tema 1: La estructura electrónica de los átomos | 11

Page 10: Enlace Quimico Y Estructura de Materia

7/22/2019 Enlace Quimico Y Estructura de Materia

http://slidepdf.com/reader/full/enlace-quimico-y-estructura-de-materia 10/89

n = 1 n = 2 n = 3Figura 1.9. En un espacio de una dimensión, lasondas estacionarias estables tienen que tener unaamplitud nula en los extremos y se puedencaracterizar mediante un número cuántico.

Recuadro 1.6. El modelo de Bohr y el carácter de onda estacionaria del electrón

Tal como se muestra a continuación, el carácter de onda del electrón justifica la cuantización y el valor del momentoangular postulado por Bohr.

n = 3

Suponiendo un electrón que gira en una órbita circular estacionaria es necesario que lalongitud de la circunferencia contenga un número entero de longitudes de onda:

2πr = nλ 

La longitud de onda y el momento del electrón están relacionados por la ecuación de DeBroglie:

λ = h/(mv)Igualando el valor de la longitud de onda en ambas ecuaciones se obtiene el valor delmomento angular postulado por Bohr:

2πr = nh/(mv) ⇒ mvr = nh/2π

1.5 El átomo de hidrógeno en el modelo mecano-cuántico. Forma de los orbitales atómicos

Coordenadas polares esféricas. En el átomo de hidrógeno, o en cualquier átomo o ion monoelectrónico,

el electrón se mueve en un campo esférico cuyo valor depende de la distancia r al núcleo. En estos casos,

es más conveniente un sistema de coordenadas polares esféricas (figura 1.10) que un sistema cartesiano. z 

 y

 x

r θ

φ

Figura 1.10. Un punto en un espacio tridimensional sedefine mediante tres coordenadas. Las siguientesecuaciones relacionan las coordenadas cartesianas ( x , y, z)con las coordenadas polares esféricas (r , θ , φ ):

 z = r cos θ  r 2= x 2+ y2+ z2

 x = r sen θ cosφ θ = arcos( z/r ) y = r sen θ sen φ φ = arctg( y/ x )

Orbitales atómicos del hidrógeno. La resolución completa de la ecuación de ondas para un átomo mono-

electrónico introduce tres números cuánticos (n, l y ml) análogos a los descritos en el modelo de Bohr

(tabla 1.3). Cada trío de valores n, l y ml define un estado electrónico del hidrógeno, que es descrito por

una función de onda ψ n,l,ml, tiene una energía bien definida y recibe el nombre de orbital atómico. Los

números cuánticos de espín (s y ms) sólo aparecen cuando se modifica la ecuación de Schrödinger para

hacerla coherente con la teoría de la relatividad (Dirac).

La resolución de la ecuación de Schrödinger para el átomo de hidrógeno puede consultarse en

www2.uah.es/edejesus/resumenes/EQEM/ 

 Energía de los orbitales. En el átomo de hidrógeno libre y sin perturbar por campos externos, el valor de

la energía obtenido de la ecuación de Schrödinger coincide con el del modelo de Bohr:

  E n = ±

mee4

8h2ε 0

2

 Z 2

n2

Energía de los orbitales de un átomo o ion monoelectrónico decarga nuclear Z . En el hidrógeno Z = 1.

Los orbitales del mismo número cuántico principal n están degenerados: tienen la misma energía

 Forma de los orbitales. Las funciones ψ n,l,mlexpresadas en coordenadas polares (r , θ , ϕ ), se pueden

descomponer en una parte radial (función del radio) y otra angular  (función de los ángulos), simplifi-cando su análisis: ψ n,l,ml

(r , θ , ϕ ) = Rn,l(r ) Al,ml(θ , ϕ ). La parte radial no depende de ml mientras que la

angular no depende de n. Las tablas 1.5 y 1.6 tabulan las funciones R y A, respectivamente. En las figuras

1.11 y 1.12 se estudia la distribución electrónica espacial de algunos orbitales del átomo de hidrógeno.

12 | Enlace químico y estructura de la materia Licenciatura en Química

Page 11: Enlace Quimico Y Estructura de Materia

7/22/2019 Enlace Quimico Y Estructura de Materia

http://slidepdf.com/reader/full/enlace-quimico-y-estructura-de-materia 11/89

Page 12: Enlace Quimico Y Estructura de Materia

7/22/2019 Enlace Quimico Y Estructura de Materia

http://slidepdf.com/reader/full/enlace-quimico-y-estructura-de-materia 12/89

Page 13: Enlace Quimico Y Estructura de Materia

7/22/2019 Enlace Quimico Y Estructura de Materia

http://slidepdf.com/reader/full/enlace-quimico-y-estructura-de-materia 13/89

   4

   1   2

   1   0

   r   (    Å   )

   2

   6

   8

   4

   1   2

   1   0

   r   (    Å   )

   2

   6

   8

   r   (    Å   )

   4

   1   2

   1   0

   2

   6

   8

   4

   1   2

   1   0

   r   (    Å   )

   2

   6

   8

   4

   1   2

   1   0

   r   (    Å

   )

   2

   6

   8

   4

   1   2

   1   0

   r   (    Å   )

   2

   6

   8

   4

   1   2

   1   0

   r   (    Å   )

   2

   6

   8

   4

   1   2

   1   0

   r   (    Å   )

   2

   6

   8

   r   (    Å   )

   4

   1   2

   1   0

   r   (    Å   )

   2

   6

   8

   4

   1   2

   1   0

   2

   6

   8

   4

   1   2

   1   0

   r   (

    Å   )

   2

   6

   8

   4

   1   2

   1   0

   r   (

    Å   )

   2

   6

   8

   2  p

   4  p

   3  p

   3   d

   4   d    4   f

     d   x   y

   y

   x

   z   z

   x

   y     d

   y   z

   z

   x

   y

     d   z

   2

   z

   y

   x

     d   x   2  -   y

   2

     d   x   z

   y

   x

   z

   p   x

   p   z

    p      y     y

   x

   z

   y

   x

   z   z

   x

   y

    R   n ,  l  (   r  ) 4      π   r   2

    R   2   n ,  l  (   r  )

   4      π   r   2

    R   2   n ,  l  (   r  )    R   n ,  l  (   r  )

   P  a  r   t  e   A  n  g

  u   l  a  r

   P  a  r   t  e   R  a   d   i  a   l R   n ,  l  (   r  ) 4      π   r

   2    R   2

   n ,  l  (   r  )

   F   i  g  u  r  a   1 .   1

   2 .

   R  e  p  r  e  s  e  n   t  a  c   i   ó  n

   d  e   l  a  s  s  o   l  u  c   i  o

  n  e  s   d  e   d  e   l  a

  e  c  u  a  c   i   ó  n   d  e   S

  c   h  r   ö   d   i  n  g  e  r

  c  u  a  n   d  o     l     ≠    0

   (  o  r   b   i   t  a   l  e  s   p ,

     d  y     f   ) .

  •   E  n   l  a  p  a  r   t  e   i  z  q  u   i  e  r   d  a   d  e   l  a   f   i  g  u  r  a

  s  e  r  e  p  r  e  s  e  n   t  a

     A   2

   p  a  r  a   l  o  s  o  r   b   i   t  a   l  e  s   p

  y     d   (   l  o  s  o  r   b

   i   t  a   l  e  s     f

  n  o  s  e  m  u  e  s   t  r  a

  n   ) .   E  n   l  o  s   l   ó   b  u   l  o  s  e  n   b   l  a  n  c  o     A   e

  s

  p  o  s   i   t   i  v  a ,  m

   i  e  n

   t  r  a  s  q  u  e  e  n   l  o  s  o  s  c  u  r  o  s  e  s  n  e  g  a   t   i  v  a

 .

  •   P  a  r  a   l  o  s  o  r   b

   i   t  a   l  e  s  q  u  e  n  o  s  o  n   s ,

   l  a  p  a  r   t  e  a  n  g  u   l  a

  r  n  o  e  s

  e  s   f   é  r   i  c  a  :   l  o  s  o

  r   b   i   t  a   l  e  s   p ,

     d ,

     f ,  e   t  c .  s  o  n   d   i  r  e  c  c   i  o  n  a   l  e  s .

  •   L  a  p  a  r   t  e  a  n  g

  u   l  a  r   d  e  c  a   d  a  o  r   b   i   t  a   l   t   i  e  n  e     l  p   l  a  n  o  s

  n  o   d  a   l  e  s .

   L  a  p  a  r   t  e  r  a   d   i  a

   l   t   i  e  n  e   n –

     l –   1  s  u  p  e  r   f   i  c   i  e  s  n  o   d  a   l  e  s  e

  s   f   é  r   i  c  a  s .

   E  n   t  o   t  a   l ,  u  n  o

  r   b   i   t  a   l   t   i  e  n  e   n –

   1  s  u  p  e  r   f   i  c   i  e  s  n  o   d  a   l  e  s .

Universidad de Alcalá Tema 1: La estructura electrónica de los átomos | 15

Page 14: Enlace Quimico Y Estructura de Materia

7/22/2019 Enlace Quimico Y Estructura de Materia

http://slidepdf.com/reader/full/enlace-quimico-y-estructura-de-materia 14/89

Orbitales atómicos hidrogenoides. Al escribir las soluciones de la ecuación de ondas, hemos tenido el

cuidado de no sustituir Z por su valor para el hidrógeno,que es 1, de forma que dichas soluciones son

igualmente válidas para cualquier ion monoelectrónico. Obsérvese que el aumento de la carga nuclear:

• disminuye la energía de los orbitales (ver ecuación),

• modifica la parte radial (tabla 1.6), resultando en una mayor contracción de los orbitales, pero

• no modifica la forma de los orbitales, ya que la parte angular no depende de Z (tabla 1.7).Estos orbitales, de la igual forma pero de distinta energía y tamaño que los del hidrógeno, reciben el

nombre genérico de orbitales atómicos hidrogenoides.

 Difusión y penetración de orbitales. La difusión de un orbital (figura 1.13a) es importante a la hora de

discutir el enlace químico (tema 3) mientras que la penetración (figura 1.13b) permite comprender cómo

varían las energías de los orbitales en los átomos polielectrónicos (ver 1.7).

2 p 2sb)

4 1210r (Å)

2 6 8

4πr 2 R2n,l(r )

4πr 2 R2n,l(r )

1s

a)2s Figura 1.13. a) El orbital 2s es más difuso que el 1s, su densidad

electrónica está más extendida. La difusión de los orbitales aumentacon su número cuántico principal.

b) El orbital 2s tiene una mayor concentración electrónica en zonasmuy cercanas al núcleo que el 2 p, por lo que se dice que el orbital2s es más penetrante que el 2 p. El orden de penetración paraorbitales del mismo n es s>p>d>f . Lógicamente, los orbitales demenor n son más penetrantes.

 Momentos angulares. Los números cuánticos l y s definen el valor de los momentos angulares de orbital

y de espín que, dentro de un modelo clásico (no mecano-cuántico), se asocian respectivamente al giro del

electrón en torno al núcleo y en torno de su propio eje (Figura 1.14).

l s

 Ze + e –Figura 1.14. Representación clásica delorigen de los momentos angularesorbital y de espín.

 l = número cuántico azimutal

l = momento angular orbitall = l l + 1 (h/2π)

s = número cuántico de espín

s = momento angular de espíns = s s + 1 (h/2π)

Según la mecánica clásica, el momento angular podría orientarse de cualquier forma con respecto a una

dirección dada. La mecánica cuántica predice que para los momentos angulares de orbital y de espín solo

son posibles las orientaciones definidas por los números cuánticos ml y ms (figura 1.15).

1.6 Los átomos polielectrónicosLa ecuación de Schrödinger es irresoluble de forma exacta para átomos de más de un electrón, y no es

por su complejidad, pues lo son siempre las ecuaciones, por sencillas que sean, de cualquier sistema

físico de más de dos partículas interaccionando entre sí, por ejemplo, Sol-Tierra-Luna. Situando el Sol en

el centro de coordenadas, la irresolubilidad es debida a que no es posible separar las coordenadas que

definen la trayectoria de la Tierra de aquéllas que definen la de la Luna, pues ambas se afectan

mutuamente. En otras palabras, no se puede conocer la trayectoria de la Tierra sin conocer previamente la

de la Luna y viceversa. Una forma de obtener una solución, aunque aproximada, es despreciar alguna

interacción. Así se puede obtener una trayectoria aproximada de la Tierra alrededor del Sol despreciando

la influencia de la Luna, y la de la Luna alrededor de la Tierra despreciando la del Sol. Obsérvese que seha transformado un problema irresoluble de tres partículas (Sol-Tierra-Luna) en dos problemas

resolubles, pero aproximados, de dos partículas (Sol-Tierra y Tierra-Luna).

16 | Enlace químico y estructura de la materia Licenciatura en Química

Page 15: Enlace Quimico Y Estructura de Materia

7/22/2019 Enlace Quimico Y Estructura de Materia

http://slidepdf.com/reader/full/enlace-quimico-y-estructura-de-materia 15/89

+2

+1

0

 –1

 –2

 z

(h/2π)m = +2

l

m = +1l

m = 0l

m = –1l

m = –2l

(a)

l = 2 | l | = 2,45 (h/2π)

+1/2

0

 –1 /2

 z

(h/2π)m = +1/2

s

m = –1/2s

(b)

| s | = 0,87 (h/2π)

(c)

0

Figura 1.15. Los números cuánticos magnéticos de orbital y de espín ml y ms definen el módulo de la componente delmomento angular de orbital y de espín, respectivamente, en el eje de referencia (eje z). La relación entre el módulo del vectorcomponente y el número cuántico es:

 ml = número cuántico magnético orbital

mm l l = proyección del momento angular 

orbital en el eje de referencia

mm l l  = m l (h/2π )  ms = número cuántico magnético de espín

mms s = proyección del momento angular 

de espín en el eje de referencia

mms s  = ms (h/2π )

(a) Orientaciones posibles para el momento angular orbital de un electrón en un orbital de l = 2.(b) Orientaciones posibles para el momento angular de espín de un electrón.(c) La orientación así definida no es única sino en realidad un cono de orientaciones.

 La aproximación orbital. En un átomo polielectrónico, el segundo electrón y siguientes introducen la

repulsión entre electrones, ausente en el átomo de hidrógeno (figura 1.16). La aproximación orbital es un

método de resolución que consiste en despreciar las repulsiones interelectrónicas, con lo que los

electrones se independendizan unos de otros, comportándose cada uno de ellos tal como lo haría en un

sistema monoelectrónico de carga nuclear Z . Por esta vía, el problema irresoluble de un átomo con n

electrones se convierte en n problemas resolubles de 1 sólo electrón, cuyas soluciones conocemos ya, sonlos orbitales atómicos hidrogenoides. El resultado es que el estado de un átomo polielectrónico se apro-

xima mediante la superposición de estados individuales de los electrones u orbitales atómicos, y la

energía total mediante la suma de las energías de cada uno de dichos estados monoelectrónicos.

Núcleo Ze+

electrón 1

e-

e-electrón 2

r 1

r 2

r 12

 Z = 2 para Helio

Figura 1.16. El átomo de helio

La resolución de la ecuación de Schrödinger para átomos polielectrónicos, dentro de la aproximación

orbital, puede consultarse en www2.uah.es/edejesus/resumenes/EQEM/ 

El nombre de aproximación orbital procede, por tanto, de que introduce en los átomos polielectró-

nicos la idea de orbital como estado de un electrón en el átomo. Ahora bien, un orbital atómico del

hidrógeno es una solución exacta de la ecuación de ondas que representa uno de los estados de energía enque puede estar el átomo debido a su único electrón. Un orbital en un átomo polielectrónico representa el

estado de energía de uno de sus electrones, y es el resultado de una aproximación , que desprecia las

repulsiones interelectrónicas y permite hablar en términos de electrones independientes.

Universidad de Alcalá Tema 1: La estructura electrónica de los átomos | 17

Page 16: Enlace Quimico Y Estructura de Materia

7/22/2019 Enlace Quimico Y Estructura de Materia

http://slidepdf.com/reader/full/enlace-quimico-y-estructura-de-materia 16/89

Configuración electrónica. Una configuración electrónica es la distribución de los electrones en los

orbitales. Una configuración con 4 electrones, dos en orbitales 1s y dos en orbitales 2s se simboliza 1s2

2s2. El estado de mínima energía de un sistema cuántico se llama estado fundamental, y el resto, estados

excitados. La configuración electrónica fundamental de un átomo es la del estado de mínima energía.

 El principio de Aufbau o de construcción. Del modelo de la aproximación orbital se infiere de forma

lógica el llamado principio de Aufbau según el cual “la configuración electrónica fundamental se obtienellenando los orbitales por orden creciente de energía.”

 El principio de exclusión de Pauli . La configuración fundamental de un átomo de, por ejemplo, 6

electrones no es 1s6. Los datos experimentales indican que sólo puede haber un máximo de dos

electrones en cada orbital, cada uno con un espín distinto, o, en otras palabras, “una configuración no

puede contener dos electrones con idénticos valores en todos sus números cuánticos.” Ésta es una formu-

lación particular del principio de exclusión de Pauli, un principio fundamental de la mecánica cuántica.

Carga nuclear efectiva. ¿Podemos mejorar la descripción electrónica del átomo sin perder la simplicidad

de la construcción orbitalaria? La construcción orbitalaria se introdujo más arriba como el resultado de

despreciar las interacciones interelectrónicas y suponer que cada electrón siente únicamente el campo

esférico y centrado en el núcleo creado por la carga nuclear Z . Ahora bien, se puede mejorar el modelo

sustituyendo la carga nuclear Z por una carga nuclear efectiva Z * que considera no sólo la atracción

nuclear sino también la repulsión del resto de electrones (figura 1.17). Tal como se explica en la figura,

las repulsiones se introducen sólo parcialmente pues debe suponerse que cada electrón ve al resto como

una nube media en el tiempo y esférica con el objeto de mantener la construcción orbitalaria.

Núcleo Z +

electrón 1

e-

ApantallamientoS 

Figura 1.17. El campo eléctrico que siente 1 es esférico siempreque “vea” al resto de electrones como una nube media en el

tiempo, esférica y centrada en el núcleo. El electrón 1 siente la

atracción de la carga nuclear Z . La porción de nube situada máscerca del núcleo que el electrón 1, produce sobre él un efectoneto de repulsión hacia afuera, que podemos equiparar al queproduciría una carga negativa S (apantallamiento) colocada enel núcleo. El efecto neto sobre 1 es el mismo que si fuera elúnico electrón de un átomo de carga nuclear Z* = Z  – S . A Z* sele llama carga nuclear efectiva.

Recordemos que la carga nuclear  Z  afecta al tamaño y energía de un orbital pero no a su forma.

Tendremos que tener en cuenta que se obtiene una mejor estimación del tamaño y energía de un orbital si

se utiliza su carga nuclear efectiva Z * en lugar de la carga nuclear real  Z  (obsérvese que  Z * no es un

parámetro del átomo sino que toma un valor distinto para cada orbital). En el recuadro 1.9 se describe unmétodo cuantitativo que usa el modelo de la carga nuclear efectiva.

En www2.uah.es/edejesus/resumenes/EQEM/, puede encontrarse la descripción del método SCF de

Hartree-Fock, un método cuantitativo que usa el modelo de la carga nuclear efectiva.

Valores de la carga nuclear efectiva. Los valores de carga nuclear efectiva calculados por el método

SCF de Hartree–Fock permiten prever algunas propiedades de los átomos (tabla 1.7).

18 | Enlace químico y estructura de la materia Licenciatura en Química

Page 17: Enlace Quimico Y Estructura de Materia

7/22/2019 Enlace Quimico Y Estructura de Materia

http://slidepdf.com/reader/full/enlace-quimico-y-estructura-de-materia 17/89

_____________________________________________________________________________________________________Tabla 1.7. Cargas Nucleares Efectivas de los elementos del 1 al 36 

 Elemento 1s 2s 2 p 3s 3 p 4s 3d  4 pH 1,000He 1,688Li 2,691 1,279Be 3,685 1,912

B 4,680 2,576 2,421C 5,673 3,217 3,136N 6,665 3,847 3,834O 7,658 4,492 4,453F 8,650 5,128 5,100Ne 9,642 5,758 5,758Na 10,626 6,571 6,802 2,507Mg 11,619 7,392 7,826 3,308Al 12,591 8,214 8,963 4,117 4,066Si 13,575 9,020 9,945 4,903 4,285P 14,558 9,825 10,961 5,642 4,886S 15,541 10,629 11,977 6,367 5,482Cl 16,524 11,430 12,993 7,068 6,116Ar 17,508 12,230 14,008 7,757 6,764K 18,490 13,006 15,027 8,680 7,726 3,495Ca 19,473 13,776 16,041 9,602 8,658 4,398Sc 20,457 14,574 17,055 10,340 9,406 4,632 7,120Ti 21,441 15,377 18,065 11,033 10,104 4,817 8,141V 22,426 16,181 19,073 11,709 10,785 4,981 8,983Cr 23,414 16,984 20,075 12,368 11,466 5,133 9,757Mn 24,396 17,794 21,084 13,018 12,109 5,283 10,528Fe 25,381 18,599 22,089 13,676 12,778 5,434 11,180Co 26,367 19,405 23,092 14,322 13,435 5,576 11,855Ni 27,353 20,213 24,095 14,961 14,085 5,711 12,530Cu 28,339 21,020 25,097 15,594 14,731 5,858 13,201Zn 29,325 21,828 26,098 16,219 15,369 5,965 13,878Ga 30,309 22,599 27,091 16,996 16,204 7,067 15,093 6,222

Ge 31,294 23,365 28,082 17,760 17,014 8,044 16,251 6,780As 32,278 24,127 29,074 18,596 17,850 8,944 17,378 7,449Se 33,262 24,888 30,065 19,403 18,705 9,758 18,477 8,287Br 34,247 25,643 31,056 20,218 19,571 10,553 19,559 9,028Kr 35,232 26,398 32,047 21,033 20,434 11,316 20,626 9,769_____________________________________________________________________________________________________

 Reglas de Slater. Slater construyó una serie de reglas para determinar los valores de apantallamiento (S )

que permiten calcular cargas nucleares efectivas ( Z * = Z  – S ) que se corresponden aceptablemente con las

obtenidas mediante cálculos SCF. Estas reglas, apropiadas para electrones hasta el subnivel 3d , son:

1 Se dividen los electrones en el átomo en los siguientes grupos: (1s)(2s,2 p)(3s,3 p)(3d )

2 La constante de apantallamiento S para un orbital asociado con alguno de los anteriores grupos es la

suma de las siguientes contribuciones:(a) nada de cualquier grupo a la derecha del grupo considerado

(b) 0,35 de cada electrón en el grupo (0,30 si es un grupo 1s)

(c) Para un orbital s o p, 0,85 por cada electrón en el nivel n –1, 1,00 para cada electrón más interno.

Para un orbital d , 1,00 para todos los electrones más internos.

Si bien los valores de cargas nucleares efectivas ( Z *) obtenidos por las reglas de Slater son menos exac-

tos que los recogidos en la tabla 1.9, estas reglas resumen la forma en que se apantallan entre sí los

electrones situados en distintos orbitales: los electrones en orbitales más penetrantes apantallan bien a los

electrones en orbitales menos penetrantes. Así, los electrones en orbitales 1s, 2s, 2 p, 3s y 3 p contribuyen

en 1,00 al apantallamiento de los electrones en orbitales 3d . En cambio la contribución al apantallamiento

de un electrón 3 p es nula por parte de un electrón en un orbital 3d , 0,35 por uno 3s y 0,85 por uno 2s o

2 p, de acuerdo con el orden de penetración de los orbitales: 3s > 3 p >3d . Obsérvese, sin embargo, que en

estas reglas simplificadas, un orbital 3 p es igualmente apantallado por uno 3s que viceversa.

Universidad de Alcalá Tema 1: La estructura electrónica de los átomos | 19

Page 18: Enlace Quimico Y Estructura de Materia

7/22/2019 Enlace Quimico Y Estructura de Materia

http://slidepdf.com/reader/full/enlace-quimico-y-estructura-de-materia 18/89

Recuadro 1.7. Ejemplo de cálculo de carga nuclear efectiva

En el átomo de potasio, cuyo número atómico es 19, la distribución de 18 de sus 19 electrones es la siguiente: 1s2 2s2

2 p6 3s2 3 p6. Calcula la carga nuclear efectiva sobre el electrón restante si se coloca en un orbital (a)3d , (b) 4s.Solución: Agrupando los electrones de la forma indicada en la regla 1, queda (1s)2 (2s, 2 p)8 (3s, 3 p)8.En el caso (a), S = 8 × 1,00 + 8 × 1,00 + 2 × 1,00 = 18, por lo que  Z * = Z  – S = 19 – 18 = 1,00.En el caso (b), S = 8 × 0,85 + 8 × 1,00 + 2 × 1,00 = 16,8, por lo que  Z * = Z  – S = 19 – 16,8 = 2,20.

¡El electrón será más fuertemente atraído por el núcleo si se coloca en el orbital 4 s que si lo hace en el 3d !

Conclusión. La aproximación orbital es una manera imperfecta pero útil de describir la estructura

atómica y de entender, predecir y calcular resultados experimentales. Hay maneras más precisas pero

normalmente más difíciles de interpretar. Pero siempre hay que estar atento a la posibilidad de que

existan diferencias significativas entre el sistema real y el modelo matemático para este sistema. En la ta-

bla 1.8 se comparan resultados experimentales con calculados por diversos métodos.

Tabla 1.8. Energías calculadas para el helio en su estado fundamental a partir de funciones de onda aproximadasTipo de función de onda Energía (en eV)

1 Producto de orbitales del He+

(aprox. orbital)  –74,832 Producto de orbitales con Z * fijada por el método SCF  –77,483 Producto de funciones más apropiado  –77,8709174 Función no orbitalaria de Pekeris (combinación lineal de 1078 términos)  –79,00946912

Valor experimental ( I 1 + I 2) –79,014

1.7 Configuraciones electrónicas de los átomos polielectrónicos en su estado fundamental

Orden de llenado de orbitales. Para cada tipo de orbital y como consecuencia de su diferente

penetración, la carga nuclear efectiva que sobre él ejerce el núcleo y su energía varían de forma distinta

al aumentar el número atómico (figura 1.18). A veces, se altera el orden relativo de sus energías y, por

tanto, el de su llenado. Generalmente, pero no siempre, los orbitales se llenan en el orden1s < 2s < 2 p < 3s < 3 p < 4s < 3d < 4 p < 5s < 4d < 5 p < 6s < 4 f < 5d < 6 p < 7s < 5 f < 6d < 7 p …

3d 

4s 4 pK

Ca

ScTi

V

Número atómico, Z 20 50 75 100

1

2

3

4

5

5 p

5s4 f 4d 4 p4s3d 3 p3s2 p2s1s

   E  n  e  r  g   í  a  r  e   l  a   t   i  v

  a   d  e   l  o  s  o  r   b   i   t  a   l  e  s

n

 Regla de Hund (primera versión). “Para una configuración dada, el estado más estable es aquél con el

máximo número de electrones desapareados”Configuraciones electrónicas. Las reglas anteriores predicen correctamente las configuraciones

electrónicas experimentales de los átomos polielectrónicos en su estado fundamental (tabla 1.9), salvo las

excepciones entre los metales de transición detalladas a continuación:

20 | Enlace químico y estructura de la materia Licenciatura en Química

Figura 1.18. Niveles de energía de los átomospolielectrónicos en la tabla periódica. Se muestra unavista ampliada del orden de energías alrededor de Z =20.En el átomo de hidrógeno, la energía de los orbitalesdepende sólo de n. Al aumentar el número atómico, losorbitales más penetrantes (s>p>d>f ) son menosapantallados por los electrones más internos y sientenmás fuertemente el aumento de la carga nuclear. Enconsecuencia, su energía decae más rápidamente.Obsérvese como se producen alteraciones en el orden deenergía de los orbitales. Así, en la zona expandida, elorbital 4s tiene en K y Ca menos energía que el 3d .

Page 19: Enlace Quimico Y Estructura de Materia

7/22/2019 Enlace Quimico Y Estructura de Materia

http://slidepdf.com/reader/full/enlace-quimico-y-estructura-de-materia 19/89

• Muchos casos se pueden justificar por la pequeña diferencia de energía entre los orbitales (n –1)d y ns:

Nb [Kr]4d 45s1 Ru [Kr]4d 75s1 Rh [Kr]4d 85s1 Pt [Xe]4 f 145d 96s1

• El trasvase de electrones del orbital ns al (n –1)d  puede verse favorecido por la estabilidad de las

configuraciones esféricas (estados con todos los subniveles completos, completos a mitad o vacíos):

Cr [Ar]3d 54s1 Cu [Ar]3d 104s1 Mo [Kr]4d 55s1 Pd [Kr]4d 10

Ag [Kr]4d 10

5s1

Au [Xe]4 f 14

5d 10

6s1

• Razonamientos similares se pueden hacer en lantánidos y actínidos para excepciones análogas con

orbitales (n –2) f y (n –1)d .

La [Xe]5d 16s2 Ce [Xe]4 f 15d 16s2 Gd [Xe]4 f 75d 16s2

Ac [Rn]6d 17s2 Th [Rn]6d 27s2 Pa [Rn]5 f 26d 17s2 U [Rn]5 f 36d 17s2

Np [Rn]5 f 46d 17s2 Cm [Rn]5 f 76d 17s2

• Al formar cationes, los electrones se sacan primero de los orbitales np, luego ns y finalmente (n –1)d .

Mn [Ar]3d 54s2 Mn2+ [Ar]3d 5 Ga [Ar]3d 104s24 p1 Ga3+ [Ar]3d 10_____________________________________________________________________________________________________Tabla 1.9. Configuraciones electrónicas de los átomos en el estado fundamental*

 Z Sím. Configuración Z Sím. Configuración Z Sím. Configuración

1 H 1s1 35 Br [Ar]3d 104s24 p5 69 Tm [Xe]4 f 136s2

2 He 1s2 36 Kr [Ar]3d 104s24 p6 70 Yb [Xe]4 f 146s2

3 Li [He]2s1 37 Rb [Kr]5s1 71 Lu [Xe]4 f 145d 16s2

4 Be [He]2s2 38 Sr [Kr]5s2 72 Hf [Xe]4 f 145d 26s2

5 B [He]2s22 p1 39 Y [Kr]4d 15s2 73 Ta [Xe]4 f 145d 36s2

6 C [He]2s22 p2 40 Zr [Kr]4d 25s2 74 W [Xe]4 f 145d 46s2

7 N [He]2s22 p3 41 Nb [Kr]4d 45s1 75 Re [Xe]4 f 145d 56s2

8 O [He]2s22 p4 42 Mo [Kr]4d 55s1 76 Os [Xe]4 f 145d 66s2

9 F [He]2s22 p5 43 Tc [Kr]4d 55s2 77 Ir [Xe]4 f 145d 76s2

10 Ne [He]2s22 p6 44 Ru [Kr]4d 75s1 78 Pt [Xe]4 f 145d 96s1

11 Na [Ne]3s1 45 Rh [Kr]4d 85s1 79 Au [Xe]4 f 145d 106s1

12 Mg [Ne]3s2 46 Pd [Kr]4d 10 80 Hg [Xe]4 f 145d 106s2

13 Al [Ne]3s23 p1 47 Ag [Kr]4d 105s1 81 Tl [Xe]4 f 145d 106s26 p1

14 Si [Ne]3s23 p2 48 Cd [Kr]4d 105s2 82 Pb [Xe]4 f 145d 106s26 p2

15 P [Ne]3s23 p3 49 In [Kr]4d 105s25 p1 83 Bi [Xe]4 f 145d 106s26 p3

16 S [Ne]3s23 p4 50 Sn [Kr]4d 105s25 p2 84 Po [Xe]4 f 145d 106s26 p4

17 Cl [Ne]3s23 p5 51 Sb [Kr]4d 105s25 p3 85 At [Xe]4 f 145d 106s26 p5

18 Ar [Ne]3s23 p6 52 Te [Kr]4d 105s25 p4 86 Rn [Xe]4 f 145d 106s26 p6

19 K [Ar]4s1 53 I [Kr]4d 105s25 p5 87 Fr [Rn]7s1

20 Ca [Ar]4s2 54 Xe [Kr]4d 105s25 p6 88 Ra [Rn]7s2

21 Sc [Ar]3d 14s2 55 Cs [Xe]6s1 89 Ac [Rn]6d 17s2

22 Ti [Ar]3d 24s2 56 Ba [Xe]6s2 90 Th [Rn]6d 27s2

23 V [Ar]3d 34s2 57 La [Xe]5d 16s2 91 Pa [Rn]5 f 26d 17s2

24 Cr [Ar]3d 54s1 58 Ce [Xe]4 f 15d 16s2 92 U [Rn]5 f 36d 17s225 Mn [Ar]3d 54s2 59 Pr [Xe]4 f 36s2 93 Np [Rn]5 f 46d 17s2

26 Fe [Ar]3d 64s2 60 Nd [Xe]4 f 46s2 94 Pu [Rn]5 f 67s2

27 Co [Ar]3d 74s2 61 Pm [Xe]4 f 56s2 95 Am [Rn]5 f 77s2

28 Ni [Ar]3d 84s2 62 Sm [Xe]4 f 66s2 96 Cm [Rn]5 f 76d 17s2

29 Cu [Ar]3d 104s1 63 Eu [Xe]4 f 76s2 97 Bk [Rn]5 f 97s2

30 Zn [Ar]3d 104s2 64 Gd [Xe]4 f 75d 16s2 98 Cf [Rn]5 f 107s2

31 Ga [Ar]3d 104s24 p1 65 Tb [Xe]4 f 96s2 99 Es [Rn]5 f 117s2

32 Ge [Ar]3d 104s24 p2 66 Dy [Xe]4 f 106s2 100 Fm [Rn]5 f 127s2

33 As [Ar]3d 104s24 p3 67 Ho [Xe]4 f 116s2 101 Md [Rn]5 f 137s2

34 Se [Ar]3d 104s24 p4 68 Er [Xe]4 f 126s2 102 No [Rn]5 f 147s2

* Estas configuraciones han sido obtenidas experimentalmente para los átomos en fase gaseosa._____________________________________________________________________________________________________

1.8 Estados atómicos de energía. Términos de Russell–Saunders

Al mencionar la regla de Hund hemos señalado que los electrones de una configuración pueden a veces

disponerse de varias formas en los orbitales de la misma subcapa. Por ejemplo, los dos electrones de una

Universidad de Alcalá Tema 1: La estructura electrónica de los átomos | 21

Page 20: Enlace Quimico Y Estructura de Materia

7/22/2019 Enlace Quimico Y Estructura de Materia

http://slidepdf.com/reader/full/enlace-quimico-y-estructura-de-materia 20/89

configuración p2 pueden distribuirse en los tres orbitales p de 15 formas distintas, llamadas microestados

(ver recuadro 1.8). En la aproximación orbital, la energía total de una configuración es la suma de las

energías de los orbitales ocupados (ver 1.6) y es por tanto idéntica para todos sus microestados. Sin

embargo, la realidad es que de los tres microestados siguientes, el (b) es más estable, tal como predice la

regla de Hund.

ml +1 0 –1 +1 0 –1 +1 0 –1

a) b) c)

Recuadro 1.8. Número de microestados de una configuraciónp 2

2 electrones

en

6 "casillas"

l = +1

ml = –10+1

ms= +1/2  –1/2 +1/2  –1/2 +1/2  –1/2

Uno de los microestados posibles para unaconfiguración p2. El número total de microestados deuna configuración p2 es igual al número decombinaciones de 6 elementos tomados de 2 en 2:

 62 =

6!(6 - 2)! 2! =

302 = 15 microestados

Una de las razones por las que (b) es más estable que (a) es porque la repulsión interelectrónica es menor

al estar los dos electrones en distintos orbitales, es decir, en zonas diferentes del espacio. El modelo

orbitalario no predice estas diferencias de energía porque calcula la energía de cada electrón considerando

una distribución de densidad media y esférica del resto de electrones. Otra razón la podemos encontrar en

las interacciones magnéticas, no consideradas hasta ahora, que también contribuyen, aunque en menor

medida que las electrostáticas, a la energía total. Los microestados (a–c) difieren en la orientación relativa

de sus momentos orbitales, en paralelo en (a) y antiparalelo en (c), y de espín, en paralelo en (b) pero en

antiparalelo en (a) y (c).

 Acoplamientos LS o Russell–Saunders. La pregunta que vamos a contestar es en cuántos niveles de

energía se dividen los 15 microestados de una configuración p2. Para ello usaremos el esquema llamado

acoplamiento LS o Russell–Saunders. En este esquema se considera primero las formas posibles de

acoplamiento entre los momentos de espín s (figura 1.19a). Cada una de estas formas resulta en un

momento angular de espín total S característico que podemos etiquetar mediante un número cuántico S .

+1

0

 –1

l = 1

 L = 2

 M  = +2, +1, 0, –1, –2 L

l

L

+1

0

 –1

l = 1

 L = 0

 M  = 0 L

l

l

+1

0

 –1

l = 1

 L = 1

 M  = +1, 0, –1 L

l

L

l

s = 1/2

S = 0

 M  = 0S 

+1/2

0

 –1 /2

s = 1/2

S = 1

 M  = +1, 0, –1S 

s

S

0

s

s

+1/2

 –1 /2

s

(a) (b)

Figura 1.19. Consideremos el caso de dos electrones en los orbitales p.(a) Los espines pueden estar mutuamente orientados paralela o antiparalelamente. El momento de espín resultante quedará

caracterizado por el número cuántico S que será 1 en la orientación paralela y 0 en la antiparalela. Las restricciones

cuánticas habituales afectan a S que podrá tener 2S + 1 orientaciones definidas por un número cuántico M S = +S ,…, –S.(b) Los momentos orbitales pueden estar mutuamente orientados de tres maneras, dando un momento angular resultantequedará caracterizado por el número cuántico L que puede valer 2, 1 y 0. Las restricciones cuánticas habituales afectan a

 L que podrá tener 2 L + 1 orientaciones definidas por un número cuántico M  L = + L,…, – L.Al sumar los vectores hay que tomar en consideración que estamos sumando conos de vectores.

22 | Enlace químico y estructura de la materia Licenciatura en Química

Page 21: Enlace Quimico Y Estructura de Materia

7/22/2019 Enlace Quimico Y Estructura de Materia

http://slidepdf.com/reader/full/enlace-quimico-y-estructura-de-materia 21/89

Así, en una configuración p2 los dos electrones pueden tener sus espines en paralelo, dando lugar a un

estado de S  = 1, o en antiparalelo (S  = 0). De la misma forma, existen tres formas distintas

acoplamiento entre los momentos orbitales l (figura 1.19b) caracterizables mediante los números

cuánticos L = 2, 1, 0, según el valor del momento angular orbital total L.

Términos de Russell–Saunders. Un término de Russell–Saunders agrupa a un conjunto de microestados

con el mismo L y el mismo S y se representa como(2S + 1) X 

donde X = S , P, D, F , G, H , I , K ,… cuando L = 0, 1, 2, 3, 4, 5, 6, 7,…, respectivamente.

De la figura 1.19 se deduce que una configuración p2 podría contener 6 términos, resultado de la

combinación de los dos valores posibles para S y los tres para L. Estos términos serían 3 D, 1 D, 3P, 1P, 3S 

y 1S . Sin embargo, un término 3 D ( L = 2 y S = 1) supondría colocar los dos electrones con el mismo espín

en el mismo orbital ¡violando el principio de Pauli! No se abordará aquí ningún método sistemático de

deducción de los términos de una configuración. Baste señalar que los términos de una configuración p2

son 1 D, 3P y 1S .

En www2.uah.es/edejesus/resumenes/EQEM/, puede encontrarse un método sistemático de

deducción de los términos de una configuración.

El símbolo de un término no sólo indica el valor del momento orbital total sino que, al igual que el

símbolo de un orbital, también indica la distribución espacial de la densidad electrónica. Por ejemplo,

todos los microestados de un término S ( L = 0) tienen una distribución electrónica esférica.

Los términos de las configuraciones electrónicas más habituales se recogen en la tabla 1.10. Puede

observarse que algunas configuraciones como la p2 y la p4 contienen los mismos términos. Como regla,

una subcapa de n electrones y, por tanto, de N -n huecos, siendo N la capacidad de la capa, da lugar a los

mismos términos que una capa de N -n electrones.

Tabla 1.10. Términos Russell–Saunders para algunas configuracionesConfiguración Términos Término fundamental  s1 2S  2S

s2 1S  1S p1 y p5 2P 2 Pp2 y p4 1S , 1 D, 3P 3 P

 p3 2P , 2 D, 4S  4S

 p6 1S  1Sd 1 y d 9 2 D 2 Dd 2 y d 8 1S , 1 D, 1G, 3P , 3F  3 Fd 3 y d 7 2P , 2 D(2), 2F , 2G, 2 H , 4P, 4F  4 F

d 4 y d 6 1S (2), 1 D(2), 1F , 1G(2), 1 I , 3P(2), 3 D, 3F (2), 3G, 3 H , 5 D 5 Dd 5 2S , 2P, 2 D(3), 2F (2), 1 I , 2G(2), 3 H , 2 I , 4P, 4 D, 4F , 4G, 6S  6S

 Degeneración de un término. Los términos 1 D, 3P y 1S agrupan en su conjunto a los 15 microestados de

la configuración p2. El número de microestados que contiene cada término o degeneración del término se

calcula multiplicando las orientaciones posibles de su momentos orbital (multiplicidad orbital) por las de

su momento de espín (multiplicidad de espín).

Número de M  L

posibles: (2 L + 1) (multiplicidad orbital del término)

Número de M S posibles: (2S + 1) (multiplicidad de espín del término)

 Degeneración del término: (2S + 1) × (2 L + 1).

Cuando S = 0 (multiplicidad 2S + 1 = 1) el término se llama singlete, cuando S = 1/2 (multiplicidad = 2),

doblete, cuando S = 1 (multiplicidad = 3), triplete, etc.

Universidad de Alcalá Tema 1: La estructura electrónica de los átomos | 23

Page 22: Enlace Quimico Y Estructura de Materia

7/22/2019 Enlace Quimico Y Estructura de Materia

http://slidepdf.com/reader/full/enlace-quimico-y-estructura-de-materia 22/89

Así, los 15 microestados de una configuración p2 se distribuyen de la siguiente forma:

 Término  D1 ⇒

 L = 2

S = 0

 M  L = + 2, +1, 0, ±1, ±2

 M S  = 0(5×1 = 5 microestados)

 Término  P3 ⇒

 L = 1

S = 1

 M  L = +1, 0, ±1

 M S  = +1, 0, ±1(3×3 = 9 microestados)

 Término S1 ⇒  L = 0S = 0

 M  L = 0 M S  = 0

(1×1 = 1 microestado)

 Primera y segunda reglas de Hund. Hund desarrolló una serie de reglas empíricas que predicen el

término de menor energía, pero que no dicen nada en cuanto al orden de energía del resto de estados:

1 De los términos de Russell–Saunders que proceden de una configuración dada, el más estable es el de

mayor multiplicidad de espín (mayor valor de S ).

2 Para un grupo de términos con el mismo valor de S , el de mayor L es el de menor energía.

Por tanto, el término fundamental de una configuración p2 es el 3P. Obsérvese que la primera regla es

análoga al enunciado anterior de la regla de Hund: el término 3P corresponde al máximo desapareamiento

de espín. Tres de sus nueve microestados se representan a continuación:

ml +1 0 –1 +1 0 –1 +1 0 –1

La figura 1.20 esquematiza los desdoblamientos en términos de energía Russell–Saunders para una

configuración p2.

   E  n  e  r  g   í  a

Configuración Término

Interaccionesinterelectrónicas

 p2

1S 

1 D

3P

Figura 1.20. El desdoblamiento de laconfiguración p2 por interacción interelectrónica.

Términos de configuraciones con sólo capas o subcapas completas . Aunque la derivación completa de

los términos de una configuración puede ser laboriosa, analizaremos algunos casos sencillos. Unoespecialmente simple es el de las configuraciones con sólo capas o subcapas completas. En ellas, sólo hay

una disposición posible para los electrones en la cual los momentos orbitales y de espín de los distintos

electrones se anulan entre sí, es decir  L = 0 y S  = 0. Ello significa, en primer lugar, que tales

configuraciones poseen únicamente el término1S :

 ns2

ns2np6

(n±1)d 10

ns2

⇒L = 0S = 0

Término S1

(1 microestado)

Una segunda derivación lógica, e implícita en lo explicado hasta ahora, es que a la hora de determinar los

términos de una configuración dada, no es necesario considerar las capas y subcapas completas, pues esnula su aportación al momento total tanto orbital como de espín.

Términos de configuraciones con sólo un electrón. Lógicamente, si además de las capas o subcapas

llenas tenemos una subcapa con un único electrón, los momentos totales serán los aportados por ese único

24 | Enlace químico y estructura de la materia Licenciatura en Química

Page 23: Enlace Quimico Y Estructura de Materia

7/22/2019 Enlace Quimico Y Estructura de Materia

http://slidepdf.com/reader/full/enlace-quimico-y-estructura-de-materia 23/89

electrón. Tendremos un único término correspondiente a un espín total siempre de 1/2 y un momento

orbital igual al del orbital en el que esté situado dicho electrón:

 ns1 ⇒

 L = 0

S = 12

Término S2

(2 microestados)

 np1 ⇒  L = 1

S = 12

Término P2 (6 microestados)

 nd 1 ⇒

 L = 2

S = 12

Término D2

(10 microestados)

 Derivación del término fundamental de una configuración. Para otras configuraciones es bastante

sencillo determinar al menos su término fundamental. Como ejemplo, determinaremos el término

fundamental de la configuración de menor energía del átomo de titanio que es 1s2 2s2 2s6 3s2 3 p6 3d 2 4s2.

1 Escribe un diagrama de casillas de las subcapas incompletas, etiquetando los orbitales con su valor de

ml. Para el titanio, la única subcapa incompleta es la 3d .2 Coloca los electrones lo más desapareados posible, preferentemente en los orbitales de mayor ml.

ml +2 +1 0 –1 –2

 M S máximo = 1/2 + 1/2 = 1

 M  L máximo = 2 + 1 =3

3 Del diagrama anterior se deduce que el espín máximo para la configuración (1ª regla de Hund) es S = 1.

También se deduce un valor máximo de la proyección del momento angular  M L de 3. Eso significa que

el máximo L = 3 (2ª regla de Hund). El término fundamental del titanio es el 3F .

1.9 Periodicidad de algunas propiedades físicas

 El sistema periódico: bloques, períodos y grupos. En 1871, Mendeleev (1834–1907) propone su tabla enbase a las repeticiones periódicas que observó en las propiedades químicas de los elementos ordenados

por su masa atómica (ahora sabemos que es ordenados por su número atómico). En su forma actual (ver

tablas), la tabla periódica está compuesta por 7  periodos horizontales, 18 grupos verticales y otro grupo

constituido por 2 series de elementos (lantánidos y actínidos). Los grupos se numeran del 1 al 18. Los

grupos 1–2 forman el bloque s, los grupos 3–12, el bloque d , los grupos 13–17, el bloque p, y los

lantánidos y actínidos el bloque f .

Carga nuclear efectiva. Las propiedades periódicas están relacionadas con el electrón o electrones más

externos del átomo. Muchas de esas propiedades se pueden discutir en términos de la carga nuclear

efectiva que actúa sobre dichos electrones y que varía de la siguiente forma (figura 1.21):

• aumenta al avanzar en un período. Este aumento se debe a que la carga nuclear aumenta en una unidad

al pasar de un elemento al siguiente pero el apantallamiento lo hace mucho más débilmente ya que los

electrones de una misma capa se apantallan poco entre sí (0,35 según las reglas de Slater).

• varía muy poco al avanzar a lo largo de una serie de transición: el nuevo electrón se coloca en un orbital

(n –1)d , que al ser más interno apantalla bien al electrón más externo ns (0,85 según las reglas de Slater).

• desciende al completar un período e iniciar uno nuevo (p. ej. de Ne a Na). El nuevo electrón inaugura

una nueva capa y es muy bien apantallado por el resto de electrones situados en capas más internas.

• aumenta al descender en un grupo.

Universidad de Alcalá Tema 1: La estructura electrónica de los átomos | 25

Page 24: Enlace Quimico Y Estructura de Materia

7/22/2019 Enlace Quimico Y Estructura de Materia

http://slidepdf.com/reader/full/enlace-quimico-y-estructura-de-materia 24/89

Número atómico, Z 

5º Período

Y

Cd

bloque d 

10 20 30 40

   C  a  r  g  a  n  u  c   l  e  a

  r  e   f  e  c   t   i  v  a ,     Z

     *

2º Período

   1   º   P  e  r   í  o   d  o 3º Período 4º Período

NeAr

Kr

Sc

Li

Na

K

Zn

He

H

bloque d 

2

4

6

8

10

12

14

50 60

Xe

RbFigura 1.21. Gráfica dela carga nuclearefectiva sobre elelectrón más externopara los primeros 54elementos.

 Radio atómico. El radio atómico se puede relacionar con el tamaño de los orbitales externos, es decir con

el número cuántico principal n y la carga nuclear efectiva Z *. El radio atómico (figura 1.22):• disminuye a lo largo de un período. Los electrones se colocan en orbitales del mismo n pero que se

contraen progresivamente por el aumento de Z*.

• disminuye más suavemente en los metales de transición ya que  Z * aumenta poco. Incluso se produce

hacia el final de las series de transición externa un cierto aumento del radio debida a la fuerte repulsión

interelectrónica que generan tantos electrones d en la misma zona del espacio.

• se expande fuertemente al completar un período e iniciar uno nuevo. El nuevo electrón inaugura una

nueva capa, que además está atraída por una menor Z*.

• aumenta al descender por un grupo. Aunque Z * aumenta, el efecto del aumento de n es más importante.

• no aumenta casi al descender de la 2ª a la 3ª serie de los metales de transición externa. Este efecto sellama contracción de los lantánidos al ser ellos los responsables parciales de la contracción de la 3º

serie.

En la figura 1.23 se comparan los radios de algunos átomos con los de sus iones.

0 10 20 30 60 70 80 90Número atómico, Z 

   R  a   d   i  o  a   t   ó  m   i  c  o ,

    Å

1

2

lantánidosLi

Na

K

Rb

Cs

At

I

Br

Cl

FH

40 50

bloque d bloque d bloque d 

HgLa

2º Per.

   1   º   P  e  r .

3º Per. 4º Período 5º Período 6º Período

Figura 1.22.

Gráfica de radiosatómicos para losprimeros 86elementos.

26 | Enlace químico y estructura de la materia Licenciatura en Química

Page 25: Enlace Quimico Y Estructura de Materia

7/22/2019 Enlace Quimico Y Estructura de Materia

http://slidepdf.com/reader/full/enlace-quimico-y-estructura-de-materia 25/89

1

Li BeO F

Na Mg S Cl

Li+Be2+

Na+Mg2+

O2– F –

Cl –S2–

Figura 1.23. Los radios iónicos y atómicosde algunos elementos. Obsérvese que loscationes son menores que suscorrespondientes átomos, pero los anionesson más grandes.

 Energía de ionización. Es la energía mínima necesaria para sacar un electrón de un átomo gaseoso en su

configuración fundamental. La primera energía de ionización es la energía necesaria para sacar el primer

electrón (A(g) → A+(g) + e –), la segunda energía de ionización, el segundo (A+(g) → A2+(g) + e –), etc.

El comportamiento general de las energías de ionización es inverso al del radio atómico (figuras

1.24 y 1.25): disminuyen al descender en un grupo y aumentan al avanzar en un periodo. Excepciones

importantes a este comportamiento son:

• aumentos en la energía de ionización al descender en un grupo que se producen principalmente en elbloque d y sobre todo al pasar del 5º al 6º periodo y son efecto de la contracción de los Lantánidos.

• descensos en la energía de ionización al avanzar en un periodo. Muchas de estas excepciones son

periódicas y se pueden justificar por la estabilidad de las configuraciones esféricas (figura 1.26).

Las segundas energías de ionización son siempre mayores que las primeras (tabla 1.11). Arrancar

un electrón de un átomo o ion con configuración externa de gas noble (ns2np6) cuesta muchísima energía.

 Afinidad electrónica. La afinidad electrónica ( EA) de un átomo es la energía que se desprende en el pro-

ceso A(g) + e – → A –(g). Un átomo tiene una alta afinidad electrónica cuando el proceso anterior es muy

exotérmico. Con la excepción de los gases nobles, los elementos con altas energías de ionización tienen

altas afinidades electrónicas. Sin embargo, el comportamiento periódico de las afinidades electrónicas esmás complejo que el de las energías de ionización (figura 1.29). Las segundas afinidades son siempre

endotérmicas. Así para el oxígeno, EA1 = –∆ H EA1= +142 kJ mol –1, EA2 = –∆ H EA2

= –844 kJ mol –1.

La

Hg

bloque d 

Y

Cd

bloque d    P  r   i  m  e  r  a  e  n  e  r  g   í  a   d  e   i  o  n   i  z  a

  c   i   ó  n   (   k   J   /  m  o   l   )

Período   1   º   P  e  r .

Período4º Período

Ne

Ar

Kr

Sc

Li NaK

Zn

He

H

bloque d 500

1000

1500

2000

2500

10 20 30 40 50 60 70

Número atómico, Z 

80 90

5º Período 6º Período

3000

Xe

Rb

Rn

Cs

lantánidos

60

Figura 1.24.

Gráfica de lasprimerasenergías deionizaciónpara losprimeros 86elementos.

Universidad de Alcalá Tema 1: La estructura electrónica de los átomos | 27

Page 26: Enlace Quimico Y Estructura de Materia

7/22/2019 Enlace Quimico Y Estructura de Materia

http://slidepdf.com/reader/full/enlace-quimico-y-estructura-de-materia 26/89

Ac666

Ag731

Au890

Ni757

Pd804

Pt870

Ti658

Zr661

Y617

Ta761

C1090

Si786

Ge762

Pb716

Sn707

17

Br1140

I1010

At920

N1400

P1060

As966

Sb833

Bi703

1

O1310

H1310

V650

Hf681

Nb664

Cr652

Mo685

W770

Be900

Mg736

Ca590

Sr548

Ba502

Ra509

Mn717

Tc702

Re760

Li519

Na494

K418

Rb402

Cs376

Fr

B799

Al577

16

S1000

Se941

Po812

Te870

13 14

Hg1007

Cd868

Cu745

2

3 4 5 6 7 8 9 10 11

Fe759

Ru711

Os840

Co758

Rh720

Ir880

Zn906

He2370

12

15

Kr1350

Rn1040

Xe1170

Ga577

< 400

400 a 700

700 a 1000

1000 a 1500

1500 a 2000

>2000

La538

Sc631

In556

Tl590

Ne2080

F1680

Ar1520

Cl1260

Figura 1.25. Variación de la

primera energía de ionización (enkilojulios por mol) en los gruposprincipales y de transición externade la tabla periódica.

   P  r   i  m  e  r  a  e  n  e  r  g   í  a   d  e   i  o  n   i  z  a  c   i   ó  n   (   k   J   /  m  o   l   )

Ne

Ar

Kr

Be

Li

2º Período

3º Período

4º Período

B

N

O

Mg

Al

P S

As Se

400

800

1200

1600

2000

2400

1 2 13 14 15 16Grupo

18

s p s p s p s p s ps p

17

s p s p

KNa

Figura 1.26.

Gráfica de lasprimeras

energías deionización paralos elementos s

y p del 2º al 4ºperíodo, dondese observan lasirregularidadesperiódicas.

Tabla 1.11. Energías de ionización de los elementos del 2 al 5

 Elemento Primera Segunda Tercera Cuarta

He 2372 kJ mol –1

Li 519 kJ mol –1 7300 kJ mol –1

Be 900 kJ mol –1 1760 kJ mol –1 14800 kJ mol –1

B 799 kJ mol –1 2420 kJ mol –1 3660 kJ mol –1 25000 kJ mol –1

C

122Si

134Ge119

Pb35

Sn107

P72As78Sb103Bi91

N0

1 18

O141

Be–241Mg

–230Ca

–156Sr

–167Ba

–52Ra

H72Li60Na53K48Rb47Cs45Fr

Ga29

Tl19

B27Al42

In29

16

S200Se195

Po183

Te190

13 142 15 17

I295At

270

Ne–29Ar–34Kr–39

Rn–41

Xe–40

< 100

–100 a 0

0 a +100

+100 a +200

+200 a +300

> +300 F328Cl

349Br325

He–21

Figura 1.27. Variación de laprimera afinidad electrónica (enkilojulios por mol) en los gruposprincipales de la tabla periódica.

Bibliografía

1 Atkins, págs. 46–55 y 231–2753; Butler, págs. 15–67; Shriver, págs. 3–36; Sharpe, págs. 37–96.

2 D. Cruz, J. A. Chamizo, A. Garritz, “Estructura atómica. Un enfoque químico”, Addison-Wesley,

Wilmington, 1987, 820 páginas.

Bibliografía complementaria

1 S. Weinberg, “Partículas subatómicas”, Prensa Científica, Barcelona, 1985, 206 páginas.

2 S. W. Hawking, “Historia del tiempo, del big bang a los agujeros negros”, Crítica, Barcelona, 1988.

3 T. Hey, P. Walters, “El universo cuántico”, Alianza Editorial, Madrid, 1989, 211 págs.

28 | Enlace químico y estructura de la materia Licenciatura en Química

Page 27: Enlace Quimico Y Estructura de Materia

7/22/2019 Enlace Quimico Y Estructura de Materia

http://slidepdf.com/reader/full/enlace-quimico-y-estructura-de-materia 27/89

4 R. P. Feynman, “¿Está Vd. de broma, sr. Feynman?”, Alianza Editorial, Madrid, 1987, 403 págs.

5 R. P. Feynman, “¿Qué te importa lo que piensen los demás?”, Alianza Editorial, Madrid, 1990, 290 págs.

Seminarios

1.1 Di cuántos protones, neutrones y electrones hay en el:

a) nitrógeno–14; b) nitrógeno–15; c) tántalo–179; d) uranio–234; e) sodio–23 monopositivo; f)

oxígeno–16 dinegativo.1.2 ¿Cuáles de los siguientes pares son isótopos?:

a) 2H+ y 3H; b) 3He y 4He; c) 12C y 14N+; d) 3H y 4He –.

1.3 ¿Por qué el número másico A y la masa relativa de un átomo no son iguales?

1.4 Si se desplazan a la misma velocidad, ¿quien tiene mayor longitud de onda, un electrón o un protón?.

1.5 ¿Por qué aparece únicamente la serie de Lyman en un espectro atómico de absorción?

el átomo de hidrógeno

1.6 ¿Cómo puede un electrón pasar a un estado excitado?.

1.7 ¿Cuáles son los cuatro números cuánticos de un electrón en el átomo de hidrógeno? ¿Cuáles son losvalores permitidos para cada uno de ellos?

1.8 ¿Cuál es la principal diferencia entre una órbita de Bohr y un orbital en mecánica cuántica?.

1.9 Da el valor del momento cuántico azimutal y di cuántos orbitales tiene cada una de las siguientes

subcapas:

a) 3s, b) 4 p, c) 5 p, d) 3d , e) 2s, f) 5s, g) 4 f .

1.10 ¿Cuántos orbitales hay en cada una de las siguientes capas o subcapas?

a) capa n = 1, b) capa n = 2, c) capa n = 3, d) subcapa 3d , e) subcapa 4 p.

1.11 ¿Cuáles de los siguientes orbitales no pueden existir?

a) 2d , b) 5 p, c) 3 f , d) 10s, e) 1 p, f) 4 f , g) 4g, h) 4d .1.12 ¿Qué tipo de soluciones son aceptables en la ecuación de ondas para la función de ondas de un electrón

bajo la atracción del núcleo atómico?

1.13 Comenta el significado de las siguientes afirmaciones:

a) el número de nodos de las funciones radiales Rn,l(r ) es (n –l–1).

b) el radio más probable (valor de r para el que 4πr 2 R2(r ) es máximo) aumenta al aumentar n, siendo l

constante.

c) el primer máximo relativo de la función 4πr 2 R2(r ) se da a mayores radios al aumentar el valor de l,

siendo n constante.

1.14 Di si son ciertas o falsas las siguientes afirmaciones, razonando la respuesta:a) las funciones radiales para el átomo de hidrógeno constan del producto de una constante por un

polinomio y por una función que crece exponencialmente al aumentar el radio.

b) el orbital 1s no tiene dependencia angular. Sin embargo, el orbital 3s sí tiene dependencia angular.

c) el orbital pz es independiente de φ y, por tanto, presenta simetría a lo largo del eje z.

d) la representación gráfica de A2( px) da lóbulos alargados, de signo contrario, a lo largo del eje x .

1.15 Di cuántos nodos radiales (esféricos) y angulares tiene cada uno de los siguientes orbitales:

a) 4s, b) 3d , c) 2 p, d) 5 p.

1.16 Di qué es lo que entiendes por orbital.

1.17 Dadas las funciones angulares:

 A(θ ,φ ) = (31/2/2π1/2) senθ cosφ 

 A(θ ,φ ) = (31/2/2π1/2) senθ senφ 

a) ¿en qué direcciones presentarán el máximo valor para A2?

Universidad de Alcalá Tema 1: La estructura electrónica de los átomos | 29

Page 28: Enlace Quimico Y Estructura de Materia

7/22/2019 Enlace Quimico Y Estructura de Materia

http://slidepdf.com/reader/full/enlace-quimico-y-estructura-de-materia 28/89

b) ¿a qué orbitales pertenecen estas funciones?

1.18 ¿En qué dirección o direcciones es máxima la probabilidad de encontrar un electrón para un orbital:

a) s, b) px, c) d xy, d) d x2 –y2?

1.19 Para cada uno de los siguientes orbitales, di si la probabilidad de encontrar el electrón en la dirección del

eje z es máxima, mínima o nula.

a) px, b) py, c) pz.1.20 ¿Cómo es la densidad de probabilidad para un orbital  pz en un punto de coordenadas radiales r = 1 Å,

θ = 90°, φ = 45°?

1.21 Indica como variará en función del radio la probabilidad de encontrar el electrón en los orbitales 3s, 2 py y

3d z2 en los puntos de coordenadas (r , 0°, 0°).

1.22 ¿Cuál es la relación entre la energía del estado fundamental de los iones monoelectrónicos He+ y Be3+?

¿Cómo esperarías que fuesen las primeras energías de ionización del H, He+, Li2+ y Be3+ al compararlas?

1.23 Calcula la ecuación general de Rydberg para un átomo monoelectrónico de número atómico  Z .

1.24 ¿Cuánto vale el momento angular orbital para los orbitales s, p y d ?

1.25 Si un electrón tiene un momento angular orbital de (12)1/2(h/2π), ¿en qué tipo de orbital puede estarsituado?

1.26 ¿A qué niveles puede pertenecer un orbital cuyo vector de momento angular orbital tiene una proyección

sobre el eje z que vale 2(h/2π)?¿Y si vale (h/2π)?

átomos polielectrónicos

1.27 ¿Cuál es la razón por la que no pueden resolverse exactamente la función de onda para átomos

polielectrónicos? ¿Cómo se puede solucionar el problema?

1.28 Utilizando las reglas de Slater, calcula la carga nuclear efectiva para los siguientes electrones:

a) un electrón de valencia del potasio; b) un electrón 4s del manganeso; c) un electrón 3d del manganeso;

d) un electrón de valencia del bromo; e) un electrón de valencia del catión Fe2+.

1.29 Calcula los valores de la carga nuclear efectiva por el método de Slater para un electrón 4s del Ca, Mn,

Ga y Se. Compara los resultados obtenidos para cada átomo.

1.30 ¿Están siempre en el mismo orden los niveles de energía de los distintos orbitales, independientemente

del átomo que se considere?

1.31 Describe la variación de la energía de los orbitales 3s, 3 p y 3d  con el número atómico. Justifica esta

variación aplicando los conceptos de penetración y apantallamiento de orbitales.

1.32 ¿Se puede sugerir alguna sencilla explicación física para el hecho de que, según regla de Hund, los tres

electrones 2 p de un átomo de nitrógeno prefieran tener valores diferentes de sus números cuánticosmagnéticos?

1.33 Escribe el símbolo y el nombre del elemento de número atómico más bajo que tenga:

a) un electrón p; b) un subnivel p completo; c) un electrón f ; d) cinco electrones d .

1.34 Escribe la configuración electrónica en el estado fundamental de los siguientes átomos:

a) Li, b) C, c) Kr, d) Si, e) Co, f) Br, g) Sr, h) As, i) V, j) Cr, k) Nb, l) Fe, m) Au, n) Tl.

1.35 Escribe la configuración electrónica en el estado fundamental de los siguientes iones:

a) S2–, b) Rb+, c) N3–, d) Mg2+, e) Ti4+, f) Cl –, g) Ga3+, h) Fe2+, i) Fe3+, j) Tl+, k) Au3+.

1.36 Di cuál de los siguientes átomos o iones debe ser paramagnético en su estado fundamental:

a) Li, b) Mg, c) S, d) Zn, e) Ba, f) Re, g) Cu2+, h) Fe3+.1.37 Predice el número de electrones no apareados en las especies siguientes:

a) Al3+; b) Mn5+; c) Cu+; d) Zr3+; e) Na+.

1.38 Muestra que un átomo de configuración ns2np6 es esféricamente simétrico. ¿Es lo mismo cierto para un

30 | Enlace químico y estructura de la materia Licenciatura en Química

Page 29: Enlace Quimico Y Estructura de Materia

7/22/2019 Enlace Quimico Y Estructura de Materia

http://slidepdf.com/reader/full/enlace-quimico-y-estructura-de-materia 29/89

átomo de configuración ns2np3?

1.39 ¿Cuáles de los siguientes átomos no tienen distribuciones esféricas totales de la nube de carga electrónica

en su estado fundamental?:

a) Na, b) O, c) Ca, d) Xe, e) Cr, f) Mn.

1.40 Deduce sin recurrir a la tabla periódica, la configuración electrónica de los elementos Si, Z = 25, Rb y

Níquel. Identifica correctamente a cada uno de ellos y asigna el término Russell–Saunderscorrespondiente a su estado fundamental.

1.41 Escribe las configuraciones electrónicas y determina el símbolo correspondiente al término fundamental

de Russell–Saunders de cada uno de los iones siguientes:

a) Ti2+; b) Mn2+; c) Cu2+; d) Pt2+.

1.42 ¿A qué grupo pertenecen los elementos con los siguientes términos fundamentales de Russell–Saunders?

a) 2S; b) 3P; c) 2D; d) 2P; e) 4S.

1.43 Dado el término 3D, determina:

a) el número de microestados que contiene; b) su multiplicidad orbital; c) ¿podría surgir de una

configuración electrónica nd3?

1.44 Un ion de un metal de transición tiene los siguientes términos derivados de su configuración electrónica

fundamental: 1S, 3P, 1D, 1G, 3F. ¿Qué cationes di– o tripositivos serían posibles?

1.45 Considera el átomo de carbono excitado, según la configuración electrónica 1s22s22p13p1. ¿Cuál es el

término fundamental ?.

1.46 Deduce los términos fundamentales Russell–Saunders correspondientes a la configuración fundamental y

a la primera configuración excitada del átomo de titanio.

propiedades periódicas

1.47 Define los siguientes términos: período, grupo, elemento representativo, elemento de transición,lantánido, metal alcalino, energía de ionización, afinidad electrónica.

1.48 ¿Qué ion crees que tendrá menor radio, Fe2+ o Fe3+?

1.49 Ordena los siguientes iones en orden decreciente de radio: Se2–, S2–, Te2–, O2–.

1.50 Las siguientes partículas son isoelectrónicas, es decir, tienen la misma configuración electrónica.

Ordénalas según su radio decreciente: Ne, F –, Na+, O2–, Mg2+.

1.51 Ordena los siguientes iones isoelectrónicos por orden de radio iónico decreciente: Ti4+, P3–, Sc3+, S2–,

Mn7+.

1.52 Ordena los elementos siguientes en orden creciente de su primera energía de ionización: Na, F, I, Cs, Ne.

1.53 Justifica cuál de los átomos de los siguientes pares tienen mayor energía de ionización:a) S, P; b) K, Cu; c) Ca, Rb; d) Al, Mg; e) Cs, Al; f) Ar, K; g) Mo, W.

1.54 Explica el siguiente hecho: “la primera energía de ionización de Pd es 805 kJ/mol y el de Pt es 870

kJ/mol”.

1.55 Comenta la variación existente entre Y y La frente a Zr y Hf, en los valores de las primeras energías de

ionización.

1.56 Los elementos de número atómico Z = 7, 8 y 9 tienen valores de la primera energía de ionización de

1400, 1310 y 1680 kJ mol –1, respectivamente. Da una explicación razonable a este hecho.

1.57 La primera energía de ionización de un átomo es aquella que hay que suministrarle para arrancar el

primer electrón, la segunda es la que hay que suministrarle para arrancar el segundo electrón, etc. ¿Porqué es mayor la segunda energía de ionización que la primera?

1.58 Explica el hecho siguiente: “la segunda energía de ionización del magnesio es mayor que la primera, pero

no tan grande como la segunda energía de ionización del sodio”.

Universidad de Alcalá Tema 1: La estructura electrónica de los átomos | 31

Page 30: Enlace Quimico Y Estructura de Materia

7/22/2019 Enlace Quimico Y Estructura de Materia

http://slidepdf.com/reader/full/enlace-quimico-y-estructura-de-materia 30/89

1.59 De los siguientes pares de elementos, indica el que posee mayor afinidad electrónica:

a) F, Cl, b) Cl, Br; c) O, S, d) S, Se.

¿Por qué la segunda afinidad electrónica para O y S resulta ser un proceso endotérmico?.

1.60 Di cuáles de las siguientes afirmaciones son ciertas:

a) El ion Na+ es menor que el ion K+.

b) El ion Na+ es menor que el átomo de Na.c) El ion F – es menor que el átomo de F.

d) Los iones de los metales alcalinos son más pequeños que los iones de los metales alcalino-térreos del

mismo período.

e) La energía de ionización crece al bajar en el grupo.

f) Una razón por la que el helio no es reactivo es por su alta energía de ionización.

g) La primera energía de ionización del cesio es mayor que la del bario.

h) La primera energía de ionización del He+ es la misma que la segunda del átomo de Helio.

i) La afinidad electrónica de un catión es mayor que la del átomo correspondiente.

Problemas

isótopos, partículas elementales

1.1 El cobre contiene dos isótopos: 69,09% de cobre–63 ( M r = 62,9298) y 30,91% de cobre–65

( M r = 64,9278) ¿Cuál es la masa relativa ( M r) promedio del cobre?

1.2 El carbono contiene dos isótopos: 98,89% de carbono–12 (su  M r es exactamente 12) y 1,11% de

carbono–13 ( M r = 13,003) ¿Cuál es la masa relativa ( M r) promedio del carbono?

naturaleza ondulatoria de la luz

1.3 Si el dibujo representa dos ondas electromagnéticas A y B situadas en la región del infrarrojo, calcula:

a) Su longitud de onda y su frecuencia (c = 2,998 108 m s –1).b) El tiempo en que ambas recorrerán la distancia marcada.

Onda A Onda B

6,0 10 -3 cm

6,0 10 -3 cm

1.4 Calcula la frecuencia de cada una de las radiaciones siguientes (c = 2,9979 108 m s –1):

a) radiación infrarroja de λ = 10 –4 m; b) radiación ultravioleta de λ = 3 10 –8 m; c) radiación infrarroja de

λ = 5 10 –6 m.

longitud de onda y cuantos de energía

1.5 El efecto fotoeléctrico (figura 1.4) se utiliza en algunos dispositivos de alarma contra robo. El rayo de luz

incide sobre el cátodo (–) y arranca electrones de su superficie. Estos electrones son atraídos hacia el

ánodo (+), y el circuito eléctrico se cierra por medio de una batería. Si el rayo de luz se encuentra

bloqueado por el brazo de un ladrón, el circuito eléctrico se rompe y salta el sistema de alarma. ¿Cuál es

la máxima longitud de onda que se podría utilizar en un sistema de alarma si el cátodo de la célula es de

wolframio y los electrones son arrancados del mismo con una energía cinética de 8,0 10 –19 J cuando la

longitud de onda de la luz incidente es de exactamente 1,25 103 Å? (h = 6,626 10 –34 J s, 1 Å = 10 –10 m,

c = 2,998 108 m s –1).

1.6 La longitud de onda umbral o crítica del efecto fotoeléctrico para el litio es 5200 Å. Calcula la velocidadde los electrones emitidos como resultado de la absorción de luz de 3600 Å (h = 6,626 10 –34 J s, 1

Å = 10 –10 m, c = 2,998 108 m s –1, me = 9,11 10 –31 kg).

1.7 Calcula la longitud de onda, en nanometros, de la línea de la serie de Balmer que resulta de la transición

32 | Enlace químico y estructura de la materia Licenciatura en Química

Page 31: Enlace Quimico Y Estructura de Materia

7/22/2019 Enlace Quimico Y Estructura de Materia

http://slidepdf.com/reader/full/enlace-quimico-y-estructura-de-materia 31/89

de n = 3 a n = 2 [ℜ vale 3,29 1015 s –1]. ¿En qué región del espectro electromagnético (figura 1.3)

aparecerá?

1.8 La energía de ionización para el hidrógeno es la que debe absorber para separar el electrón, cuando éste

está en el estado fundamental, H(g) → H+(g) + e –. Calcula, a partir de la ecuación de Balmer, la energía

de ionización del hidrógeno atómico gaseoso (h = 6,626 10 –34 J s, c = 2,998 108 m s –1, ℜ = 3,29 1015 s –1,

 N A = 6,022 1023 mol –1).ecuación de de Broglie

1.9 Calcula la longitud de onda de

a) un electrón que se desplaza a 3,0 107 m s –1 [h = 6,626 10 –34 J s, me = 9,11 10 –31 kg].

b) una pelota de 140 g que se mueve a 160 km/h.

1.10 Calcula la longitud de onda de:

a) un electrón cuya energía es 10 keV [h = 6,626 10 –34 J s, me = 9,11 10 –31 kg, 1 eV = 1,602 10 –19 J].

b) una partícula de masa 125 g cuya velocidad es de 45 m/s.

1.11 ¿Cuál de las radiaciones siguientes sería la apropiada para producir fenómenos de difracción sobre un

metal? (1 eV = 1,602 10 –19 J, h = 6,626 10 –34 J s).

a) electrones de energía cinética igual a 1,00 eV (m e = 9,109 10 –31 kg);

b) neutrones de energía cinética igual a 0,0200 eV (m n = 1,674 10 –27 kg).

1.12 ¿Con qué velocidad se tendría que mover una pelota de tenis de 50 gramos para que su onda asociada

fuera la de un electrón de energía igual a 2,0 eV (1 eV = 1,602 10 –19 J, m e = 9,109 10 –31 kg, h = 6,626

10 –34 J s)?

  principio de incertidumbre

1.13 La velocidad medida para un electrón da un resultado de 90 m s –1 con una imprecisión del 10%. ¿Cuál es

la precisión máxima con la que se puede determinar su posición? (h = 6,626 10 –34

J s, m e = 9,109 10 –31

kg)

1.14 Un electrón se desplaza con una velocidad de 1,08 m s –1. Si se quiere determinar la posición con una

precisión de 0,010 Å (1 Å = 10 –10 m), calcula su momento y la indeterminación cometida. Compara los

resultados obtenidos. (m e = 9,109 10 –31 kg, h = 6,626 10 –34 J s)

 

1.15 Calcula la frecuencia de la luz necesaria para ionizar átomos de litio si la primera energía de ionización

del litio es 520 kJ mol –1 (h = 6,626 10 –34 J s, N A = 6,022 1023 mol –1).

1.16 Se puede usar luz de 590 nm (1 nm = 10 –9 m) de longitud de onda para excitar el electrón 3s del sodio a

un orbital 3 p. Dado que la energía de ionización del sodio en su estado fundamental es 494 kJ mol –1,¿cuál será la energía de ionización del átomo en su estado excitado [Ne]3 p1 (h = 6,626 10 –34 J s,

c = 2,998 108 m s –1, N A = 6,022 1023 mol –1).

1.17 La máxima longitud de onda de luz que puede expulsar un electrón de un ion Li – y conseguir la

formación de un átomo de litio neutro es 2000 nm. Calcula la de afinidad electrónica del litio

(h = 6,626 10 –34 J s, c = 2,998 108 m s –1, N A = 6,022 1023 mol –1).

Soluciones a los seminarios1.1 Protones, neutrones, electrones: a) 7, 7, 7; b) 7, 8, 7; c) 73, 106, 73; d) 92, 142, 92; e) 11, 12, 10; f) 8, 8, 10.1.2 a) y b).

1.3 Se trata de dos conceptos diferentes. El número másico ( Z ) es, por definición, un número entero igual al número total denucleones, mientras que la masa relativa es un número decimal igual a la relación entre la masa de la partícula considerada y ladoceava parte de la masa de un átomo de carbono–12. Sin embargo, existe una relación accidental, y es que, como la masarelativa del protón y del neutrón son cercanas a la unidad y la del electrón es muy pequeña, el valor de de la masa relativa deun átomo es cercano al valor de su número másico.

Universidad de Alcalá Tema 1: La estructura electrónica de los átomos | 33

Page 32: Enlace Quimico Y Estructura de Materia

7/22/2019 Enlace Quimico Y Estructura de Materia

http://slidepdf.com/reader/full/enlace-quimico-y-estructura-de-materia 32/89

1.4 Un electrón ya que su masa es menor.1.5 En un espectro de emisión, los átomos de hidrógeno son previamente excitados a estados superiores de energía, emitiendo luz

por caída desde este estado excitado a cualquier otro estado de n inferior y dando lugar a la serie de Lyman si n =1, de Balmer,si n =2, etc. En un espectro de absorción, la luz es absorbida por los átomos de hidrógeno que son excitados a estadossuperiores de energía. Como la gran mayoría de los átomos están en su estado fundamental (n =1), solo se observantransiciones desde este estado (serie de Lyman).

1.6 Con una radiación electromagnética de frecuencia adecuada ( E = hν ).1.7 Ver tabla 1.3.1.8 En el modelo de Bohr, una órbita tiene una trayectoria y una energía definidas. En el modelo cuántico, el orbital tiene una

energía, pero no una trayectoria, definidas.1.9 Número de orbitales de una subcapa = 2l + 1: a), e) y f)l = 0, 1 orbital; b) y c) l = 1, 3 orbitales; ; d) l = 2, 5 orbitales; e) l = 3,

7 orbitales.1.10 a) 1 orbital; b) 4 orbitales; c) 9 orbitales; d) 5 orbitales; e) 3 orbitales.1.11 No pueden existir a), c), e) y g).1.12 Las funciones tienen que ser continuas, finitas y con un sólo valor cada uno de sus puntos. Ello exige que en sus extremos se

anule (condiciones frontera).1.13 b) Esta afirmación está relacionada con la difusión de los orbitales, tal como se muestra en la figura 1.13a. Por ejemplo, el

máximo absoluto del orbital 2s (n =2) aparece a un radio mayor que el máximo absoluto y único del orbital 1s (n = 1). c) Esta

afirmación está relacionada con la penetración de los orbitales, tal como se muestra en la figura 1.13b. Por ejemplo, el primery único máximo del orbital 2 p (l = 1) aparece a un radio mayor que el primer máximo relativo del orbital 2s (l = 0).1.14 a) falsa, decrece al aumentar el radio; b) falsa, ningún orbital s tiene dependencia angular; c) cierta; d) falso, A2 solo puede ser

positiva, es A quien cambia de signo.1.15 Nodos radiales = n –l –1, nodos angulares = l, nodos totales = n –1. a) 3 y 0; b) 0 y 2; c) 0 y 1; d) 3 y 1.1.16 Un orbital, en sentido puro, es cada uno de los estados de energía electrónicos de un átomo o ion monoelectrónico, como el

átomo de hidrógeno. En un átomo polielectrónico, cada electrón no tiene en realidad un estado de energía independiente. Sinembargo, una aproximación que se realiza es suponer que sí son independientes (aproximación orbital), y, por extensión, acada estado de energía de un electrón en un átomo polielectrónico, se le llama orbital.

1.17 Primera función: a) (90°, 0°) y (270°, 0°); b) p x ; segunda función: a) (90°, 90°) y (270°, 90°); b) p y.1.18 a) En todas las direcciones la probabilidad es la misma; b) en el eje  x ; c) en las bisectrices de los ejes  x e y; d) en los ejes x e y.1.19 a) nula; b) nula; c) máxima.

1.20 Nula, ya que se trata de un punto del plano  xy.1.21 Se trata de los puntos del eje z. Para el orbital 2 p y; la probabilidad es nula en todos los puntos. Para los demás casos, ver la

variación de la función radial con el radio para orbitales 3s y 3d en las figuras 1.11 y 1.12, respectivamente.1.22 a) 1/4; b) 1:4:9:16.1.23 ν = Z 2ℜ[(1/n1

2)–(1/n22)]

1.24 s, 0; p, 1,41(h/2π); d , 2,45(h/2π).1.25 En un orbital f .1.26 a) nivel 3 o superior; b) nivel 2 o superior.1.27 Ver teoría.1.28 a) Z* = 2,2; b) Z* = 3,6; c) Z* = 5,6; d) Z* = 7,6; e) Z* = 6,25.1.29  Z* = 2,85; 3,6; 5,0; 6,95. La carga nuclear efectiva aumenta a lo largo del periodo.1.30 No. Ver figura 1.18.1.31 Ver teoría.1.32 Al colocar los tres electrones en tres orbitales distintos cuya mayor densidad se sitúa en distintas regiones del espacio, se

minimizan las repulsiones interelectrónicas.1.33 a) B; b) Ne; c) Ce (no el La, ya que su configuración electrónica es anómala, ver tabla 1.11); d) Cr (no el Mn, ya que la

configuración del Cr es anómala, ver tabla 1.11).1.34 a) [He]2s1; b) [He]2s22 p2; c) [Ar]3d 104s24 p6; d) [Ne]3s23 p2; e) [Ar]3d 74s2; f) [Ar]3d 104s24 p5; g) [Kr]5s2; h) [Ar]3d 104s24 p3; i)

[Ar]3d 34s2; j) [Ar]3d 54s1; k) [Kr]4d 45s1; l) [Ar]3d 64s2; m) [Xe]4 f 145d 106s1; n) [Xe]4 f 145d 106s26 p1.1.35 a) [Ne]3s23 p6 = [Ar]; b) [Kr]; c) [He]2s22 p6 = [Ne]; d) [Ne]; e) [Ar]; f) [Ne]3s23 p6 = [Ar]; g) [Ar]3d 10; h) [Ar]3d 6; i) [Ar]3d 5;

 j) [Xe]4 f 145d 106s2; k) [Xe]4 f 145d 8.1.36 En negrita, los orbitales en los que hay electrones desapareados: a) [He]2s1, paramagnético (1 electrón desapareado); b)

[Ne]3s2, diamagnético; c) [Ne]3s23 p4, paramagnético (2 electrones desapareados); d) [Ar]3d 104s2, diamagnético; e) [Xe]6s2,

diamagnético; f) [Xe]4 f 145d 56s2, paramagnético (5 electrones desapareados); g) [Ar] 3d 9, paramagnético (1 electróndesapareado); h) [Ar]3d 5, paramagnético (5 electrones desapareados).1.37 a) ninguno; b) dos; c) ninguno; d) uno; e) ninguno.

34 | Enlace químico y estructura de la materia Licenciatura en Química

Page 33: Enlace Quimico Y Estructura de Materia

7/22/2019 Enlace Quimico Y Estructura de Materia

http://slidepdf.com/reader/full/enlace-quimico-y-estructura-de-materia 33/89

1.38 Orbital Función Angular (A) A 2

 p x  ( √ 3/2 √π) senθ cosφ  (3/4π) sen2θ cos2φ 

 p y ( √ 3/2 √π) senθ senφ  (3/4π) sen2θ sen2φ 

 p z

( √ 3/2 √π) cosφ  (3/4π) cos2φ 

Suma de A 2 3/4π =constante

Siempre que los tres orbitales  p estén llenos por igual, tendremos una distribución de carga esférica. Por tanto, unaconfiguración ns2np3 será esférica en su estado fundamental, pues posee un electrón en cada orbital p.

1.39 Sí tienen distribución esférica: a) [Ne]3s1; c) [Ar]4s2; d) [Kr]4d 105s25 p6; e) [Ar]3d 54s1; f) [Ar]3d 54s2.No tienen distribución esférica: b) [He]2s22 p4.

1.40 Si, 3P; Mn, 6S ; Rb, 2S ; Ni, 3F .1.41 a) [Ar]3d 2, 3F ; b) [Ar]3d 5, 6S ; c) [Ar]3d 9, 2 D; d) [Ar]3d 8, 3F .1.42 a) Grupo 1; b) grupos 14 y 16; c) grupos 3 y 11; d) grupos 13 y 17; e) grupo 15.1.43 a) 15; b) 5; c) no.1.44 Los iones de configuración d 2 y d 8. M2+: grupos 4 y 10. M3+: grupos 5 y 11.1.45 3 D.1.46 Estado fundamental: [Ar]4s2 3d 2: 3F ; estado excitado: [Ar]4s1 3d 3: 5F .

1.47 Ver teoría.1.48 Fe3+, porque tiene menos electrones y además los electrones que quedan son más atraídos por la carga nuclear al haber menos

repulsiones (apantallamiento) entre ellos.1.49 r (Te2–) > r (Se2–) > r (S2–) > r (O2–).1.50 Al tener los mismos electrones, el tamaño dependerá de lo fuertemente que sean atraídos por el núcleo, es decir, de la carga

nuclear: r (O2–) > r (F –) > r (Ne) > r (Na+) > r (Mg2+).1.51 P3– > S2– > Sc3+ > Ti4+ > Mn7+.1.52 Cs < Na < I < F < Ne.1.53 a) P (y no S, ver teoría); b) Cu; c) Ca; d) Mg (y no Al, ver teoría); e) Al; f) Ar; g) W (y no Mo, ver seminarios 1.54 y 1.55).1.54 La contracción de los lantánidos explica que los potenciales de ionización de los metales de la 2ª serie de transición sean

menores que los de la 3ª serie. Al bajar de la 2ª a la 3ª serie, el radio permanece aproximadamente constante pero la carganuclear efectiva crece.

1.55 Ver seminario anterior.1.56 Ver teoría.1.57 Porque al arrancar un electrón, los electrones que quedan son más atraídos por la carga nuclear al haber menos repulsiones

(apantallamiento) entre ellos.1.58 La segunda energía de ionización del sodio corresponde a un proceso en el que se arranca un electrón de una configuración de

gas noble.1.59 Ver tablas: a) Cl (¡atención!); b) Cl; c) S (¡atención!); d) S.1.60 Ciertas: a), b), f), h), i). Falsas: c), d), e), g).

Soluciones a los problemas1.1  M r = 63,547. 1.9 a) λ = 2,4 10 –11 m; b) λ = 1,06 10 –34 m.1.2  M r = 12,01. 1.10 a) λ =1,23 10 –11 m; b) λ = 1,18 10 –34 m.1.3 a) Onda A: λ = 1,0 10 –3 cm, ν = 3,0 1013 s –1, 1.11 a) λ = 1.23 10 –9 m; b) λ = 2,03 10 –10 m. El neutrón (las

onda B: λ = 2,0 10 –3 cm, ν = 1,5 1013 s –1; distancias interatómicas son aprox. 2–4 Å =2–4 10 –10 m)b) t = 2,0 10 –13 s. 1.12 v = 1,53 10 –23 m s –1

1.4 a) ν = 3 1012 s –1; b) ν = 10 1015 s –1; c) ν = 6 1013 s –1. 1.13 6,43 10 –6 m = 6,43 104 Å.1.5 λ = 2,51 103 Å. 1.14 mv = 9,838 10 –31 kg m s; ∆mv = 5,27 10 –23 kg m s;.1.6 v = 6,09 105 m s –1. 1.15 ν = 1,30 1015 s –1.1.7 a) λ = 656,3 nm. Aparecerá en el visible. 1.16 4,84 10 –19 J por átomo, 291 kJ por mol.1.8  E = 1320 kJ para 1 mol de hidrógeno. 1.17 9,94 10 –20 J por átomo, 59,9 kJ por mol.

Universidad de Alcalá Tema 1: La estructura electrónica de los átomos | 35

Page 34: Enlace Quimico Y Estructura de Materia

7/22/2019 Enlace Quimico Y Estructura de Materia

http://slidepdf.com/reader/full/enlace-quimico-y-estructura-de-materia 34/89

_________________________________________________________________________________________

2 El enlace en las moléculas2.1 La formación de pares iónicos

2.2 El enlace covalente: el enlace del par de electrones

2.3 Parámetros de los enlaces covalentes

2.4 La teoría de la repulsión de los pares electrónicos de la capa de valencia (VSEPR)

2.5 El carácter iónico de los enlaces covalentes y el carácter covalente de los enlaces iónicos

2.6 Las moléculas polares_________________________________________________________________________________________

Las fuerzas de atracción que unen a los átomos en los compuestos, se llaman enlaces químicos. En los

temas 2 y 3 estudiaremos los enlaces que se forman entre un limitado número de átomos en una

molécula. Al acercarse dos átomos, sus electrones se redistribuyen minimizando el potencial en la nueva

situación: los átomos se enlazan si la energía alcanzada es inferior a la que tenían por separado. Los dos

modelos extremos de redistribución de electrones son el iónico y el covalente, pero la mayoría de enlaces

se describen mejor considerando una mezcla de ambos.

2.1 La formación de pares iónicosEl enlace iónico se estabiliza por la atracción entre las cargas opuestas de cationes y aniones. Ningún

enlace es puramente iónico. En los compuestos iónicos (aquellos formados por enlaces principalmente

iónicos) cada ion tiende a rodearse del mayor número posible de iones de carga contraria, resultando que,

en condiciones normales, son sólidos con cationes y aniones alternándose en una red tridimensional.

Estos sólidos serán tratados en el tema 4. Aquí estudiaremos el enlace iónico mediante pares iónicos, un

modelo simple, aunque ficticio, formado por atracción entre un catión y un anión.

La formación de un enlace iónico es favorable si la energía necesaria para ionizar los átomos es

compensada por la energía liberada por las atracciones electrostáticas entre catión y anión (figura 2.1), lo

que es más fácil cuando reaccionan átomos de bajas energías de ionización con átomos de altasafinidades electrónicas.

–374 kJ/mol

① ∆ H I(K)+ ∆ H EA(Cl) = 418 + (–349) = 69 kJ/mol

–443 kJ/mol

K+Cl–(g)

K+(g) + Cl –(g)

K(g) + Cl( g)

69 kJ/mol

Figura 2.1. Balance energético para la formación de un mol depares iónicos de K+Cl–(g). La entalpía del proceso directo (flecharellena) es igual a la suma de las entalpías de los procesos ① y ➁(flechas huecas):① Formación de iones gaseosos a partir de los átomos gaseosos➁ Formación de los pares iónicos a partir de los iones

La entalpía① se calcula a partir de las entalpías de ionización (∆ H I)y de afinidad electrónica (∆ H EA). La energía➁ ha sido estimadamediante la ecuación derivada de la ley de Coulomb:

  E = 1

4πε 0

 N A z+ez–e

 z+,z

–= Cargas iónicas (catión, anión)e = unidad elementalde carga= 1,602 10–19 C

d = distancia entre catión y anión

ε 0 = 8,854 10–12 C2 m–1 J–1

tomando una distancia entre catión y anión de 3,14 Å, igual a lasuma de los radios iónicos de K+ y Cl–.

Carga iónica. ¿Por qué el potasio forma cationes monopositivos K+ mientras que el magnesio los forma

dipositivos Mg2+? La formación de un dicatión M2+ frente a un monocatión M+ supone invertir energía

en arrancar un segundo electrón, pero también una mayor atracción electrostática con el anión. Formar

K2+ supone arrancar un electrón de una configuración de gas noble y es demasiado desfavorable (figura

2.2). En el caso del magnesio, la formación del dicatión es favorable ya que la mayor atracciónelectrostática compensa la energía invertida en arrancar el segundo electrón(figura 2.3).

Habitualmente, los elementos de los grupos principales adquieren una carga iónica correspondiente

a una configuración de gas noble (o de pseudogas noble). Esta regla no es de aplicación general, sobre

Page 35: Enlace Quimico Y Estructura de Materia

7/22/2019 Enlace Quimico Y Estructura de Materia

http://slidepdf.com/reader/full/enlace-quimico-y-estructura-de-materia 35/89

todo en el caso de los metales de transición que pueden perder un número variable de electrones d .

+1018 kJ/mol

① ∆ H I1(K)+  ∆ H I2(K)+ 2 × ∆ H EA(Cl) =418 + 3070 + 2 × (–349) = 3488 kJ/mol

–4 × 443 =–1772 kJ/mol

K2+(Cl –)2(g)

K2+(g) + 2Cl –(g)

K(g) + 2Cl( g)

2790 kJ/mol

Figura 2.2. Balance energético para la formación de un mol depares iónicos de K2+(Cl–)2(g).

① La ionización del potasio requiere ahora 3070 kJ/molsuplementarios (segunda energía de ionización del potasio). Estaenergía es muy elevada porque supone arrancar un electrón deuna configuración de gas noble.

➁ La energía de atracción electrostática para K+(Cl–)2 es cuatroveces superior que para K+Cl–, pues el catión tiene doble carga yatrae a dos aniones (para simplificar se ha supuesto que ladistancia catión–anión es igual en ambos casos).

Como resultado, la formación de K+(Cl–)2(g) es desfavorable.

➁①

–178 kJ/mol

① ∆ H I1(Mg)+ ∆ H EA(Cl) =736 +  (–349) = 387 kJ/mol

–565 kJ/mol

Mg+Cl–(g)

Mg+(g) + Cl –(g)

Mg(g) + Cl( g)387 kJ/mol

–772 kJ/mol

① ∆ H I1(Mg)+  ∆ H I2(Mg)+ 2 × ∆ H EA(Cl) =

736 + 1450 + 2 × (–349) = 1488 kJ/mol

–4 × 565 =–2260 kJ/mol

Mg2+(Cl –)2(g)

Mg2+(g) + 2Cl –(g)

Mg(g) + 2Cl( g)

1488 kJ/mol

Figura 2.3. Balance energético para la formación de un mol de pares iónicos de (a) Mg+Cl–(g) y (b) Mg2+(Cl–)2(g). Laformación de esta segunda especie es más favorable ya que la segunda entalpía de ionización del magnesio (1450 kJ/mol) es

compensada por el aumento en la energía de atracción electrostática➁.

2.2 El enlace covalente: el enlace del par de electrones

Compartimiento de electrones. El modelo iónico no puede explicar la formación de muchos enlaces. Por

ejemplo, no justifica la estabilidad de los enlaces de elementos centrados en la tabla periódica (figura

2.4), ni la existencia de compuestos formados no por partículas iónicas sino por partículas neutras

llamadas moléculas. El compartimiento de electrones es un modelo alternativo a la formación de iones.

➁①

+291 kJ/mol

711 kJ/mol –420 kJ/mol

P+Cl–(g)

P+(g) + Cl –(g)

P(g) + Cl( g)

① ∆ H I(P)+ ∆ H EA(Cl) = 1060 + (–349) = 711 kJ/mol

Figura 2.4. La estabilidad de un enlace P–Cl no puede ser justificada mediante un modelo iónico. El alto potencial deionización del fósforo (1060 kJ/mol) no es compensado por laatracción entre iones, resultando en un proceso de formaciónendotérmico (+291 kJ/mol).

Teoría de Lewis. En 1916, Lewis propuso que la formación de un enlace covalente se produce por

compartimiento de dos electrones. El descubrimiento posterior del principio de Pauli justifica que los

electrones se compartan por pares. Dos átomos pueden formar un enlace simple, compartiendo un sólo

par de electrones, o enlaces múltiples (doble, triple, etc.), compartiendo dos o más pares. El número de

pares electrónicos compartidos se llama orden o multiplicidad de enlace. Las estructuras de Lewis son

representaciones en las que se indica, mediante puntos y guiones, la distribución de los electrones de

Universidad de Alcalá Tema 2: El enlace en las moléculas | 37

Page 36: Enlace Quimico Y Estructura de Materia

7/22/2019 Enlace Quimico Y Estructura de Materia

http://slidepdf.com/reader/full/enlace-quimico-y-estructura-de-materia 36/89

valencia en pares solitarios y pares de enlace entre los átomos que constituyen un compuesto.

Más información en la página Web http://www2.uah.es/edejesus/resumenes/EQEM.

 Regla del octeto. “Cuando los átomos reaccionan, tienden a conseguir una esfera externa de ocho

electrones.” Esta es una regla empírica que encuentra su justificación en la tendencia a la ocupación que

tienen los orbitales ns y np (4 orbitales, 8 electrones). Sin embargo, no es una regla universal, existiendovarios tipos de excepciones a la regla del octeto:

• Compuestos de octeto expandido, como el PCl5, donde el fósforo se supone rodeado de 10 electrones.

Se observan octetos expandidos para elementos del 3º período y siguientes, lo que se suele justificar

por la existencia en la capa de valencia de también orbitales nd , aunque este argumento es dudoso.

• Compuestos con octeto incompleto, como el BF3 donde el boro se supone rodeado de 6 electrones.

Aún así, las propiedades del BF3 vienen influenciadas por su tendencia a completar el octeto.

•  Moléculas con número impar de electrones (radicales), como el NO. Los radicales de los grupos prin-

cipales son raramente estables debido a la tendencia a aparear los electrones y obtener el octeto, por

ejemplo, enlazándose entre sí. Sin embargo, existen algunas excepciones como el NO.•  Metales de transición, para los que tenemos disponibles 9 orbitales, uno ns, tres np y cinco (n–1)d .

Podría pensarse en una elevada tendencia de los metales de transición a completar 18 electrones, pero

los orbitales d , que pertenecen a una capa anterior, son menos reactivos que los s y p y tienen menor

tendencia a ser ocupados. Aunque para estos elementos existe la regla de los 18 electrones, su cumpli-

miento es muy limitado y el número de electrones es muy variable entre 8 y 18, incluyendo números

impares ya que, por la misma razón, los radicales estables en compuestos de transición no son nada

raros. Dadas las diferencias existentes entre grupos principales y de transición, en el estudio del enlace

químico, dejaremos un tanto de lado los compuestos de estos últimos.

Carga formal. La carga formal de un átomo en una molécula se asigna asumiendo que cada átomo vecompensada su carga nuclear por uno de los electrones de cada uno de sus pares de enlace (de acuerdo

con un igual compartimiento) y por los dos electrones de cada uno de sus  pares solitarios (que sólo le

pertenecen a él). La experiencia indica que las estructuras de Lewis más estables son normalmente:

• las que poseen cargas formales más pequeñas, y

• colocan las cargas negativas preferentemente sobre los átomos más electronegativos.

De acuerdo a este criterio, la estructura de Lewis más adecuada para el trifluoruro de boro es la A:

B F

F

F

B F

F

F

(A) (B)

Cargas formalesulas para todos

los átomos

El criterio de las cargas formales, junto a la regla del octeto, permite estimar cuál o cuáles son las

estructuras de Lewis más razonables para una molécula determinada. En ocasiones, como es el caso del

BF3, cada criterio predice, para una misma molécula, una estructura diferente. No siempre es fácil señalar

cuál es la más adecuada, aunque algunas ideas se dan a continuación.

Cómo escribir una estructura de Lewis. Se pueden seguir los siguientes pasos:

1 Cuenta el número total de electrones de valencia, considerando la carga iónica.

2 Escribe el esqueleto de la estructura, uniendo los elementos por enlaces simples. Por supuesto, es

necesario conocer con anterioridad la forma en la que están unidos los átomos.Opción A (regla del octeto).

3 Añade a los enlaces simples el número de enlaces múltiples necesario para poder cumplir la regla del

octeto. El número total de enlaces necesarios es igual a (número de átomos × 8 (ó ×2) – número de

38 | Enlace químico y estructura de la materia Licenciatura en Química

Page 37: Enlace Quimico Y Estructura de Materia

7/22/2019 Enlace Quimico Y Estructura de Materia

http://slidepdf.com/reader/full/enlace-quimico-y-estructura-de-materia 37/89

electrones)/2. Sólo suelen darse enlaces múltiples cuando al menos uno de los dos átomos es cabecera

de grupo (fundamentalmente C, N y O), o es azufre.

4 Completa el octeto alrededor de cada átomo colocando el número de pares libres necesario.

Finalmente, comprueba que el número de electrones colocados es el calculado en el punto 1.

Opción B (criterio de las cargas formales).

3 Une cada átomo terminal con el central con el número de enlaces adecuado para que su carga formalsea la más adecuada. Ten en cuenta que los átomos terminales siempre cumplen la regla del octeto.

4 Completa el átomo central con el número de electrones adecuado.

Algunas ideas para elegir en cada caso el criterio más adecuado:

• Los elementos del 2º periodo no sobrepasan el octeto.

• Para C,N y O como átomos centrales, intentar hacer cumplir siempre la regla del octeto.

• Para los elementos del 3º periodo y siguientes como átomos centrales, aplicar preferentemente la regla

de las cargas formales.

 Resonancia. A veces las propiedades de una molécula se interpretan mejor suponiendo que su

distribución electrónica es intermedia entre varias estructuras de Lewis. El caso más evidente es cuandotenemos dos estructuras idénticas en energía:

N OO N OO– –

Los experimentos muestran que la estructura real es una media de las dos estructuras de Lewis (así, por

ejemplo, los dos enlaces N–O son idénticos). A esta media se le llama resonancia y a la estructura

resultante de la media se le llama híbrido de resonancia. Las estructuras de resonancia sólo se diferen-

cian en la asignación de la posición de los pares electrónicos, nunca en las posiciones de los átomos.

El híbrido de resonancia tiene una energía menor que cada una de las formas resonantes. Esta

diferencia de energía recibe el nombre de energía de resonancia. Resonancia entre formas de diferente energía. En el caso de resonancia descrito anteriormente, las dos

estructuras propuestas son de idéntica energía y ambas participan por igual en la descripción del híbrido.

A veces, interesa describir una molécula como un híbrido de resonancia de estructuras diferentes:

S OO S OOS OO

La primera estructura cumple con el criterio de las cargas formales, mientras que la segunda y tercera

cumplen la regla del octeto. La estructura real será una mezcla de las señaladas y se parecerá más a la

estructura de menor energía (que a priori no sabemos cuál es). Eso se expresa diciendo que la estructura

de más baja energía tiene más peso en la descripción del híbrido de resonancia. El enlace covalente coordinado. A veces, en la formación (real o conceptual) de un enlace, el par

electrónico es aportado enteramente por uno de los átomos. Tal enlace se llama coordinado . Hay que

señalar que el origen de los electrones de enlace no implica por sí mismo nada sobre el carácter del

enlace formado.

2.3 Parámetros de los enlaces covalentes

Se llaman parámetros de enlace a aquellas propiedades características de los enlaces que dependen de los

átomos específicos que se enlazan pero que varían poco de compuesto a compuesto.

 Longitud de enlace. La longitud de un enlace covalente puede ser estimada aproximadamente mediantela suma de los radios covalentes: r c(C) = 0,77 Å, r c(H) = 0,37 Å ⇒ d (C-H) ≈ 1,14 Å. Este valor de dis-

tancia C–H es prácticamente independiente del compuesto considerado. Sin embargo, la longitud de un

enlace disminuye al aumentar el orden de enlace, lo que se utiliza como criterio para estimar la

Universidad de Alcalá Tema 2: El enlace en las moléculas | 39

Page 38: Enlace Quimico Y Estructura de Materia

7/22/2019 Enlace Quimico Y Estructura de Materia

http://slidepdf.com/reader/full/enlace-quimico-y-estructura-de-materia 38/89

multiplicidad del enlace (tabla 2.1).

Tabla 2.1. Longitudes medias de enlaces simples y múltiples en ångströms (1 Å = 10 –10 m)H–H 0,74 F–F 1,28 Cl–Cl 2,00 Br–Br 2,28 I–I 2,66

H–F 0,92 H–Cl 1,27 H–Br 1,42 H–I 1,61 O–H 0,94 N–H 0,98 C–H 1,10O–O 1,32 N–N 1,40 C–C 1,54 N–O 1,36 C–O 1,43 C–N 1,47

C=C 1,34 N=O 1,15 C=O 1,22 C=N 1,27C≡≡≡≡C 1,21 N≡≡≡≡O 1,08 C≡≡≡≡O 1,13 C≡≡≡≡N 1,15

 Entalpía de enlace. La entalpía de enlace (∆ H E) es la puesta en juego en el proceso de ruptura de enlaces

en el estado gaseoso. Así, la entalpía de enlace para el hidrógeno es la correspondiente al proceso

H2(g) → 2H(g). La entalpía de un enlace aumenta al aumentar la multiplicidad del enlace (tabla 2.2).

Tabla 2.2. Entalpías medias de enlace en kilojulios por mol H–H 436 F–F 158 Cl–Cl 242 Br–Br 193 I–I 151

H–F 565 H–Cl 431 H–Br 366 H–I 299 O–H 463 N–H 388 C–H 412O–O 157 N–N 163 C–C 348 N–O 200 C–O 360 C–N 305

O=O 496 N=N 409 C=C 612 N=O 600 C=O 743 C=N 615N≡≡≡≡N 944 C≡≡≡≡C 837 C≡≡≡≡O 1070 C≡≡≡≡N 891

2.4 La teoría de la repulsión de los pares electrónicos de la capa de valencia (VSEPR)

A diferencia del enlace iónico, el enlace covalente es direccional. La forma de una molécula explica una

parte esencial de sus propiedades. La teoría de la repulsión de los pares electrónicos de la capa de

valencia (VSEPR) hace una simple adición a las ideas de Lewis que permite prever la forma de una

molécula dada: “Los pares electrónicos se repelen unos a otros, por lo que se sitúan lo más alejados

posible entre sí”.

 Moléculas sin pares solitarios. La VSEPR predice las siguientes formas de moléculas:

Cl Be ClCl Be Cl

F B F

F F

BF F

H

C

HH

HH C H

H

H

Cl P

Cl

Cl Cl

ClCl

P

Cl

Cl

Cl

Cl

F S F

F F

F F

F

S

FF

F

FF

90°

120°

90°

109,5°

120°

BeCl2

BF3

CH4

PCl5

SF6

2

3

4

5

6

lineal

triangular plana

tetraédrica

bipirámide trigonal

octaédrica

 Molécula Estructura de Lewis Pares electrónicos Geometría molecular

 Moléculas con pares solitarios. Para los pares solitarios se sigue el mismo esquema que para los pares de

enlace. Ahora bien, la forma de una molécula viene dada por la disposición de los átomos alrededor del

40 | Enlace químico y estructura de la materia Licenciatura en Química

Page 39: Enlace Quimico Y Estructura de Materia

7/22/2019 Enlace Quimico Y Estructura de Materia

http://slidepdf.com/reader/full/enlace-quimico-y-estructura-de-materia 39/89

átomo central, por lo que los pares solitarios son, desde ese punto de vista, transparentes. Por otra parte,

los pares solitarios ocupan un espacio mayor y más cercano al átomo que los pares de enlace, pues

mientras aquellos son son atraídos por un único núcleo éstos lo son por dos. Para disminuir las

repulsiones, los ángulos tienen que ser mayores alrededor de los pares solitarios y decrecer en el orden

(PS,PS) > (PS,PE) > (PE,PE) [PE = par de enlace, PS = par solitario]. En una molécula con pares

solitarios de geometría electrónica tetraédrica, los ángulos no son iguales a 109,5°, sino que son mayoresen torno a los pares solitarios y, por tanto, menores entre pares de enlace. Así, el ángulo H–N–H es de

107° en NH3 y el H–O–H es de 105° en H2O.

<109,5°

<109,5°

<120°

SnCl2

NH3

H2O

SF4

ClF3

I3

BrF5

XeF4

2

3

2

triangular plana

tetraédrica

tetraédrica

bipirámide

trigonal

bipirámidetrigonal

bipirámidetrigonal

octaédrica

octaédrica

angular

pirámide trigonal

angular

forma de T

tetraedro

deformadoo balancín

lineal

pirámidecuadrada

plano-cuadrada

1 3

1 4

2 4

4 1 5

3 2 5

2 3 5

5 1 6

4 2 6

 Mol. Est. de Lewis PE PS P Geometría electrónica Geometría Molecular

Cl Sn Cl

H N

SnCl Cl

HH

H O

NH

HH

H

F S

O

HH

F

F

S

F

F F

F

F

F Cl F

F

F

Cl

F

F

I I I

I

I

I

F Br F

FF

BrF F

F Xe

FF

FF

F

F F Xe FF

FF

En una bipirámide trigonal, los tres vértices situados en el plano ecuatorial no son equivalentes con los

Universidad de Alcalá Tema 2: El enlace en las moléculas | 41

Page 40: Enlace Quimico Y Estructura de Materia

7/22/2019 Enlace Quimico Y Estructura de Materia

http://slidepdf.com/reader/full/enlace-quimico-y-estructura-de-materia 40/89

dos situados en los ápices. Un vértice ecuatorial tiene como vecinos más próximos dos ecuatoriales a

120° y dos axiales a 90°, mientras que uno apical tiene tres vecinos próximos a 90°. Al haber más espacio

en las posiciones ecuatoriales, los pares solitarios ocupan preferentemente estas posiciones. Por la misma

razón, en un octaedro dos pares solitarios se colocan en posiciones opuestas.

 Moléculas con sustituyentes de distinta electronegatividad Un sustituyente electronegativo retira

densidad electrónica del átomo central de forma que el par de enlace ocupa más espacio en torno alátomo sustituyente que en torno al central. Cuanto más electronegativo sea el sustituyente, menor es el

espacio que ocupa el par electrónico en el átomo central y, en consecuencia, los ángulos entre enlaces a

átomos muy electronegativos son menores que a átomos menos electronegativos. Por ejemplo, el ángulo

X–P–X es de 97,8˚ en PF3, 100,3˚ en PCl3 y 101,0˚ en PBr3.

 Moléculas con enlaces múltiples. En un enlace doble o triple, los dos o tres pares de enlace se encuen-

tran en el espacio situado entre los dos átomos que enlazan. Una forma conveniente de abordar un enlace

múltiple es considerar que ocupa una única zona del espacio y tratarlo como uno sencillo. Ahora bien, un

enlace múltiple ocupa más espacio y, tal como pasa con los pares solitarios, los ángulos de enlace en sutorno son mayores y ocupan preferentemente las posiciones ecuatoriales de las bipirámides trigonales.

H

C

H

C

H

H

O C OO C O

H

C

H

C

H

H

≈120°

Geometría molecular Enlaces Estructura de Lewis Molécula

triangular plana

lineal

3

2

C2H4

CO2

bipirámidetrigonal5SOF4

 PS

0

0

0

F

S

F

OF

F

F

S

F

OF

F

balancín4XeO2F2 1

F

Xe

F

O

OXe

O

O

F

F

2.5 El carácter iónico de los enlaces covalentes y el carácter covalente de los enlaces iónicos

La mayor parte de los enlaces no son ni puramente iónicos ni puramente covalentes. La ionicidad de unenlace covalente lo relacionamos con el concepto de electronegatividad , mientras que el carácter

covalente de un enlace iónico nos lleva al concepto de polarización.

 Electronegatividad. Los enlaces de moléculas homodiatómicas como H2 son covalentes puros. En una

molécula heterodiatómica como el HF, el par de enlace está más localizado sobre el flúor, produciendo

una carga parcial positiva sobre el hidrógeno, y una negativa sobre el flúor. Un enlace de este tipo se

llama polar , ya que presenta un momento dipolar eléctrico. La capacidad de cada átomo de atraer los

electrones del enlace covalente determina la distribución de cargas y se llama electronegatividad . A

continuación se describen algunos de los métodos desarrollados para cuantificar la electronegatividad.

 Escala de Pauling. Esta escala se basa en analizar el carácter iónico de un enlace covalente en términos

de resonancia y en suponer que la entalpía de resonancia iónica (figura 2.5) es una medida de la

participación iónica en el enlace y, por tanto, de la diferencia de electronegatividad entre los átomos.

42 | Enlace químico y estructura de la materia Licenciatura en Química

Page 41: Enlace Quimico Y Estructura de Materia

7/22/2019 Enlace Quimico Y Estructura de Materia

http://slidepdf.com/reader/full/enlace-quimico-y-estructura-de-materia 41/89

+– .

 . H .

 .

....FF

....

 .

 . H

F—H ↔ F– H+

H—Fcov. pura

H(g) + F(g)

565 kJ/mol

258 kJ/mol

307 kJ/mol

HF(g) Molécula real

Figura 2.5. Entalpía de resonancia iónica.① Formación de un enlace covalente puro HF a partir de los átomos gaseosos.

Esta entalpía no puede obtenerse experimentalmente y se estima suponiendoque es la media geométrica de las entalpías de enlace de sus átomos:

 ∆ H E(F–H, estimada) = ∆ H E(H–H)×∆ H E(F–F) = 153×436 = 258 kJ/mol

➁ Formación de un enlace "real" a partir de los átomos gaseosos. Esta es laentalpía de enlace obtenida experimentalmente.

➂ La diferencia entre ambas energías se debe a la participación de la estructuraiónica en el enlace real, por lo que se llama entalpía de resonancia iónica.

Pauling definió su escala de forma que la diferencia de electronegatividad se obtiene según la ecuación  χ F – χ H =

110

∆ H (resonancia iónica, en kJ/mol) =1

10307 = 1,9

fijando arbitrariamente χ H = 2,1, por lo que χ F = 4,0 (figura 2.6).

Los enlaces con una diferencia de electronegatividad menor de 1 se pueden considerar covalentes,

mientras que los enlaces con una diferencia mayor de 2 se pueden considerar iónicos (figura 2.7).

1613 142

3 4 5 6 7 8 9 10 11 12

15 17

< 1,0 2,0–2,4

1,0–1,4 2,5–2,9

1,5–1,9 3,0–4,0Li1,0

Ar

Mo1,6

P2,1

O3,5

F4,0

H2,1

V1,5

Ti1,4

Hf1,3

Nb1,5

Cr1,6

W1,5

Be1,5Mg1,2Ca1,0Sr1,0Ba1,0Ra

1,0

Sc1,3Y1,2La1,1

Zr1,3

Mn1,6Tc1,7Re1,7

Na1,0K0,9Rb0,9Cs0,8Fr

0,8

B2,0Al1,5

In1,6

S2,5Se2,4

Po1,9

Te2,1

C2,5Si1,8Ge1,9

Pb1,7

Sn1,8

Hg1,7

Cd1,6

Cu1,8

Ta1,4

Fe1,7Ru1,8Os1,9

Co1,7Rh1,8Ir

1,9

Ni1,8Pd1,8Pt1,8

Zn1,6

He

Cl3,0Br2,8I

2,5At2,1

Ne

Kr

Rn

Xe

N3,0

As2,1Sb1,9Bi1,8

Ga1,7

Tl1,6

Ag1,6Au1,9

Ac

1,1

Figura 2.6. Valores deelectronegatividad de loselementos, salvo gases nobles, en laescala de Pauling, ordenados segúnla tabla periódica. Laelectronegatividad aumenta al irhacia la derecha en un periodo y

hacia arriba en un grupo.

0%

20%

40%

60%

80%

100%

0 1 2 3Diferencia de electronegatividad

   C  a  r   á  c   t  e  r   i   ó  n   i  c  o

Enlaces iónicos

Enlacescovalentes

Figura 2.7. Gráfica que relaciona elcarácter iónico de un enlace con ladiferencia de electronegatividad delos dos átomos enlazados, según unafórmula propuesta por Pauling. Elcarácter iónico crece con ladiferencia de electronegatividad.Incluso en CsF (con la diferencia deelectronegatividad más grande), elenlace es solo 95% iónico.

 Escala de Allred–Rochow. Otra medida de la electronegatividad puede ser la fuerza ejercida por la carga

nuclear efectiva en la periferia del átomo. Suponiendo el átomo como una esfera de tamaño r , la fuerza

que el átomo ejerce sobre la densidad electrónica de otro átomo que se coloque en su vecindad es

proporcional a Z */ r 2, donde Z * es la carga nuclear efectiva sobre el orbital de valencia. Este es el origen

de la definición de electronegatividad de Allred–Rochow:

  χ  AR

= 0,359  Z *(r , enÅ)2 + 0,744

Los coeficientes 0,359 y 0,744 se introducen para que los valores obtenidos sean comparables con los dePauling. Con la escala de Allred–Rochow se gana conocimiento sobre la variación de la electronegativi-

dad a través de la tabla periódica, pues ya conocemos cómo varían Z* y r de elemento a elemento.

 Factores que afectan a la electronegatividad. Las escalas de Pauling y de Allred–Rochow asignan un

Universidad de Alcalá Tema 2: El enlace en las moléculas | 43

Page 42: Enlace Quimico Y Estructura de Materia

7/22/2019 Enlace Quimico Y Estructura de Materia

http://slidepdf.com/reader/full/enlace-quimico-y-estructura-de-materia 42/89

único valor de electronegatividad a cada átomo. Sin embargo, la capacidad de un átomo para atraer

electrones está influenciada por el ambiente que le rodea. Factores importantes son la carga del átomo, la

hibridación de los orbitales atómicos que participan en un enlace dado, otros sustituyentes, etc. Las

escalas de Mulliken y de Jaffé permiten asignar varios valores de electronegatividad a un mismo átomo.

 Escala de Mulliken. El compartimiento no igualitario de electrones entre átomos en un enlace de una

molécula hace que uno gane densidad electrónica (convirtiéndose en parte en un anión) y el otro la pierda(convirtiéndose en parte en un catión). Valores altos de la afinidad electrónica y de la energía de

ionización de un átomo significan que el átomo gana un electrón con facilidad y lo pierde con dificultad,

lo que sugiere que es muy electronegativo. Mulliken propuso como definición de electronegatividad

  χ M =

I v + EAv

2 I v = Energía de ionizaciónen el estado de valencia

 EAv = Afinidad electrónicaen el estado de valencia

Por estado de valencia puede entenderse el estado del átomo libre o el que se supone que tiene en una

molécula dada (figura 2.8). En ocasiones se emplea una ecuación ligeramente modificada, que da los

valores de electronegatividad de Mulliken en unidades de Pauling:

  χ M = 0,336 v + v2

– 0,207 I v y EAv, en eV

La electronegatividad de un átomo se incrementa con el carácter s del orbital híbrido que participa en el

enlace (figura 2.9).

   E  n  e  r  g   í  a

 E = 0

2s

2 p2sp

 I  p  I s  I sp = ( I s + I  p)/2

Figura 2.8. La energía de ionización experimental del berilio se relaciona conla configuración electrónica 2s2 del átomo libre en su estado de menor energía( I 

s). El átomo libre se puede excitar pasando un electrón del nivel 2s al 2 p y

medir la energía de ionización en este estado ( I  p). El estado de valencia delberilio en el BeCl2 es distinto al del átomo libre y se supone que correspondea situar los electrones de valencia en dos híbridos sp. La energía de ionizaciónen el estado de valencia ( I 

sp

) es la media de I 

s

e I 

 p

. El mejor valor de

electronegatividad para el berilio en el BeCl2 es el calculado a partir de laenergía de ionización y afinidad electrónica en el estado de valencia sp.

   E  n  e  r  g   í  a

Energía media de los orbitaless y p del átomo de carbono

2sp

2 p 2 p

2s

2 p

2sp22sp3

Orden ascendente de electrone- gatividad del átomo de carbono

HC≡CHH2C=CH2

sp3

sp2 sp Hibridación del orbital 

 del C unido al H 

H3C–CH3 Ejemplo

 Electronegatividad de Mulliken 2,48 2,75 3,29

Figura 2.9. Hibridación y electronegatividad. El carbono participa en cada enlace C—H de etano, eteno o etino con unhíbrido sp3, sp2 o sp, respectivamente. En ese orden, los híbridos del carbono disminuyen su energía y aumentan su

electronegatividad, tal como reflejan los valores de Mulliken. La electronegatividad del hidrógeno es 2,2, por lo que lapolaridad del enlace C—H es mucho más elevada en etino que en etano. Experimentalmente se observa que el hidrógeno del

etino es ácido, pero no el del etano.

 Escala de Jaffé. Relaciona la electronegatividad con la carga parcial que soporta el átomo al definirla

como la pendiente de la curva de la energía total del átomo frente a su carga (figura 2.10). La curva de

energía es aproximadamente cuadrática  E = aq + bq2

donde q es la carga y a y b son parámetros que varían con el átomo considerado. La electronegatividad de

Jaffé, definida como la pendiente de la curva anterior, es χ J = dE/dq = a + 2bq

44 | Enlace químico y estructura de la materia Licenciatura en Química

Page 43: Enlace Quimico Y Estructura de Materia

7/22/2019 Enlace Quimico Y Estructura de Materia

http://slidepdf.com/reader/full/enlace-quimico-y-estructura-de-materia 43/89

–5

0

5

10

15

20

–1 0 1Carga (q)

   E  n  e  r  g   í  a   t  o   t  a   l   (

  e  n  e   V   )

 I 

 EA

 χ M

Cl+

Cl

Cl–

 χ > 0(tendencia a atraer

electrones)    χ

  =   0

 χ < 0

Figura 2.10. La energía total del átomo de cloro en función desu carga. Se toma arbitrariamente como nula la energía de unátomo neutro. En valores positivos de carga, la energía es alta.Al disminuir la carga, la energía desciende progresivamente

hasta alcanzar un mínimo cerca de –1. A partir de este punto,aumenta. Este es el comportamiento esperado para un átomoelectronegativo.

Jaffé ha propuesto que la pendiente de la curva es unamedida de la electronegatividad. En valores positivos de carga,la electronegatividad es alta. Al disminuir la carga, desciendehasta llegar a un punto en que se hace nula: el cloro se hasaturado electrónicamente. A partir de ese punto, laelectronegatividad se hace negativa, indicando que la especiees electropositiva, tiende a dar electrones, no a atraerlos

El parámetro a mide la electronegatividad inherente o neutra del átomo ( χ J = a, si q = 0) y se simboliza

 χ M. El parámetro b es una medida de la dureza del átomo (figura 2.11), y se simboliza η. De forma que χ J = χ M + 2η q E = χ Mq + η q2

Cl: 2η = 11,30 V/e  χ M = 9,38 V

F: 2η = 17,36 V/e  χ M = 12,18 V

   E  n  e  r  g   í  a   t  o   t  a   l   (  e

  n  e   V   )

Carga (q)

–5

–4

–3

–2

–1

0

1

2

–1,2 –1,0 –0,8 –0,6 –0,4 –0,2 0 0,2

ClF

Figura 2.11. Curvas de energía total para el flúor y el cloro. Elflúor tiene un valor alto de η, lo que se refleja en lo curvado de sugráfica. El resultado es que, aunque el flúor es un átomo muyelectronegativo, se “satura” rápidamente (agota pronto su capacidadde atraer electrones, aprox. a –0,7). Por contra, el cloro, menoselectronegativo que el flúor, se satura con más dificultad (a más de–0,8). Obsérvese que, como resultado, el ion F– es menoselectronegativo (más electropositivo) que el ion Cl–.

Este hecho se relaciona con el pequeño tamaño del flúor. Losátomos pequeños poseen una capacidad limitada para donar oabsorber densidad electrónica. Su densidad electrónica es muycompacta y se deforma con dificultad, es decir, son muy duros. Elparámetro η es grande para los átomos duros como el flúor ypequeño para los blandos. Por ello se le denomina dureza.

Los átomos más duros son los átomos pequeños situados cercadel flúor. Los átomos más ligeros de un grupo son generalmenteduros y los más pesados blandos.

Tanto la electronegatividad neutra χ M como la dureza η están relacionadas con la energía de ionización yafinidad, es decir, con la energía de los orbitales frontera:

 I = E +1 – E 0 = χ M + η  EA = E 0 – E –1 = – (– χ M + η)

Sumando y restando ambas expresiones se obtiene que I + EA = ( χ M + η ) – (– χ M + η) = 2 χ M  I – EA =  I =  χ M + η + (– χ M + η) = 2η

La electronegatividad neutra es igual a ( I  +  EA)/2, lo que equivale a la energía media de HOMO y

LUMO (figura 2.12) (y también a la electronegatividad de Mulliken). La dureza es igual a ( I – EA)/2, lo

que equivale a la mitad de la diferencia de energía entre HOMO y LUMO (figura 2.12).

Universidad de Alcalá Tema 2: El enlace en las moléculas | 45

Page 44: Enlace Quimico Y Estructura de Materia

7/22/2019 Enlace Quimico Y Estructura de Materia

http://slidepdf.com/reader/full/enlace-quimico-y-estructura-de-materia 44/89

   E  n  e  r  g   í  a

 E = 0

 I 

 EA

χΜ

LUMO

HOM

Figura 2.12. Interpretación de la electronegatividad neutra y dela dureza en términos de niveles de energía de los orbitales

rontera (HOMO y LUMO).Electronegatividad absoluta  χ M = 1 / 2( I + EA)Dureza η = 1 / 2( I – EA)

Los átomos son blandos cuando sus orbitales frontera estánpróximos.HOMO = orbital ocupado de más alta energía.LUMO = orbital vacío de más baja energía.

_____________________________________________________________________________________________________Tabla 2.3. Electronegatividades de los elementos de los bloques s y p*

 χχ  χ  χ J χχ  χ  χ P  χχ  χ  χ AR Orb.  χχ  χ  χ M  χχ  χ  χ M 2ηηηη

 Elem. (en unidades de Pauling) (en V) (en V/e)

H 2,20 2,20 s 2,21 7,17 12,85Li 0,98 0,97 s 0,84 3,10 4,57Be 1,57 1,47 sp 1,40 4,78 7,59B 2,04 2,01 sp3 1,81 5,99 8,90

sp2 1,93 6,33 9,91C 2,55 2,50  p 1,75 5,80 10,93

sp3 2,48 7,98 13,27sp2 2,75 8,79 13,67sp 3,29 10,39 14,08

N 3,04 3,07  p 2,28 7,39 13,10sp3 3,68 11,54 14,78sp2 4,13 12,87 15,46sp 5,07 15,68 16,46

O 3,44 3,50  p 3,04 9,65 15,27sp3 4,93 15,25 18,28sp2 5,54 17,07 19,16

F 3,98 4,10  p 3,90 12,18 17,36Ne 4,84Na 0,93 1,01 s 0,74 2,80 4,67Mg 1,31 1,23 sp 1,17 4,09 6,02Al 1,61 1,47 sp2 1,64 5,47 6,72Si 1,90 1,74 sp3 2,25 7,30 9,04P 2,19 2,06  p 1,84 6,08 9,31

sp3 2,79 8,90 11,33S 2,58 2,44  p 2,28 7,39 10,01

sp3 3,21 10,14 10,73Cl 3,16 2,83  p 2,95 9,38 11,30

 χχ  χ  χ J χχ  χ  χ P  χχ  χ  χ AR Orb.  χχ  χ  χ M  χχ  χ  χ M 2ηηηη

 Elem. (en unidades de Pauling) (en V) (en V/e)

Ar 3,20K 0,82 0,91 s 0,77 2,90 2,88Ca 1,00 1,04 sp 0,99 3,30 4,74Ga 1,81 1,82 sp2 1,82 6,02 7,48Ge 2,01 2,02 sp3 2,50 8,07 6,82As 2,18 2,20 sp 1,59 5,34 8,03

sp3 2,58 8,30 8,99Se 2,55 2,48  p 2,18 7,10 9,16

sp3 3,07 9,76 11,05Br 2,96 2,74  p 2,62 8,40 9,40Kr 2,9 2,94Rb 0,82 0,89 s 0,50 2,09 4,18Sr 0,95 0,99 sp 0,85 3,14 4,41

In 1,78 1,49 sp2 1,57 5,28 6,79Sn 1,96 1,72 sp3 2,44 7,90 5,01Sb 2,05 1,82  p 1,46 4,96 7,57

sp3 2,64 8,48 9,37Te 2,10 2,01  p 2,08 6,81 8,46

sp3 3,04 9,66 10,91I 2,66 2,21  p 2,52 8,10 9,15Xe 2,60 2,40Cs 0,79 0,86Ba 0,89 0,97Tl 2,04 1,44Pb 2,33 1,55

Bi 2,02 1,67

* Datos extraídos de J. E. Huheey “Química Inorgánica. Principios de estructura y reactividad” Ed. Harla, 1981._____________________________________________________________________________________________________

 Electronegatividad de grupos. Los sustituyentes que

posee un átomo afectan a su electronegatividad. Así,

el carbono no presenta la misma capacidad de atraer

electrones en el grupo CH3 que en el CF3. Muchos

métodos como el de Mulliken–Jaffé son aplicables al

cálculo de electronegatividades de grupos. Losvalores calculados para algunos grupos se dan en la

tabla 2.4.

Cálculo de cargas parciales. La carga parcial que soporta un átomo en una molécula puede ser estimada

46 | Enlace químico y estructura de la materia Licenciatura en Química

Tabla 2.4. Electronegatividades de algunos grupos

Grupo χχ  χ  χ 

M(en unid. Pauling)

 χχ  χ  χ M

(en V)

2ηηηη

(en V/e)

CH3 2,28 7,45 4,64

CH2CH3 2,29 7,52 3,78

CF3 3,55 10,50 5,32

CCl3 2,83 10,12 4,33CBr3 2,59 9,87 3,96

CI3 2,51 9,43 3,77

Page 45: Enlace Quimico Y Estructura de Materia

7/22/2019 Enlace Quimico Y Estructura de Materia

http://slidepdf.com/reader/full/enlace-quimico-y-estructura-de-materia 45/89

experimentalmente con técnicas como la medida de momentos dipolares o calculada teóricamente por

métodos cuánticos. Alternativamente, Sanderson ha propuesto un método grosero pero simple para

calcular cargas parciales por igualación de las electronegatividades de Jaffé de los átomos de la molécula.

 Polarización. Reglas de Fajans. Un método alternativo al seguido hasta ahora para estudiar las situacio-

nes intermedias iónico–covalentes es partir de un enlace iónico y considerar cómo puede adquirir cierta

covalencia. Los cationes pueden deformar ( polarizar ) la distribución de carga de un anión, atrayéndolahacia sí. Si la deformación es muy grande, obtenemos un enlace covalente (figura 2.13). Los aniones muy

 polarizables o blandos son distorsionados fácilmente por cationes muy polarizantes, produciendo enlaces

de un elevado grado de covalencia. Las siguientes reglas cualitativas conocidas como reglas de Fajans

permiten interpretar el grado de covalencia de un enlace iónico:

• Los aniones grandes y de alta carga son blandos, es decir, muy polarizables.

• Los cationes pequeños y de alta carga polarizan más que los grandes y de baja carga.

• Los cationes de metales de transición son más polarizantes que los de los grupos principales.

+ –a)

c)

b)+ –

–+

Figura 2.13. Efectos de la polarización.(a) Par iónico idealizado sin polarizar.(b) Par iónico polarizado. Obsérvese que ambos, catión y anión, sepolarizan mutuamente. Sin embargo, la polarización significativapara el grado de covalencia es la del anión por el catión.(c) Polarización suficiente para formar un enlace. El catión hadeformado tanto al anión que se ha formado un enlace covalente.Las lineas punteadas simbolizan iones hipotéticos no polarizados.

2.6 Las moléculas polares

Una de las propiedades que depende de la forma de una molécula es su polaridad . Una molécula polar esuna molécula con un momento dipolar eléctrico no nulo. Es importante diferenciar entre enlace polar y

molécula polar . En algunas moléculas, la suma de los momentos dipolares de los enlaces puede ser nula,

dando lugar a una molécula no polar.

 Algunas moléculas apolares

F

B

F F H

H

C

HH

F

Cl

F

F

 Algunas moléculas polares

H

NH H

SnCl Cl

OHH

H

Cl

CH H

O C O

F

S

F

FF

FF

Cl Be Cl

Cl

P

Cl

Cl ClCl

F

S

F

FF

F

Br FF

FF

H

H

C C

H

H

FXe FF

F

Bibliografía

Atkins, págs. 277–317; Whitten, págs. 145–175, 447–449; Butler, págs. 68–71, 107–114; Shriver, págs.38–42, 123–130; Sharpe, págs. 139–152, 174–176.

Bibliografía complementaria

1 H. Weyl, “Simetría”, McGraw–Hill, Madrid, 1990, 130 páginas.

Universidad de Alcalá Tema 2: El enlace en las moléculas | 47

Page 46: Enlace Quimico Y Estructura de Materia

7/22/2019 Enlace Quimico Y Estructura de Materia

http://slidepdf.com/reader/full/enlace-quimico-y-estructura-de-materia 46/89

2 P. W. Atkins, “Molecules”, W. H. Freeman and Company, New York, 1987, 197 páginas.

Seminarios

iones

2.1 Da la fórmula de:

a) los cationes formados por K, Ca y Sc.

b) los aniones formados por S y Cl.c) los compuestos iónicos que contienen un catión de a) y un anión de b).

2.2 Escribe la fórmula de los haluros (aniones de los halógenos) con catión del grupo 1 en los que anión y

catión sean isoelectrónicos (mismo número de electrones).

2.3 Indica cuáles de las siguientes fórmulas no representa un compuesto iónico estable y por qué:

a) BaCl2,b) KF, c) SrS, d) Ca2O3, e) NaBr2, f) Li2S, g) AlF4.estructuras de Lewis, regla del octeto, carga formal, resonancia

2.4 El nitrógeno forma el trifluoruro NF3, mientras que NF5 no existe. Del fósforo se conocen ambos, PF3 y

PF5

.

a) Escribe las estructuras de Lewis del NF3, PF3 y PF5.

b) Da explicaciones al hecho de que el PF5 sea estable pero no el NF5.

c) A la luz de dichas explicaciones, ¿cuáles de las siguientes moléculas es de esperar que no existan?:

OF2, OF4, OF6, SF2, SF4, SF6. Escribe, en cada caso, la estructura de Lewis acompañada de los

comentarios oportunos.

2.5 ¿Cuáles de los siguientes elementos podrían tener octetos expandidos?

a) C, b) P, c) O, d) F, e) Cl, f) B, g) Se, h) Sn.

2.6 Define: octeto expandido, octeto incompleto, carga formal, par de enlace, par solitario.

2.7 ¿Qué significa el concepto de resonancia?2.8 Una de las estructuras de Lewis posibles para el ácido hidrazoico es H N N N

a) Dibuja otras dos estructuras de resonancia para esta molécula.

b) ¿EsN N H

Notra forma de resonancia para el ácido hidrazoico?

2.9 Selecciona de cada par de estructuras de Lewis, la que posiblemente haga una contribución dominante a

un híbrido de resonancia.

F :O: F

O :C: O O :C: O

O :S: O O :S: O

O :N: N O :N: N

: O :

P O

: O :

: O

: O :

P O

: O :

O

F :O: F

..

..........

..

........ .. ..

.. .. ..

.. ....

e)

d)

c)

b)

a)

o..

.  ..

..

 . .

..

3–3–

..

 . .

....

o

o

o

o....

..

......

2.10 a) Dibuja las tres estructuras resonantes posibles para el ion cianato, NCO–. En base a las cargas

formales, decide cuál es la estructura que tiene una distribución de carga más razonable.

b) El anión fulminato, CNO–, se diferencia del anterior en que el nitrógeno está en el centro y en que es

muy inestable (el fulminato de mercurio se utiliza como detonante). Da una explicación , en base a las

cargas formales, para esta inestabilidad.

2.11 Escribe estructuras de Lewis para las siguientes moléculas. En el caso que proceda, muestra lasestructuras resonantes.

a) BrF, b) S2, c) Cl2, d) P2, e) BeCl2, f) CS2, g) SO3, h) ICl3, i) BF3, j) CBr4, k) SiH4, l) NCl3,

m) SeF6, n) PF3, o) SF2, p) XeO3, q) SO2, r) SF6, s) ClO2, t) IF5, u) OF2, v) H2Te, w) N2F2, x) POBr3.

48 | Enlace químico y estructura de la materia Licenciatura en Química

Page 47: Enlace Quimico Y Estructura de Materia

7/22/2019 Enlace Quimico Y Estructura de Materia

http://slidepdf.com/reader/full/enlace-quimico-y-estructura-de-materia 47/89

2.12 Escribe estructuras de Lewis para los siguientes oxoácidos. En el caso que proceda, muestra las

estructuras resonantes.

a) HClO3, b) H2SO4, c) H3PO4.

2.13 Escribe estructuras de Lewis para los siguientes iones. En el caso que proceda, muestra las estructuras

resonantes.

a) CN–, b) BF4–, c) CH3–, d) PH4+, e) ClO–, f) SO42–, g) PO43–, h) HSO3–, i) CO32–, j) O2–.2.14 Escribe las estructuras de Lewis para las siguientes sustancias orgánicas:

a) CHClF2 (un clorofluorocarbono), b) HCOOH (ácido fórmico), c) H3C—CN (acetonitrilo),

d) H3C—OH (metanol), e) H2CCHCl (cloruro de vinilo).

2.15 Escribe estructuras de Lewis para los siguientes sales.

a) NaClO, b) Ba(NO3)2, c) Ca(NO2)2.

orden de enlace, energía de enlace, longitud de enlace

2.16 Define: orden de enlace, entalpía de enlace.

2.17 Escribe la estructura de Lewis del S2. ¿Es de esperar que la energía de enlace del S2 sea mayor o menor

que la del Cl2?.2.18 ¿En qué molécula es el enlace N–N más largo y en cuál más débil?

a) H2NNH2, b) N2, c) NNO.

2.19 Considera los enlaces carbono–oxígeno en el formaldehido (H2CO) y en el monóxido de carbono (CO).

¿En qué molécula es el enlace carbono–oxígeno más largo? ¿En cuál es más débil?

2.20 Considera los enlaces nitrógeno–oxígeno en NO2+ y en NO3

–. ¿En qué ion es el enlace

nitrógeno–oxígeno más corto?

2.21 Compara las longitudes de los enlaces carbono–oxígeno en el anión formiato, HCO2–, y en el anión

carbonato, CO3

2–. ¿En qué ion es el enlace carbono–oxígeno más corto?

2.22 Cuando en disolución se disocia el ácido acético, se produce el ion acetato. Los dos enlaces

carbono–oxígeno de este ion tienen la misma longitud. Escribe las estructuras resonantes del ion acetato

que expliquen este hecho.

2.23 En la molécula ONNO2 hay dos distancias de enlace nitrógeno–oxígeno claramente diferentes, una más

corta (1,14 Å) y la otra más larga (1,22 Å). ¿Podrías decir a qué enlace o enlaces N–O corresponde la

distancia más corta?

teoría de la repulsión de los pares electrónicos de la capa de valencia

Ejercicios interactivos en la página Web http://www2.uah.es/edejesus/interactivos/inicio.htm

2.24 Predice la forma de cada una de las siguientes moléculas:a) BeCl2, b) CS2, c) SO3, d) ICl3, e) BF3, f) CBr4, g) SiH4, h) NCl3, i) SeF6, j) PF3, k) SF2, l) XeO3

m) SO2, n) SF6, o) ClO2, p) IF5, q) OF2, r) H2Te, s) N2F2, t) POBr3.

2.25 Predice la forma de cada uno de los siguientes iones:

a) H3O+, b) ClO2

–, c) NO2–, d) SO3

2–, e) NH4+, f) SO4

2–, g) PO43–.

2.26 Predice la forma alrededor del átomo escrito en cursiva:

a) C H3—CH3, b) C H2=CH2, c) C H≡CH, d) C H3—CH2—CH3, e) HC≡C —CH=CH2.

2.27 Estima el valor de los ángulos de enlace marcados con un arco y una letra minúscula:

a

a

b bc

a

bd 

ba

c

a

b(a) (b) (c) (d) (e)

H

C

H

C

H

H

H C C

O

O HH

H

H C C

O

HC

H

H

H

H

H C C

O

O OH

H

N

O

O

OC

H

H

C

H

C

H

Universidad de Alcalá Tema 2: El enlace en las moléculas | 49

Page 48: Enlace Quimico Y Estructura de Materia

7/22/2019 Enlace Quimico Y Estructura de Materia

http://slidepdf.com/reader/full/enlace-quimico-y-estructura-de-materia 48/89

2.28 Para cada ion o molécula, di un átomo A de los grupos principales para el que se correspondan la fórmula

y la estructura:

a) [AH3] triangular plana b) [AH3] piramidal c) [AH3]– piramidal d) [AH4]– tetraédrica

e) [AH4]2– tetraédrica f) [AH2] lineal g) [AH2] angular

2.29 Contrariamente a lo que predice la VSEPR, en el Li2O, la disposición de los dos litios con respecto al

oxígeno es lineal. Busca una explicación a este hecho, teniendo en cuenta el modelo de enlace en el quese basa esta teoría.

2.30 Para la molécula F2C=O, ¿el ángulo FCF es mayor o menor que el FCO?

2.31 Compara los ángulos de enlace que se predicen para los siguientes pares de moléculas:

a) CH4, NH3, b) OF2, OCl2, c) OF2, OH2, d) NH3, NF3, e) PH3, NH3.

electronegatividad y polarización

2.32 ¿Para cuáles de las siguientes moléculas será más grande la diferencia entre la energía de enlace

experimental y la energía de enlace estimada haciendo la media geométrica de las energías de enlace de

sus átomos constituyentes?: IBr, BrF, HBr, Cl2, NO.

2.33 Las electronegatividades de Mulliken para el B sp3 (por ejemplo en BF4–) y el B sp2 (por ejemplo enBF3) son 1,81 y 1,93, respectivamente. Relaciona este resultado con la energía de los orbitales atómicos.

2.34 Teniendo en cuenta el valor de electronegatividad de cada átomo, indica cuál es el enlace más polar de

cada par y qué sentido tiene la polaridad:

a) C–N o N–O; b) P–S o S–Cl; c) Sn–I o C–I.

2.35Ordena los siguientes enlaces de menor a mayor polaridad, señalando en cada caso el átomo que está

cargado más negativamente: P–O, P–P, P–C, P–N.

2.36 La molécula de debajo es la acroleína, el material de partida para algunos plásticos.

O :CHCCH H H ..

¿Cuál es el enlace más polar en la molécula?

2.37 ¿Cuál de los siguientes iones presenta un mayor poder polarizante?

a) Cu2+ o Ag+; b) K+ o Be2+; c) Ti4+ o Li+.

2.38 Ordena los siguientes cationes en orden creciente de poder polarizante: K+ , Mg2+, Al3+, Cs+.

2.39 Ordena los siguientes aniones en orden creciente de polarizabilidad: F–, P3–, N3–, O2–.

2.30 Elige el compuesto de cada pareja que tenga mayor carácter iónico:

a) CaO o MgO, b) CaO o CaS, c) MgO o MgS, d) CaS o MgS.

2.41 Clasifica los siguientes compuestos como principalmente iónicos o significativamente covalentes:a) AgF, b) AgI, c) AlCl3, d) AlF3, e) BeCl2, f) CaCl2, g) FeCl3, h) Fe2O3.

2.42 Señala el estado de oxidación y la carga formal de los átomos del HCl. ¿Con qué modelo de enlace,

iónico o covalente, se corresponde cada uno? ¿Y la carga parcial?

polaridad de las moléculas

2.43 ¿Cuáles de las siguientes moléculas es de esperar que tengan momento dipolar?:

a) H2, b) O2, c) HI, d) HF.

2.44 Predice si serán o no polares las siguientes moléculas:

a) ICl, b) H2S, c) CH4, d) PCl3, e) HgCl2, f) CH3OH, g) HCHO (formaldehido).

2.45 Ordena los siguientes isómeros de mayor a menor momento dipolar, indicando cuáles no son polares:

50 | Enlace químico y estructura de la materia Licenciatura en Química

Page 49: Enlace Quimico Y Estructura de Materia

7/22/2019 Enlace Quimico Y Estructura de Materia

http://slidepdf.com/reader/full/enlace-quimico-y-estructura-de-materia 49/89

Cl Cl ClCl

ClCl

ClC

Cl

CHH

HC

Cl

CClH

HC

Cl

CHCl

a) b)

II III I II IIII

Problemasentalpías de enlace

2.1 Estima la cantidad de calor que hay que suministrar, a 298 K, para disociar en estado gaseoso las

moléculas a) H2O, b) NH3 y c) CH4 en átomos, conociendo las siguientes entalpías de enlace, a dicha

temperatura:

∆ H E(O–H) = 463 kJ mol–1, ∆ H E(N–H) = 388 kJ mol–1, ∆ H E(C–H) = 412 kJ mol–1.

2.2 Estima la entalpía de la reacción Cl2(g) + 3F2(g) → 2ClF3(g), a partir de las entalpías de enlace:

∆ H E(Cl–Cl) = 242 kJ mol–1, ∆ H E(F–F) = 158 kJ mol–1, ∆ H E(Cl–F) = 485 kJ mol–1.

2.3 Estima las entalpías de formación de cada molécula en fase gaseosa, a partir de las entalpías de enlace:

a) HCl, b) NH2OH.∆ H E(H–H) = 436 kJ mol–1, ∆ H E(Cl–Cl) = 242 kJ mol–1, ∆ H E(O=O) = 496 kJ mol–1, ∆ H E(N≡N) = 944 kJ

mol–1, ∆ H E(H–Cl) = 431 kJ mol–1, ∆ H E(N–H) = 388 kJ mol–1, ∆ H E(O–H) = 463 kJ mol–1,

∆ H E(N–O) = 200 kJ mol–1.

2.4 Estima las entalpías de combustión de cada uno de los siguientes compuestos gaseosos:

a) acetileno, b) octano, c) metanol.

∆ H E(O=O) = 496 kJ mol–1, ∆ H E(C–H) = 412 kJ mol–1, ∆ H E(C–C) = 348 kJ mol–1, ∆ H E(C≡C) = 837 kJ

mol–1, ∆ H E(C–O) = 360 kJ mol–1, ∆ H E(C=O) = 743 kJ mol–1, ∆ H E(O–H) = 463 kJ mol–1.

escalas de electronegatividad

2.5 Calcula la diferencia de electronegatividad entre el cloro y el hidrógeno en la escala de Pauling, a partirde la entalpía de enlace del HCl(g) (431 kJ mol–1), del Cl2(g) (242 kJ mol–1) y del H2(g) (436 kJ mol–1).

2.6 Calcula las cargas parciales de H y Cl en la molécula de cloruro de hidrógeno, por el método de

Sanderson de igualación de electronegatividades.

Soluciones a los seminarios2.1 a) K+, Ca2+, Sc3+; b) S2–, Cl–; c) KCl, CaS, etc.2.2 NaF, KCl, RbBr, CsI.2.3 Ca2O3, NaBr2 y AlF4 porque tienen cationes Na2+, Ca3+ y Al4+. Para obtener estos cationes se precisa arrancar un electrón de

una estructura electrónica de gas noble.

2.4 a) b) En ambos casos hay 10 electrones en torno al N ó P. El N pertenece al 2ºperíodo, por lo que sólo puede alojar 8 electrones en su capa de valencia(orbitales 2s y 2 p). Usar los orbitales 3s es energéticamente muydesfavorable. El P dispone de los orbitales 3s y 3 p pero también de los 3d .

c) No es de esperar que existan OF4 y OF6 ya que el oxígeno es del 2º período.2.5 P, Cl, Se y Sn, ya que son del 3º periodo o superior.2.6 Ver teoría.2.7 Ver teoría.

2.8 a) H N N NH N N N H N N N2–

b) No. Sería otra molécula diferente. Las estructuras resonantes deben tener la misma distribución de los átomos y sólo puedendiferenciarse en la distribución de los pares electrónicos.

Universidad de Alcalá Tema 2: El enlace en las moléculas | 51

F N F

F

F P F

F

P

FF FF F

Page 50: Enlace Quimico Y Estructura de Materia

7/22/2019 Enlace Quimico Y Estructura de Materia

http://slidepdf.com/reader/full/enlace-quimico-y-estructura-de-materia 50/89

2.9

O :C: O

O :S: O

O :N: N

: O :

P O

: O :

: O

F :O: F

e) carga formal –1 sobre el átomomás electronegativo.

d) carga formal –1 sobre el átomomás electronegativo

c) cargas formales cero

a) por la regla del octeto

b) cargas formales cero..

3–

..

2.10 N C O N C O N C O

C N O C N O C N O

2–

La primera estructura tiene la distribución más razonable. Latercera es poco importante en la descripción de la molécula.

3–

La primera estructura tiene la distribución más razonable.La segunda y tercera son poco importantes en ladescripción de la molécula.

2–

a)

b)

2.11 S S Cl ClF Br P P Cl Be Cl S C S

F B

F

F

O S O

O

H Si

H

H

H

Cl N

Cl

Cl

F P

F

F F S F O Xe O

O

O S O

Cl I

Cl

Cl Br C

Br

Br

Br

Se

FF

FF F

F

S

FF

FF F

FO Cl O

F FNNH Te HF O FIF

FF F

FBr P

Br

O

Br

a) b) c) d) e) f) g)

h) i) j) k) l) m)

n) o) p) q) r) s)

t) u) v) w) x)

2.12 O Cl O

O

H O S O

O

H

O

H O P O

O

H

O

H

H

c)b)a)

2.13

C N F B

F

F

F

H C

H

H

H P

H

H

H

Cl O

O S O

O

O

O P O

O

O

O S O H

O

O C OO

O O

Hay otras 5estructuras resonantesde la misma energía.

Hay otras 3estructuras resonantesde la misma energía.

2–

Hay otra estructura

resonante de lamisma energía.

3–

Hay otras 2estructuras resonantesde la misma energía.

2–

Hay otra estructuraresonante de lamisma energía.

g)

f)c) d)b)

 j)

a)

i)h)

g)

2.14

Cl C

F

F

H

H C O

O

H H C

H

C

H

N H C

H

O

H

H H C

H

C H

Cl

b)a) d)c) e)

2.15

Cl OO N O

OO N Oa) Na+ b) Ba2+ 2 c) Ca2+ 2

Hay otras 2estructuras resonantesde la misma energía.

Hay otra estructuraresonante de lamisma energía.

2.16 Ver teoría

2.17 Ver estructuras de Lewis en seminario 2.17 b) y c). Es mayor la energía de enlace en S 2 (enlace doble) que en Cl2 (enlacesencillo).2.18 Más largo y débil en H2NNH2 (enlace sencillo).

H N N H

H HN N N N ON N OOE(NN) = 1 OE(NN) = 3 OE(NN) = 2—3

52 | Enlace químico y estructura de la materia Licenciatura en Química

Page 51: Enlace Quimico Y Estructura de Materia

7/22/2019 Enlace Quimico Y Estructura de Materia

http://slidepdf.com/reader/full/enlace-quimico-y-estructura-de-materia 51/89

2.19 Más corto y más fuerte en el CO (mayor orden de enlace).

H C H

OC OOE(CO) = 2 OE(CO) = 3

2.20 En el NO2+.

O N O O N O

O

O N O

O

O N O

O

OE(NO) = 2 OE(NO) = 11 / 3

2.21 En el anión formiato.

O C O

O

O C O

O

O C O

O

H C O

O

H C O

O

OE(CO) = 11 / 2

2– 2– 2–

OE(CO) = 11 / 32.22

H C C

H

H

O

O H C C

H

H

O

O

2.23

O N NO

O O N NO

O O N NO

OOE = 1,5; 1,22ÅOE = 2; 1,14 Å

2.24 Ver las estructuras de Lewis de estas moléculas en el seminario 4.17. Se da la geometría molecular y, entre paréntesis, lageometría electrónica cuando no coincide con la molecular: a) lineal; b) lineal; c) triangular plana; d) forma de T (bipirámidetrigonal); e) triangular plana, f) tetraédrica; g) tetraédrica; h) pirámide trigonal (tetraédrica); i) octaédrica; j) pirámide trigonal(tetraédrica); k) angular (tetraédrica); l) pirámide trigonal (tetraédrica); m) angular (triangular plana); n) octaédrica; o) angular(tetraédrica); p) pirámide cuadrada (octaédrica); q) angular (tetraédrica); r) angular (tetraédrica); s) angular (triangular plana);t) tetraédrica.

2.25 Se da la geometría molecular y, entre paréntesis, la geometría electrónica cuando no coincide con la molecular: a) pirámidetrigonal (tetraédrica); b) angular (tetraédrica); c) angular (trigonal); d) piramidal (tetraédrica); e) tetraédrica; f) tetraédrica; g)tetraédrica.

2.26 a) tetraédrica; b) triangular plana; c) lineal; d) tetraédrica; e) lineal.2.27 a) a ≈ 120°, b ≈ 120°; b) a ≈ 120°, b ≈ 109,5°, c ≈ 109,5°; c) a ≈ 109,5°, b ≈ 120°; d) a ≈ 120°, b ≈ 109,5°, c ≈ 120°; d) a ≈

120°, b ≈ 109,5°, c ≈ 120°, d ≈ 109,5°; e) a ≈ 120°, b ≈ 120°.2.28 a) elementos del grupo 13, b) grupo 15, c) grupo 14, d) grupo 13, e) grupo 2, f) grupo 2, g) grupos 14 y 16.2.29 El enlace es predominantemente iónico y los iones de carga igual tienden a alejarse mutuamente.2.30 Menor.2.31 a) H–C–H > H–N–H (por par electrónico sobre N); b) F–O–F < Cl–O–Cl (F más electronegativo que Cl); c) F–O–F < H–O–H

(F más electronegativo que H); d) H–N–H > F–N–F (F más electronegativo que H); e) H–P–H < H–N–H (N máselectronegativo que P).

2.32 En HBr ya que es la molécula en la que hay mayor diferencia de electronegatividad entre sus átomos (ver discusión sobre laescala de Pauling en teoría).

2.33 El híbrido sp

3

tiene menor carácter s y, por tanto, mayor energía que el híbrido sp

2

. Por ello, el B cuando participa con unhíbrido sp3 en un enlace es menos electronegativo que cuando participa con un híbrido sp2.2.34 a) Cδ +–Nδ – (∆ χ = 0,5) es semejante en polaridad a Nδ +–Oδ – (∆ χ = 0,5); b) Pδ +–Sδ – (∆ χ = 0,4) es semejante en polaridad a

Sδ +–Clδ – (∆ χ = 0,5); c) Snδ +–Iδ – (∆ χ = 0,7) es más polar que C–I (∆ χ = 0).2.35 P–P < P–C < P–N < P – O (ver valores de electronegatividades).2.36 El C=O.2.37 a) Cu2+; b) Be2+; c) Ti4+.2.38 El poder polarizante de un catión aumenta al aumentar su carga iónica, y, a igual carga, al disminuir su tamaño: Al3+ > Mg2+ >

K+ > Cs+.2.39 La polarizabilidad de un anión aumenta al aumenta al aumentar su carga iónica, y, a igual carga, al aumentar su tamaño: P 3– >

N3– > O2– > F–.2.40 a) CaO (Ca es menos electronegativo que Mg), b) CaO (O es más electronegativo que S), c) MgO, d) CaS.

2.41 Principalmente iónicos (diferencia de electronegatividades mayor de 2): AgF, AlF3, CaCl2.Significativamente covalentes (diferencia de electronegatividades menor de 2): AgI, AlCl3, BeCl2, FeCl3, Fe2O3.

2.42 Estados de oxidación: I para H, -I para Cl. Serían sus cargas parciales si el enlace fuera 100% iónico.Carga formal: 0 para H, 0 para Cl. Serían sus cargas parciales si el enlace fuera 100% covalente.Las cargas parciales son las cargas que tiene realmente cada átomo y estarán entre 0 y +1 para el hidrógeno, y entre 0 y –1

Universidad de Alcalá Tema 2: El enlace en las moléculas | 53

Page 52: Enlace Quimico Y Estructura de Materia

7/22/2019 Enlace Quimico Y Estructura de Materia

http://slidepdf.com/reader/full/enlace-quimico-y-estructura-de-materia 52/89

para el cloro.2.43 HI y HF.2.44 Serán polares a), b), d), f) y g).2.45 a) I > II > III (III no es polar); b) II no es polar.

Soluciones a los problemas2.1 a) ∆ H = 926 kJ mol–1; b) ∆ H = 1164 kJ mol–1;

c) ∆ H = 1648 kJ mol–1.2.2 ∆ H = –2194 kJ mol–1.2.3 a) ∆ H f = –92 kJ mol–1, b) ∆ H f = –65 kJ mol–1.2.4 a) ∆ H = –997 kJ mol–1, b) ∆ H = –4170 kJ mol–1,

c) ∆ H = –535 kJ mol–1.2.5  χ Cl– χ H = 1,0.2.6 δ H = –δ Cl = 0,09.

54 | Enlace químico y estructura de la materia Licenciatura en Química

Page 53: Enlace Quimico Y Estructura de Materia

7/22/2019 Enlace Quimico Y Estructura de Materia

http://slidepdf.com/reader/full/enlace-quimico-y-estructura-de-materia 53/89

© 2003, Ernesto de Jesús Alcañiz_________________________________________________________________________________________

3 Orbitales y enlaces químicos3.1 La descripción mecano-cuántica del enlace químico3.2 La teoría del enlace de valencia (TEV)

3.3 La teoría de los orbitales moleculares (TOM)3.4 Ácidos y bases de Lewis

_________________________________________________________________________________________

3.1 La descripción mecano-cuántica del enlace químicoLa resolución exacta de la ecuación de Schrödinger es imposible para moléculas polielectrónicas, por lo

se precisa realizar algunas aproximaciones. Estas aproximaciones se basan en dos modelos alternativos.

La teoría del enlace de valencia construye la función de onda de la molécula vista como un

conjunto de pares electrónicos localizados en un átomo o entre dos átomos. Se trata de la versión

mecano-cuántica de las ideas de Lewis y comparte con ellas conceptos como la resonancia.

La teoría de los orbitales moleculares construye la función de onda de la molécula como un

conjunto de orbitales moleculares deslocalizados por toda la molécula.

En ambas teorías, el solapamiento de orbitales atómicos juega un papel fundamental. Cuandoparticipan orbitales direccionales, el solapamiento depende de la dirección de interacción:

B B BAA A

Máximo Solapamiento Solapamiento intermedio Solapamiento nulo

3.2 La teoría del enlace de valencia (TEV)La teoría de enlace de valencia supone que un enlace entre dos átomos se forma por el solapamiento de

dos orbitales, uno de cada átomo, si el total de electrones que ocupan ambos orbitales es de dos.Un enlace óptimo exige un máximo solapamiento entre los orbitales participantes, por lo que cada

átomo debe tener orbitales adecuados dirigidos hacia los otros átomos con los que se enlaza. A menudo

esto no es así, y en la formación de un enlace no participa un orbital atómico puro sino una mezcla

(hibridación) de orbitales atómicos adecuada para que los orbitales híbridos se encuentren orientados en

las direcciones de enlace. Por ejemplo, en una molécula lineal como BeCl2, los enlaces Be–Cl estarán

previsiblemente formados por orbitales híbridos sp (50% s, 50% p) del berilio:

equivale a &&AA

s p sp sp

A A

En la tabla 3.1, se da la hibridación necesaria para cada tipo de geometría. En los ejemplos, la hibridación

dada es la de los orbitales del átomo central.

Tabla 3.1. Hibridación de orbitales atómicos y geometría electrónicaGeometría electrónica Orbitales necesarios Orbitales híbridos EjemplosLineal s + p sp BeCl2, CO2

Triangular plana s + p + p sp2 BF3, SnCl2, C2H4

Tetraedro s + p + p + p sp3 (o sd 3) CH4, NH3, H2O

Bipirámide trigonal s + p + p + p + d  sp3d (o spd 3) PCl5, SF4, ClF3, I3 –

Octaedro s + p + p + p + d + d  sp3

d 2

SF6, BrF5, XeF4

La energía de un orbital híbrido es la media aritmética de la de los orbitales mezclados. Es importante

resaltar que la tabla anterior predice los orbitales híbridos que participarán en los enlaces a partir de la

geometría de molécula (estimada por la VSEPR u obtenida experimentalmente), y no viceversa.

Page 54: Enlace Quimico Y Estructura de Materia

7/22/2019 Enlace Quimico Y Estructura de Materia

http://slidepdf.com/reader/full/enlace-quimico-y-estructura-de-materia 54/89

 Enlaces sencillos. Se forman por solapamiento frontal de orbitales, llamado solapamiento σ.

2 p(Be)2 p

BeH2

2sp(Be)1s(H)

H

Be

Be

1s

2s 2 p

2sp

Enlaces σσσσ Orbitales 2 p vacíos

NH3

CH4

2sp3(N) 1s(H)

1s(H)

2sp3(C)

Enlaces σσσσ

2sp3

2 p2s

1s

N

N

H

2 p(B)

H

C

C

1s

2s 2 p

2sp3

Enlaces σσσσ

2 p

Orbital 2 p vacíoEnlaces σσσσ

2sp2

2 p2s

1s

B

B

H1s(H)

2sp2(B)

BH3

Universidad de Alcalá Tema 3: Orbitales y enlaces químicos | 55

Page 55: Enlace Quimico Y Estructura de Materia

7/22/2019 Enlace Quimico Y Estructura de Materia

http://slidepdf.com/reader/full/enlace-quimico-y-estructura-de-materia 55/89

 Enlaces dobles y triples. Sólo se puede formar un enlace entre dos átomos por solapamiento σ. En los

enlaces múltiples, un enlace se forma por solapamiento frontal σ y el resto por solapamiento lateral π.

1s(H)C2H4

2 p

Enlaces ππππEnlaces σσσσ

2sp

2

2 p2s

1s

C

C

H

2sp2(C)2 p(C)

 Moléculas con direcciones de enlace no equivalentes. En moléculas como el NH3 o el C2H4, todos los

híbridos no tienen por qué ser exactamente iguales, ya que juegan papeles diferentes (en el NH3, 3 sirven

enlazan N con H y otro aloja un par solitario; en el C2H4, dos enlazan C con H y otro C con C). En

dichos casos se pueden esperar separaciones de la hibridación prevista. Para el H2O o el H2S, podemospensar en dos modelos extremos (ver figura 3.1). Posiblemente el modelo con hibridación sea más real

para el H2O, ya que H–O–H = 104°, y el modelo sin hibridación lo sea para el H2S, ya que H–S–H = 92°.

Sólo mediante el cálculo mecano-cuántico correspondiente se podría confirmar esa previsión.

 Modelo SIN hibridación Modelo CON hibridación

X X

ns npnsp3

Carácter en orbitales de X del enlace X–H 

arácter en orbitales de X de los pares solitarios

25% s75% p

25% s75% p

0% s100% p

50% s50% p

 Angulo H–X–H acorde con hibridación 109,5° 90°

X = O (n = 2), S (n = 3)

1s(H)np(X)

(Por claridad, no sehan dibujado losorbitales s y p delos pares solitarios)

Figura 3.1. Dosmodelos para el enlace X–Hen el agua y en el sulfurode hidrógeno.

1s(H)

nsp3(X)

más caracter p

más caracter s

 Hibridación y electronegatividad. Recordemos (ver tema 2) que la capacidad de un átomo para atraer loselectrones de un enlace covalente puede ser modificada por varios factores, de manera que la

electronegatividad de un átomo es mayor cuanto mayor es su estado de oxidación y cuanto mayor es el

carácter s del orbital híbrido que participa en el enlace.

3.3 La teoría de los orbitales molecularesMuchas moléculas no son descritas correctamente mediante la teoría de Lewis. Un ejemplo es el

diborano (B2H6) que es un compuesto electro-deficiente: no hay suficientes electrones de valencia para

poder asignarle una estructura de Lewis. Otro ejemplo es el O2 que es paramagnético, mientras que la

teoría de Lewis prevee que sea diamagnético.Orbitales enlazantes y antienlazantes. En la teoría de orbitales moleculares, los orbitales atómicos al

solapar forman orbitales moleculares. Dos orbitales atómicos solapan dando un orbital molecular

enlazante y otro antienlazante. En un orbital enlazante, los electrones que lo ocupan tienen una alta

56 | Enlace químico y estructura de la materia Licenciatura en Química

Page 56: Enlace Quimico Y Estructura de Materia

7/22/2019 Enlace Quimico Y Estructura de Materia

http://slidepdf.com/reader/full/enlace-quimico-y-estructura-de-materia 56/89

probabilidad de situarse entre los átomos, por lo que su llenado estabiliza la molécula. En un orbital

antienlazante, los electrones que lo ocupan tienen una baja probabilidad de situarse entre los átomos, por

lo que su llenado desestabiliza la molécula. En un diagrama de interacción (fig. 3.2) se muestran los

niveles de energía de los orbitales atómicos y moleculares, así como los orbitales atómicos que

contribuyen a cada orbital molecular.

∆*

∆+

1s(A) 1s(B)

=

σσ σ σ s

A B BA

A BBA

σσ σ σ s∗∗ ∗ ∗ 

=

1s(B)1s(A)

 –

1s

   E  n  e  r  g   í  a  c  r  e  c   i  e  n   t  e

HA H–H HB

σ s

σ s∗

1sFigura 3.2. Diagrama deinteracción para lamolécula de hidrógeno.

Como ∆<∆*, las interacciones entre 2 orbitales son estabilizadoras si son a 2 electrones y

desestabilizadoras si son a 4 electrones. El orden de enlace es igual a

(número de electrones en orbitales enlazantes – número de electrones en orbitales antienlazantes)/2.

La interacción entre dos orbitales atómicos es mayor cuanto mayor sea su solapamiento y menor su

diferencia de energía.

 Diagramas de interacción para moléculas homodiatómicas del segundo período. (Figura 3.3). El

diagrama de la figura 3.3 es cualitativamente correcto sólo cuando se puede despreciar la interacción

entre el orbital 2s de un átomo y el 2 pz del otro. Si dicha interacción no es despreciable, los orbitales

molecularesσ s y σ z se mezclan entre sí. El resultado de su mezcla es, de acuerdo a una propiedad general

de la mecánica cuántica, un alejamiento de sus energías: el orbital σ s refuerza su carácter enlazante,

disminuyendo algo su energía, mientras que el orbital σ z pierde carácter enlazante, aumentando suenergía. Lo mismo ocurre con los orbitales σ s* y σ z*. El resultado puede ser un cambio en el diagrama

cualitativo tal como se muestra en el diagrama de la figura 3.4. Esta mezcla en la TOM entre 2s y 2 pz es

equivalente a la hibridación s–p en la TEV.

±π x

π x∗

BA

A B

BA

2 p x(B)2 px(A)

±σ z

σ z∗

BA

A BBA

2 p z (B)2 pz(A)

2s

XBX–XXA

2s

2s(A) 2s(B)

A B

BA

A B

   E  n  e  r  g   í  a  c  r  e  c   i  e  n

   t  e

 y x

 z

σ s∗

σ s

σ s∗

σ s

±

2 p2 p

σ z

π x π y

π x∗π y∗

σ z∗

Figura 3.3. Diagrama de interacción cualitativo para moléculas homodiatómicas del segundo período en las

que la interacción entre el orbital 2s de un átomo con el 2 pz del otro es pequeña. Diagramas de interacción para moléculas heterodiatómicas. (Ver figura 3.5) El átomo más

electronegativo (el oxígeno en este caso) tiene los orbitales más bajos en energía. Los orbitales

moleculares enlazantes tienen más participación de los orbitales atómicos del oxígeno, pues están más

Universidad de Alcalá Tema 3: Orbitales y enlaces químicos | 57

Page 57: Enlace Quimico Y Estructura de Materia

7/22/2019 Enlace Quimico Y Estructura de Materia

http://slidepdf.com/reader/full/enlace-quimico-y-estructura-de-materia 57/89

próximos en energía, que del nitrógeno (matemáticamenteψ σ z

= aψ  pz(N) + bψ  pz

(O), donde a < b) y los

antienlazantes del nitrógeno (ψ σ z*

= bψ  pz(N) – aψ  pz

(O), donde a < b. Por ello, los orbitales enlazantes es-

tán más localizados sobre el oxígeno y los antienlazantes sobre el nitrógeno. Como hay más orbitales

enlazantes llenos que antienlazantes llenos, el resultado es que la densidad electrónica total está más

localizada sobre el oxígeno.

σσ σ σ z∗∗∗∗

ππ π π x∗∗∗∗ππ π π y∗∗∗∗

ππ π π x ππ π π y

σσ σ σ z

2 p 2 p

σσ σ σ s

σσ σ σ s∗∗∗∗

 z

 x  y

   E  n  e  r  g   í  a  c  r  e  c   i  e  n   t  e

2s

XA X–X XB

2s

Figura 3.4. Diagrama de interaccióncualitativo para moléculashomodiatómicas del segundo período enlas que la interacción entre el orbital 2s deun átomo con el 2 pz del otro es grande.

Figura 3.5.

Diagrama de interacción para la

molécula NO.

2s

ON–ON

2s

   E  n  e  r  g   í  a  c  r  e  c   i  e  n   t  e

 y x

 z

σ s∗

σ s

2 p

2 p

σ z

π x π y

π x∗π y∗

σ z∗

ON

N O

σ z∗

σ z

ON

N O

π x∗

π x

3.4. Ácidos y bases de Lewis Ácidos y bases de Brønsted (revisión). En 1923 Brønsted y Lowry propusieron la siguiente definición de

ácido y base:

Un ácido de Brønsted es cualquier molécula o ion dadora de iones hidrógeno, H+.

Una base de Brønsted es cualquier molécula o ion aceptora de iones hidrógeno, H+.

Las sustancias que pueden comportarse como ácidos y como bases de Brønsted se llaman anfipróticas.

Ejemplos: H2O, HCO3 –, HS –, HSO4

 –. En una reacción ácido-base se transfieren iones hidrógeno del

ácido a la base HA + B A – + BH+, donde HA y BH+ son los ácidos conjudados de las bases B y A –._____________________________________________________________________________________________________ Ácidos y bases conjugados Ácido HCl HNO3 H2SO4 HSO4

 – H2CO3 CH3COOH H2O OH – H3O+ H2S HS – NH3 NH4+

 Base Cl – NO3 – HSO4

 – SO42– HCO3

 – CH3COO – OH – O2– H2O HS – S2– NH2 – NH3_____________________________________________________________________________________________________

58 | Enlace químico y estructura de la materia Licenciatura en Química

Page 58: Enlace Quimico Y Estructura de Materia

7/22/2019 Enlace Quimico Y Estructura de Materia

http://slidepdf.com/reader/full/enlace-quimico-y-estructura-de-materia 58/89

La constante del equilibrio HA + B A – + BH+ será tanto mayor cuanto mayor sea la fuerza del

ácido y de la base. Los ácidos (o bases) se clasifican por su fuerza ácida midiendo la constante de

equilibrio frente a una base (o ácido) de referencia, por ejemplo el agua._____________________________________________________________________________________________________Constantes de ionización de ácidos en agua a 25°C, HA + H  2O A – + H  3O+

 HA HClO4 H2SO4 HCl HNO3 H3O+ HF CH3COOH H2CO3 NH4+ H2O

 K (mol l  –1 ) ∞ ∞ ∞ ∞ 1 3,5 10 –4 1,8 10 –5 4,3 10 –7 5,6 10 –10 1,0 10 –14_____________________________________________________________________________________________________Constantes de ionización de bases en agua a 25°C, B + H  2O OH  – + BH +

 B H2O CO(NH2)2 CH3COO – NH3 OH – NaOH NH2 – O2–

 K (mol l  –1 ) 1,0 10 –14 1,3 10 –14 5,6 10 –10 1,8 10 –5 1 ∞ ∞ ∞_____________________________________________________________________________________________________

 Ácidos y bases de Lewis. Lewis formuló en 1923 una definición alternativa a la de Brønsted:

Un ácido de Lewis es un ion o molécula aceptor de pares electrónicos.

Una base de Lewis es un ion o molécula dador de pares electrónicos.

Se denominan anfóteras a las sustancias que pueden actuar como ácidos y como bases de Lewis. En una

reacción ácido-base, el ácido y la base comparten el par electrónico aportado por la base, formando un

enlace covalente, A + :B A—B. La definición de una base de Brønsted como aceptora de H+, H+ +B BH+, no es más que un caso particular de base de Lewis, donde H+ es el ácido de Lewis. Sin

embargo, muchos ácidos de Lewis no son ácidos de Brønsted. Por ejemplo, BF3 (BF3 + :NH3

F3B—NH3) o SO3 (SO3 + H2O: H2SO4).

 Fuerza de ácidos y bases de Lewis: ácidos y bases duros y blandos, efectos estéricos. En la definición de

Lewis, la fuerza de un ácido se puede evaluar mediante la constante del equilibrio A + :B A—B,

donde B es una base de referencia. En realidad, la escala de fuerza ácida depende de la base escogida

como referencia, de forma que un ácido puede ser más fuerte que otro frente a una base pero más débil

frente a otra. Para los ácidos y bases de Lewis se han desarrollado reglas cualitativas que permiten

preveer su fuerza y estimar qué clases de bases preferirá un ácido determinado y viceversa. Estas reglasse basan en dividir las bases en:

• bases duras, que son aquellas que tienen un átomo dador cuya densidad electrónica se polariza (se

deforma) difícilmente. Por ello, normalmente el átomo dador es pequeño y muy electronegativo (N, O y

F). Ejemplos: F –, OH –, O2–, H2O, R2O (éteres), NH3.

• bases blandas que son aquellas que tienen un átomo dador cuya densidad electrónica se polariza (se

deforma) con facilidad. Los átomos dadores son generalmente menos electronegativos y mayores que los

de las bases duras (elementos no cabecera de los grupos 15 a 17). Ejemplos: Br –, I –, CN –, SCN –, H –, R –,

RS –, CO, RNC.

En general, las bases blandas deberían ser más fuertes que las duras pues ceden con mayor facilidad

el par electrónico. Ahora bien, se ha observado que ciertos ácidos forman enlaces más estables con las

bases duras que con las blandas. Los ácidos que en proporción se enlazan mejor con las bases duras

reciben el nombre de ácidos duros. Los ácidos que en proporción se enlazan mejor con las bases blandas

reciben el nombre de ácidos blandos. La siguiente tabla muestra una lista de ácidos blandos y duros._____________________________________________________________________________________________________ Ácidos duros Ácidos intermedios Ácidos blandosH+, Li+, Na+, K+ Cu+, Ag+, Au+, Tl+, Hg2

2+, Cs+

Be2+, Mg2+, Ca2+, Sr2+, Sn2+ Fe2+, Co2+, Ni2+, Cu2+, Zn2+, Pb2+ Pd2+, Cd2+, Pt2+, Hg2+

Al3+ Tl3+

Si4+ I+, Br+, HO+, RO+_____________________________________________________________________________________________________

Para explicar este comportamiento experimental, se pueden utilizar dos modelos complementarios:

• modelo iónico–covalente. Los ácidos duros se encuentran con preferencia entre los cationes metálicos

pequeños y/o con alta carga, mientras que los ácidos blandos se encuentran con preferencia entre los

Universidad de Alcalá Tema 3: Orbitales y enlaces químicos | 59

Page 59: Enlace Quimico Y Estructura de Materia

7/22/2019 Enlace Quimico Y Estructura de Materia

http://slidepdf.com/reader/full/enlace-quimico-y-estructura-de-materia 59/89

cationes grandes y/o con baja carga. Una base dura es poco polarizable por lo que tenderá a formar

enlaces con una importante componente iónica. Estos enlaces importantemente iónicos serán más fuertes

cuando el catión sea un ácido duro, es decir pequeño y/o con alta carga. Una base blanda tenderá a

formar enlaces con una importante componente covalente. Estos enlaces covalentes serán más fuertes con

un ácido blando (ver escala de electronegatividad de Jaffé en tema 2).

• modelo de enlace ππππ. Es un modelo apropiado para los cationes de los metales de transición. Entre loscationes de transición blandos predominan aquellos que tienen electrones en orbitales d  débilmente

sujetos (a consecuencia de la baja carga y/o gran tamaño). Las bases blandas contienen ligandos con

orbitales d vacíos en el átomo dador (P, As, S, I, etc.) o tienen orbitales π∗ vacíos (CO). El enlace entre

un ácido blando y una base blanda se encuentra reforzado por una contribución π. La característica

principal de los ácidos duros es tener orbitales d  vacíos que tienen tendencia a aceptar electrones y

pueden recibir pares electrónicos de ligandos con átomo dador pequeño como O ó F.

ácido duro - base duraácido blando - base blanda

A

..

B

π

:σσ :

π

B

..

A

Bibliografía

Seminarioshibridación

3.1 Para las siguientes moléculas y iones, escribe una estructura de Lewis razonable (o más de una, si creesque mediante la resonancia de ellas se mejora la descripción de la molécula), cuenta el número de

electrones del átomo central, propón una estructura (en base a la VSEPR), y estima la hibridación

previsible que tienen los orbitales atómicos del átomo central.

a) SF4, b) SF6, c) ICl4 –, d) O3, e) CO3

2–.

3.2 Indica la geometría de la molécula, la hibridación más probable para los orbitales atómicos de valencia

del átomo central y la polaridad de cada una de las siguientes moléculas:

a) CS2, b) CBr4, c) H2Te, d) SiH4, e) OF2, f) SF2, g) BF3, h) XeOF4.

3.3 Señala la hibridación de cada átomo de carbono en las siguientes moléculas:

a) CH3CH2CH3, b) CH2=CH—CH3, c) CH3—C≡CH, d) CH2=C=C=CH2, e) HC≡C—C≡CH,f) C6H6 (benceno), g) HCHO (formaldehido).

3.4 ¿Cuál es la geometría de los entornos de cada uno de los átomos del ácido acético (H3C–COOH)? ¿Cuál

es la hibridación de los orbitales atómicos de cada uno de los átomos? ¿Qué enlace carbono–oxígeno será

más largo?

3.5 Da la composición de cada uno de los enlaces de cada molécula en la forma, por ejemplo, (Csp, H1s):

a) C2H6, b) C2H2, c) HCN, d) CH3OH, e) NO3 –.

3.6 En el etileno (H2C=CH2) los cuatro hidrógenos están en el mismo plano. Explica si la VSEPR y la TEV

 justifican dicha observación experimental.

3.7 Describe el enlace en las moléculas de CH4, NH3 y H2O, de acuerdo a la teoría de enlace de valencia.

¿Qué diferencias pueden señalarse entre ellas?

3.8 La adición de cierto carácter d  a un orbital híbrido, ¿aumenta o disminuye la electronegatividad del

átomo?

60 | Enlace químico y estructura de la materia Licenciatura en Química

Page 60: Enlace Quimico Y Estructura de Materia

7/22/2019 Enlace Quimico Y Estructura de Materia

http://slidepdf.com/reader/full/enlace-quimico-y-estructura-de-materia 60/89

3.9 Calcula cualitativamente el carácter s del orbital híbrido que el átomo de carbono emplea en los orbitales

que están dirigidos hacia los átomos de H y F, en los siguientes compuestos:Compuesto Ángulo H–C–H Ángulo F–C–FCH3F 110–112°CH2F2 119± 0,4° 108,3±0,1°CHF3 108,8±0,75°

solapamiento

3.10 ¿Cuáles de los siguientes orbitales atómicos situados sobre átomos diferentes en la geometría

representada solapan entre sí?

a) b) c) d) e) f)

3.11 Di cuando el solapamiento de los siguientes orbitales situados en átomos vecinos producirá un enlace σ y

cuando π. El eje z es el que une los dos átomos.

a) (1s, 1s), b) (2 pz, 2 pz), c) (2 px, 2 px), d) (2s, 2 pz), e) (3d z2, 3d z2)

teoría de orbitales moleculares

3.12 ¿Cuáles de los siguientes orbitales moleculares son enlazantes y cuáles antienlazantes? ¿Cuáles son σ y

cuáles π ?

A B A B A B A B CA B

a) b) c) d) e)

3.13 ¿Cuál de las siguientes combinaciones de los átomos A(a) y A(b) debe conducir al orbital molecularrepresentado en la figura? ¿Es un orbital enlazante o antienlazante? ¿Qué combinación debe conducir al

correspondiente orbital antienlazante?

A(a) A(b)

A(a) A(b)A(a) A(b)

A(a) A(b)

A(a) A(b)

A(a) A(b)

a)

b)

c)

d)

e) Orbital Molecular

3.14 ¿Cuál es el orden de enlace del catión Cl2+? ¿Sería su energía de enlace mayor o menor que la del Cl2?

¿Es paramagnético?

3.15 Escribe las configuraciones electrónicas moleculares para las especies O22–, O2

 –, O2 y O2+.

a) Diferencia las que sean diamagnéticas de las que sean paramagnéticas.

b) Ordénalas de mayor a menor distancia de enlace.

c) Ordénalas de menor a mayor energía de enlace.

3.16 ¿Cuál de los siguientes procesos endotérmicos debe tener mayor entalpía?

a) O2 –(g) → O –(g) + O(g); b) O2(g) → O(g) + O(g); c) O2

+(g) → O+(g) + O(g)

3.17 ¿Podrían existir las siguientes moléculas (aplica la teoría que consideres más adecuada)?a) He2

2+, b) C2, c) Ne2, d) F2, e) F2+.

3.18 ¿Cuál de las siguientes especies debe tener el enlace más largo?

a) CN+, b) CN, c) CN –.

Universidad de Alcalá Tema 3: Orbitales y enlaces químicos | 61

Page 61: Enlace Quimico Y Estructura de Materia

7/22/2019 Enlace Quimico Y Estructura de Materia

http://slidepdf.com/reader/full/enlace-quimico-y-estructura-de-materia 61/89

3.19 ¿Cuáles de las siguientes moléculas son paramagnéticas? (aplica la teoría que consideres más adecuada):

a) CO, b) Cl2, c) NO, d) N2, e) BN, f) NO –.

3.20 La figura siguiente representa el diagrama de interacción para el fluoruro de hidrógeno. Contesta a las

siguientes preguntas empleando únicamente la información aportada por el diagrama.

a) ¿Qué interpretación en términos de electronegatividad puedes dar al hecho de que los orbitales del

flúor tengan menor energía que los del hidrógeno?b) ¿Por qué son no enlazantes los orbitales moleculares procedentes de los orbitales atómicos 2 px y 2 py

del flúor?

c) ¿Por qué el orbital 2s del flúor no interacciona con el 1s del hidrógeno y, en cambio, si lo hace el 2 pz?

d) Coloca los electrones adecuados en el diagrama.

e) ¿Cuál es el orden de enlace?

f) ¿Cómo es la forma aproximada del orbital enlazante? ¿De qué orbitales atómicos procede?

g) ¿Es el enlace H–F un enlace covalente puro?

h) ¿Quien tendrá la carga parcial negativa?

F

   E  n

  e  r  g   í  a  c  r  e  c   i  e  n   t  e

H H—F

Energías relativas de los orbitales atómicos y molecularesdel HF. La energía del electrón 1s del átomo de hidrógenoes –1310 kJ mol-1 (la primera entalpía de ionización del

hidrógeno es +1310 kJ mol -1) y la de los orbitales 2 p delflúor es –1682 kJ mol -1 (la primera entalpía de ionizacióndel flúor es +1682 kJ mol -1)

2s

π n

x, π ny

σz

σ∗z

s

2 px, 2 py, 2 pz

 y x

 z

3.21 Dibuja el diagrama de interacción para el LiH.

3.22 ¿Qué diferencias existen entre la teoría de enlace de valencia y la teoría de orbitales moleculares al

estudiar las moléculas de B2 y de O2?

3.23 Discute la existencia o no de las siguientes moléculas, clasificándolas por orden de estabilidad: H2+, H2 –,H2

2– y H2.

ácidos y bases de Lewis

3.24 ¿Es SiO2 + CaO CaSiO3 una reacción ácido–base de Brønsted? ¿Y de Lewis?

3.25 Clasifica las siguientes sustancias como ácidos o bases de Lewis: NH3, PH3, Fe3+, H2O, CO, BF3, Cl –,

CO32–, BeCl2.

3.26 En las siguiente reacción, el dióxido de carbono se comporta ¿como ácido o como base de Lewis?

CO2(g) + H2O(l) HCO3 –(ac) + OH –(ac)

3.27 ¿Cuál de los dos aductos F3B—NH3 y F3B—PH3 es más estable?

3.28 La siguiente lista recoge algunos minerales comunes de los que se extraen metales. ¿Podrías sugerir quémetales son blandos, cuáles son duros y cuáles tienen un carácter intermedio?

óxidos: hematites, Fe2O3; magnetita, Fe3O4; bauxita, Al2O3; periclas, MgO; sílice, SiO2sulfuros: calcopirita, CuFeS2; calcocita, Cu2S; esfalerita, PbS; pirita de hierro, FeS2; cinabrio, HgS

62 | Enlace químico y estructura de la materia Licenciatura en Química

Page 62: Enlace Quimico Y Estructura de Materia

7/22/2019 Enlace Quimico Y Estructura de Materia

http://slidepdf.com/reader/full/enlace-quimico-y-estructura-de-materia 62/89

Soluciones a los seminarios3.1

SF

F

F

FS

FF

FF F

FS

F

FF F

FS

F F

F

F

F

I

ClCl Cl

ClCl

ICl Cl

Cl

Geometría molecular:plano–cuadrada

En torno al O central hay

2 Pares de enlace σ y 1

par solitario

Geometría electrónica:

Triangular plana

Hibridación: sp2

d)

En torno al S hay 6 Pares

de enlace

Geometría electrónica:

Octaédrica

Hibridación: d 2sp3Geometría molecular:

octaédrica

b)

En torno al I hay 4 Pares de

enlace y 2 Pares solitarios

Geometría electrónica:

Octaédrica

Hibridación: d 2sp3

O C O

O

c)

Geometría molecular:angular

C

O

O O

En torno al S hay 4 Pares

de enlace y 1 Par solitario

Geometría electrónica:

Bipirámide trigonal

Hibridación: dsp3

a)

Hay otras 2 est.resonantes de lamisma energía.

En torno al C hay 3 Pares

de enlace σ (y uno π)

Geometría electrónica:

Triangular plana

Hibridación: sp2

2 –

O O O

O O O

Geometría molecular:tetraedro deformado

e)

Geometría molecular:triangular plana

OO O

3.2 (Entre paréntesis se indica la geometría electrónica cuando es distinta de la molecular). a) lineal, sp, no polar; b) tetraédrica,sp3, no polar; c) angular (tetraédrica), sp3, polar; d) tetraédrica, sp3, no polar; e) angular (tetraédrica), sp3, polar; f) angular(tetraédrica), sp3, polar; g) triangular plana, sp2, no polar; h) pirámide de base cuadrada (octaédrica), d 2sp3, polar.

3.3 a) sp3, sp3, sp3; b) sp2, sp2, sp3; c) sp3, sp, sp; d) sp2, sp, sp, sp2; e) sp, sp, sp; f) todos sp2; g) sp2.3.4 De izquierda a derecha: C: tetraédrica, sp3 ; C: triangular plana, sp2; O: angular, sp3. El enlace C–OH es más largo que C=O.3.5 a) 6 enlaces sigma (C2sp3, H1s), 1 enlace sigma (C2sp3, C2sp3); b) 2 enlaces sigma (C2sp, H1s), 1 enlace sigma (C2sp, C2sp),

2 enlaces pi (C2 p, C2 p) (C2 p, C2 p); c) 1 enlace sigma (C2sp, H1s), 1 enlace sigma (C2sp, N2sp), 2 enlaces pi (C2 p, N2 p)(C2 p, N2 p); d) 3 enlaces sigma (C2sp3, H1s), 1 enlace sigma (C2sp3, O2sp3), 1 enlace sigma (O2sp3, H1s); e) 3 enlaces sigma(N2sp2, O2 p), 1 enlace pi (N2 p, O2 p).

3.6 Ver teoría.3.7 Los cuatro híbridos sp3 únicamente son puros e iguales entre sí en el metano.3.8 Disminuye si es un orbital d de la misma capa (su energía es mayor que la de los orbitales s y p). Aumenta si es un orbital d de

la capa anterior (su energía es menor que la de los orbitales s y p).3.9 Posiblemente, los híbridos sp3 del carbono tienen mayor participación s en los enlaces con el H que con el F.

3.10 b, c y f.3.11 Sigma: a, b, d y e; Pi: c.3.12 Enlazantes: a y c; antienlazantes: b y d. Sigma: a y b. Pi: c, d y e.3.13 El c.3.14 (Rellenar el apropiado diagrama de orbitales moleculares). O.E.(Cl2+) = 1,5, O.E.(Cl2) = 1. La energía de enlace del Cl2+ será

mayor que la del Cl2. El Cl2+ es paramagnético mientras que el Cl2 es diamagnético.

3.15 a) O2+ KK(σ2s)2(σ*2s)2(σ2pz)2(π2px,π2py)4(π*2px,π*2py)1 Paramagnética O.E. = 2,5

O2 KK(σ2s)2(σ*2s)2(σ2pz)2(π2px,π2py)4(π*2px,π*2py)2 Paramagnética O.E. = 2O2

 – KK(σ2s)2(σ*2s)2(σ2pz)2(π2px,π2py)4(π*2px,π*2py)3 Paramagnética O.E. = 1,5

O22–

KK(σ2s)2

(σ*2s)2

(σ2pz)2

(π2px,π2py)4

(π*2px,π*2py)4

Diamagnética O.E. = 1b) O22– > O2

 – > O2 > O2+.

c) O22– < O2

 – < O2 < O2+.

3.16 El c) ya que la mayor energía de enlace corresponde al O2+ (ver 3.21).

3.17 Aplicamos la teoría de orbitales moleculares (rellena el diagrama apropiado). a) O.E. = 1, si; b) O.E. = 2, si; c) O.E. = 0, no; d)O.E. = 1, si; e) O.E. = 1,5, si.

3.18 Aplicamos la teoría de orbitales moleculares (rellena el diagrama apropiado). a) O.E. = 2; b) O.E. = 2,5; c) O.E. = 3. El CN+

tiene el menor O.E. por lo que tendrá la mayor longitud de enlace.3.19 Aplicamos la teoría de orbitales moleculares (rellena el diagrama apropiado). a) diamagnética, b) diamagnética, c)

paramagnética, d) diamagnética, e) diamagnética, f) paramagnética.3.20 a) Que los orbitales de valencia del F tengan menor energía que los del H está relacionado con su mayor electronegatividad.

b) Porque no solapan con el orbital s del hidrógeno (su simetría es π , mientras que la del orbital s del H es σ ).

c) En realidad sí interacciona, pero la interacción fundamental es la del orbital pz pues su energía es más cercana a la del s(H).d) (2s)2 (σ z)2 (π xn, π yn)4

e) O.E. = 1.f) Procede del orbital s del H y del pz del F. Hay más participación del segundo que del primero.g) No, el orbital enlazante está más centrado sobre el flúor que sobre el hidrógeno.

Universidad de Alcalá Tema 3: Orbitales y enlaces químicos | 63

Page 63: Enlace Quimico Y Estructura de Materia

7/22/2019 Enlace Quimico Y Estructura de Materia

http://slidepdf.com/reader/full/enlace-quimico-y-estructura-de-materia 63/89

h) De acuerdo a lo dicho, el F tendrá la carga parcial negativa.3.21

HLi–HLi

   E  n  e  r  g   í  a  c  r  e  c   i  e  n   t  e

1s

2 p

 z

 x

2s

σ 

 σ s∗

σ z∗

π xn π yn

 y

Diagrama de interacción para el hidruro de litio.

3.22 Según la teoría de orbitales moleculares, B2 tiene OE = 1 y es paramagnético; O 2 tiene OE = 2 y es paramagnética (en

concordancia con los datos experimentales). Una aplicación intuitiva de la teoría de enlace de valencia, haría pensar que B2 yO2 son diamagnéticas.3.23 H2 (OE =1) > H2

+ (OE =0,5) , H2 – (OE =0,5). H2

2– (OE =0) no es estable.3.24 Es una reacción ácido–base de Lewis pero no de Brønsted.3.25 Ácidos de Lewis: Fe3+, BF3, BeCl2. Bases de Lewis: NH3, PH3, H2O, CO, Cl –, CO3

2–.3.26 Como ácido de Lewis.3.27 El del borano con el amoníaco, ya que ambos son duros.3.28 Podríamos sugerir que son duros: Al, Mg, Si; blandos: Pb, Hg, Cu; intermedios: Fe.

64 | Enlace químico y estructura de la materia Licenciatura en Química

Page 64: Enlace Quimico Y Estructura de Materia

7/22/2019 Enlace Quimico Y Estructura de Materia

http://slidepdf.com/reader/full/enlace-quimico-y-estructura-de-materia 64/89

© 2003, Ernesto de Jesús Alcañiz_________________________________________________________________________________________

4 Sólidos4.1 Comparación entre las propiedades de sólidos, líquidos y gases

4.2 Fuerzas entre moléculas y iones

4.3 Estructuras de los cristales

4.4 Bandas de orbitales moleculares

4.5 Sólidos metálicos

4.6 Sólidos iónicos: enlace y estructura

4.7 La formación de los sólidos iónicos: entalpía de red

4.8 Sólidos moleculares

4.9 Sólidos covalentes

4.10 Comparación entre las propiedades de los distintos tipos de sólidos_________________________________________________________________________________________

4.1 Comparación entre las propiedades de sólidos, líquidos y gases

 Fuerzas Orden Distancias Propiedades

Gas Despreciables frente a

la agitación térmica

Continuo

movimiento al azar

Largas,

desiguales

Volúmenes variables, fluidos, bajas densidades,

se difunden con rapidez Líquido Intermedias Semiorden Cortas,

desigualesVolúmenes fijos, fluidos, altas densidades, sedifunden a través de otros líquidos

Sólido Grandes frente a laagitación térmica

Elevado orden Cortas,iguales

Volúmenes fijos, no fluidos, altas densidades, sedifunden muy lentamente a través de otros sólidos

4.2 Fuerzas entre moléculas y iones

Las moléculas pueden atraerse entre sí mediante fuerzas relativamente débiles que se conocen con el

nombre genérico de fuerzas de Van der Waals. A continuación se citan los tipos más importantes. Se

incluye también la fuerza ion–dipolo, que es una fuerza de atracción entre un ion y una molécula.

 Fuerzas dipolo–dipolo. Las fuerzas dipolo–dipolo son atracciones entre los dipolos eléctricos de

moléculas polares.

 Fuerzas dipolo–dipolo inducido. Las fuerzas dipolo–dipolo inducido se dan entre una molécula polar y

otra no polar. La molécula polar induce un dipolo en la no polar.

 Fuerzas de dispersión o de London. Son atracciones que se dan entre cualquier tipo de moléculas debido

a los dipolos instantáneos que se forman producidos por las fluctuaciones en la densidad electrónica que

rodea a los átomos. Las fuerzas de London dependen de la forma de la molécula. Para moléculas de

forma semejante, crecen con la masa molecular y con la polarizabilidad ya que esos dos factores facilitan

la fluctuación de los electrones (tabla 4.1).Tabla 4.1. Puntos de fusión y ebullición de gases no polares.Sustancia He Ne Ar Kr Xe F2 Cl2 Br2 I2

 Punto de fusión (°C)  –270 –249 –189 –157 –112 –220 –101 –7 114 Punto de ebullición (°C)  –269 –246 –186 –153 –108 –188 –34 59 184

 Fuerzas ion–dipolo. En una fuerza ion–dipolo, un catión atrae la carga parcial negativa de un dipolo

eléctrico o un anión atrae la carga parcial positiva del dipolo. Esta fuerza es la responsable de la

hidratación de los iones en agua. La hidratación del catión persiste muchas veces en el sólido (p.ej.

Na2CO3⋅10 H2O). Un catión se hidrata más fuertemente cuanto menor sea su tamaño y mayor su carga.

 El enlace de hidrógeno. Es una interacción primordialmente de tipo dipolo–dipolo especialmente fuerte,que se da entre un átomo de hidrógeno con carga parcial positiva y un átomo electronegativo pequeño

(normalmente N, O o F). La presencia de enlace de hidrogeno en el H 2O, NH3 y HF, justifica sus

anormales puntos de fusión y ebullición (figura 4.1). Es también el responsable de la alta capacidad

Page 65: Enlace Quimico Y Estructura de Materia

7/22/2019 Enlace Quimico Y Estructura de Materia

http://slidepdf.com/reader/full/enlace-quimico-y-estructura-de-materia 65/89

calorífica molar del agua líquida, así como de sus elevados calores de vaporización y de fusión.

 –180

 –120

 –60

0

60

120

2 3 4 5

H2O

HF

NH3

Periodo

Grupo 17Grupo 16Grupo 15Grupo 14

CH4

SiH4

GeH4

SnH4H2S

H2Se H2Te

HCl

HBr

HISbH3

   T  e  m  p  e  r  a   t  u  r  a

   (   °   C   )

PH3

AsH3

Figura 4.1. Temperaturas deebullición de algunos compuestossimples de hidrógeno. Las líneasconectan moléculas que contienenátomos del mismo periodo. Obsérveseel efecto del enlace de hidrógeno enlos puntos de ebullición del agua,fluoruro de hidrógeno y amoníaco.

 Energía de las interacciones interiónicas e intermoleculares. La tabla 4.2 ordena los tipos de

interacciones por su fortaleza. La energía de las interacciones dipolo–dipolo y de London disminuye muyrápidamente con la distancia. En la tabla 4.3 se desglosa la contribución aproximada de cada tipo de

fuerza a la energía total de interacción intermolecular para algunas sustancias. La contribución de las

fuerzas de London puede ser mayor que la de las fuerzas dipolo–dipolo incluso en moléculas polares.

Tabla 4.2. Energía de las interacciones interiónicas e intermoleculares

Tipo de interacción Principales factores responsables de laenergía de interacción

 Energía Típica*en kJ/mol 

 Dependencia de laenergía con la distancia

Ion–ion Carga de los iones 250 1/d 

Enlace de hidrógeno (tipo especial de interacción dipolo–dipolo) 20

Ion–dipolo Carga del ion; momento dipolar 151/d 

2

Dipolo–dipolo Momentos dipolares 2 a 0,3 1/d 3 a 1/d 6 

Dispersión o London Polarizabilidades 2 1/d 6 

Dipolo–dipolo inducido Momento dipolar; polarizabilidad 0,05

* A una distancia de 5,00 Å

Tabla 4.3. Contribuciones aproximadas a la energía total de interacción entre moléculas en kJ/mol.

 Molécula Momento dipolar  permanente (D)

 Energía permanentedipolo–dipolo Energía de London Energía total  

 Entalpía molar devaporización (kJ/mol)

Ar 0 0 8,5 8,5 6,7CO 0,1 ≈ 0 8,7 8,7 8,0

HCl 1,03 3,3 17,8 21 16,2NH3 1,5 13* 16,3 29 27,4

H2O 1,8 36* 10,9 47 40,7

* Existe enlace de hidrógeno

4.3 Estructuras de los cristales

Sólidos cristalinos y sólidos amorfos. La mayoría de sólidos se caracterizan por ordenamientos de par-

tículas que vibran en torno a posiciones fijas en sus estructuras. Estos sólidos se denominan sólidos cris-

talinos. Algunos sólidos, denominados amorfos, no tienen estructuras ordenadas y bien definidas. Los

vidrios se consideran sólidos amorfos o líquidos superenfriados, ya que fluyen aunque con suma lentitud.

Los sólidos cristalinos se fragmentan a lo largo de los planos de la red cristalina, por lo que los

fragmentos mantienen similares ángulos interfaciales y características estructurales. Los sólidos amorfos

se rompen de forma irregular dando bordes disparejos y ángulos irregulares. Los sólidos cristalinos

tienen puntos de fusión bien definidos, mientras que los amorfos pueden fundir a temperaturas diferentes

Universidad de Alcalá Tema 4: Estructura de la materia | 66

Page 66: Enlace Quimico Y Estructura de Materia

7/22/2019 Enlace Quimico Y Estructura de Materia

http://slidepdf.com/reader/full/enlace-quimico-y-estructura-de-materia 66/89

en las diversas porciones de la muestra, al irse venciendo las distintas fuerzas que unen sus partículas.

Celdilla unidad. Las estructuras de los sólidos cristalinos se discuten mejor en términos de celdilla

unidad. Ésta es una parte del cristal que reproduce su globalidad por apilamiento repetido (figura 4.2).

Las celdillas apiladas se relacionan mediante operaciones de simetría puras de traslación.

(a) (b)

Figura 4.2. Un sólido de dos dimensiones ydos opciones de celdilla unidad. Cualquiera delas dos reproduce el cristal completo, pero seprefiere generalmente la opción (a) ya quemuestra la máxima simetría de la estructura,contrariamente a (b).

 Red cristalina. La unidad asimétrica es el átomo, ion, molécula o grupo de ellos a partir del cual se

construye el cristal. La disposición de las unidades asimétricas en un cristal se describe mediante una

matriz de puntos llamada red cristalina (figura 4.3).

(a) (b) (c)

unidadas m tr ca

Figura 4.3. (a) Representación de los puntos

de la red del NaCl, cuya estructura se muestraen (b) o (c). Cada punto representa lalocalización de una unidad asimétrica Na+Cl –.La situación del punto en la unidad asimétricaes arbitrario. En (b) se ha puesto en el ion Cl –;en (c) en un punto intermedio entre los ionesNa+ y Cl –, con igual resultado.

Sistemas cristalinos y tipos de redes. Para que una celdilla unidad por repetición pueda reproducir la red

cristalina, debe de pertenecer a uno de los 7 sistemas cristalinos de la tabla 4.4.Tabla 4.4. Dimensiones de las celdillas unidad para los siete sistemas cristalinos

Sistema Longitudes Ángulos Ejemplo

a

b

cα 

β 

γ 

celdilla unidad 

Cúbico a = b = c αα α α = ββ β β = γγ γ γ = 90° NaCl (sal de roca)

Tetragonal a = b ≠≠≠≠ c αα α α = ββ β β = γγ γ γ = 90° TiO2 (rutilo)

Ortorrómbico a ≠≠≠≠ b ≠≠≠≠ c αα α α = ββ β β = γγ γ γ = 90° MgSO4⋅7H2O (epsomita)

Monoclínico a ≠≠≠≠ b ≠≠≠≠ c αα α α = γγ γ γ = 90°; ββ β β ≠≠≠≠ 90° CaSO4⋅2H2O (yeso)

Triclínico a ≠≠≠≠ b ≠≠≠≠ c αα α α ≠≠≠≠ ββ β β ≠≠≠≠ γγ γ γ ≠≠≠≠ 90° K2Cr2O7

Hexagonal a = b ≠≠≠≠ c αα α α = ββ β β = 90°; γγ γ γ = 120° SiO2 (sílice)

Romboédrico a = b = c αα α α = ββ β β = γγ γ γ ≠≠≠≠ 90° CaCO3 (calcita)

Además de en los vértices, los puntos de la red también pueden aparecer en otras posiciones de la

celdillas unidad, produciendo 14 tipos de redes cristalinas (figura 4.4).

 I s o m o r f i s m o y p o l i m o r f i s m o . D o s s u s t a n c i a s q u e c r i s t a l i z a n e n e l m i s m o t i p o d e r e d c r i s t a l i n a s e d i c e q u e s o n

i s o m o r f a s. U n a s u s t a n c i a p r e s e n t a p o l i m o r f i s m o c u a n d o p u e d e c r i s t a l i z a r e nm á s d e u n t i p o d e o r d e n a m i e n t o .

 Empaquetamientos compactos de esferas. Las estructuras de muchos sólidos se pueden describir en

términos de esferas apiladas que representan átomos o iones. Especialmente útil es el estudio de las

formas en que se pueden apilar de la forma más compacta posible esferas de igual tamaño. En una capa,

las esferas se disponen de forma que cada esfera queda rodeada de otras seis esferas. La apilación regular

de estas capas, manteniendo la máxima compactación, se puede producir de dos maneras, dando lugar a

empaquetamientos compactos hexagonales o cúbicos (figura 4.5).

 Agujeros en empaquetamientos compactos. Una característica de las estructuras compactas es la exis-tencia de dos tipos de agujeros: octaédricos y tetraédricos (figura 4.6). Su importancia estriba en que mu-

chas estructuras, incluyendo las de algunos compuestos iónicos y aleaciones, se pueden describir como

formadas por empaquetamientos compactos en los que iones o átomos adicionales ocupan sus agujeros.

67 | Enlace químico y estructura de la materia Licenciatura en Química

Page 67: Enlace Quimico Y Estructura de Materia

7/22/2019 Enlace Quimico Y Estructura de Materia

http://slidepdf.com/reader/full/enlace-quimico-y-estructura-de-materia 67/89

Cúbica simple Cúbica centrada

en el cuer o

Cúbica centrada

en las caras

Tetragonalsimple

Tetragonal centradaen el cuerpo

Monoclínica

sim le

Monoclínica

centrada en las bases

Ortorrómbicasimple

Ortorrómbicacentrada en las bases

Ortorrómbica centradaen el cuerpo

Ortorrómbicacentrada en las caras Romboédrica

Triclínica Hexagonal

Figura 4.4. Los catorce tipos de redes cristalinas.

(a) Celdilla hexagonalA

B

A

BA

(b)A

B

A

BC

Celdilla cúbicacentrada en caras

Figura 4.5. Para construir una estructura compacta, se apila una segunda capa (B) sobre los huecos de la primera capa (A).La tercera capa se puede colocar en un empaquetamiento compacto de dos formas diferentes.

(a) Empaquetamiento hexagonal compacto. La tercera capa se coloca sobre los huecos de la segunda de forma que coincidasobre la primera. El resultado es una disposición de tipo ABAB..., cuya celdilla unidad es hexagonal.

(b) Empaquetamiento cúbico compacto. La tercera capa (C) se dispone sobre los huecos de la segunda que coinciden conhuecos de la primera. La cuarta capa coincide con la primera. El resultado es una disposición de tipo ABCABC..., cuya

celdilla unidad es cúbica centrada en las caras.

4.4 Bandas de orbitales moleculares

La teoría de orbitales moleculares que hemos aplicado a moléculas pequeñas puede ampliarse para expli-

car las propiedades de los sólidos. En un sólido, el solapamiento de un número muy grande de orbitales

próximos en energía forma virtualmente una banda continua que cubre un rango de energías (figura 4.7)Conductividad metálica. La conductividad eléctrica se debe al desplazamiento de cargas eléctricas a

través de un material y puede tener distintos orígenes. Así, las sustancias iónicas en estado líquido o en

disolución son conductores iónicos, ya que el transporte de cargas lo realizan los iones, que se desplazan

Universidad de Alcalá Tema 4: Estructura de la materia | 68

Page 68: Enlace Quimico Y Estructura de Materia

7/22/2019 Enlace Quimico Y Estructura de Materia

http://slidepdf.com/reader/full/enlace-quimico-y-estructura-de-materia 68/89

(b)agujeros tetraédricos

(a)agujeros

octaédricos

Figura 4.6. (a) Los agujeros octaédricos estánentre seis esferas de dos capas de un empaque-tamiento compacto. Se señala con puntos ne-gros la localización de los agujeros octaédricosen la celdilla unidad de una estructura cúbicacentrada en las caras.Si cada esfera tiene un radio r, cada agujerooctaédrico puede alojar otro átomo de radioinferior a 0,414r. Si hay N átomos en el cristal,hay N agujeros octaédricos.(b) Los agujeros tetraédricos están entrecuatro esferas de dos capas de un empaque-tamiento compacto.Se señala la localización delos agujeros tetraédricos en la celdilla unidadde una estructura cúbica centrada en las caras.Si cada esfera tiene un radio r, cada agujerotetraédrico puede alojar otro átomo de radioinferior a 0,225r. Si hay N átomos en el cristal,

hay 2 N agujeros tetraédricos.

Figura 4.7. Los orbitales s de dos átomos iguales solapan alaproximarlos dando lugar a dos orbitales moleculares, unoenlazante y otro antienlazante. Si situamos en una línea unnúmero N de átomos iguales, sus  N orbitales s solaparán dandolugar a  N  orbitales moleculares. Un de ellos no tendrá ningúnnodo entre átomos vecinos, será el orbital más enlazante y, portanto, el de menor energía. Otro tendrá nodos entre todos losátomos vecinos, será el orbital más antienlazante y, por tanto, elde mayor energía. El resto tendrá un número de nodos y una

energía intermedias. La diferencia de energía entre los orbitalesextremos es siempre finita aunque N tienda a infinito y dependefundamentalmente de la fortaleza de la interacción entre átomosvecinos. Eso significa que la diferencia de energía entre losorbitales tiende a cero al crecer N , de forma que en la práctica sepuede considerar que forman una banda continua de niveles deenergía. La banda que acabamos de describir está formada pororbitales s y se llama banda s. También se pueden formarbandas p o d .

2 Orbitales Atómicos dan 2 Orbitales Moleculares

 N Orbitales Atómicos dan N Orbitales MolecularesSi N es grande, los O.M. están muy próximos

formando una banda

A A—A A

 N A A N 

Banda de O.M.

Orbital más antienlazante

Orbital más enlazante

con libertad. En los sólidos, la conductividad eléctrica es casi siempre debida al movimiento de

electrones y se llama conductividad electrónica o también conductividad metálica, ya que la presentan

fundamentalmente (pero no únicamente) los metales. Respecto de la conducción electrónica, un material

puede ser conductor metálico, semiconductor, superconductor o aislante. La conductividad electrónica

de un sólido se explica adecuadamente en términos de bandas de orbitales moleculares.

Conductores metálicos. Los conductores se caracterizan por tener bandas parcialmente llenas (figura

4.8), por las que los electrones se mueven con relativa libertad. Si se aplica por ejemplo un campo

eléctrico, se moverán preferentemente en el sentido del campo, produciéndose la conducción. La conduc-

(b) Banda llena:aislante

a) Banda semillena:onducción metálica

Na(átomo)

Na(metal)

3s

Agujero

Figura 4.8. (a) Un conductor metálico, por ejemplo el sodio, secaracteriza por tener una banda no completamente llena. Loselectrones de los niveles superiores pueden ser fácilmente

promocionados a los niveles vacíos cercanos y, como resultado,pueden moverse relativamente con libertad a través del sólido.(b) Si una banda está completamente llena y hay un agujero deenergía considerable antes del siguiente orbital vacío disponible,la sustancia es un aislante.

69 | Enlace químico y estructura de la materia Licenciatura en Química

Page 69: Enlace Quimico Y Estructura de Materia

7/22/2019 Enlace Quimico Y Estructura de Materia

http://slidepdf.com/reader/full/enlace-quimico-y-estructura-de-materia 69/89

tividad de algunos metales se debe a la superposición entre bandas de orbitales moleculares (figura 4.9).

a

Mgátomo

Mgmetal

s

Agujero

 p

(b)

s

 p

Figura 4.9. (a) El agujero entre las bandas s y p de un sólidodepende de la separación de energía entre los orbitales s y p y dela fortaleza de la interacción entre los átomos (las bandas son

más anchas cuando la interacción es fuerte). (b) A veces, lasbandas pueden superponerse. Este es el caso de metales como elmagnesio, lo que justifica su conductividad eléctrica.

 Aislantes. Si la banda de valencia está completamente llena y hay un considerable agujero hasta la banda

vacía, la sustancia es un aislante. En la figura 4.10, se muestra el diagrama de bandas de un aislante: un

ejemplo de sólido iónico.

(b)(a)

Na Cl

3s

3 p z

Na+Cl –

Gran

diferenciade ener ía

Agujerogrande

NaCl(s)

3s (Na+)

3 p (Cl –)

Figura 4.10. (a) Orbitales moleculares del par Na+Cl –. La electrone-gatividad del Cl es mucho mayor que la del Na, por lo que la diferen-cia de energía entre el orbital 3s del Na y el 3 p

 zdel Cl es muy grande

y la interacción pequeña. El resultado es coherente con un modelo

iónico. (b) En un cristal de NaCl, los orbitales 3 p llenos de los Cl – ylos 3s vacíos de los Na+ forman sendas bandas. Las bandas sonestrechas, pues el solapamiento entre iones iguales es pequeño. Elagujero entre bandas es grande y el NaCl es un buen aislante.

Semiconductores. Un semiconductor tiene un agujero entre bandas pequeño, de forma que, a T > 0 K, la

agitación térmica de algunos electrones puede ser suficiente como para que salten a la banda vacía,

permitiendo la conductividad (figura 4.11).

Agujeropequeño

(a)Ocupación

T = 0 K

      E    n    e    r    g      í    a

0 100% (b)

      E    n    e    r    g      í    a

Ocupación

T > 0 K

0 100% (c)

      E    n    e    r    g      í    a

Ocupación

T > 0 K

0 100%

Figura 4.11. (a) A T = 0 K, los electrones ocupan losorbitales de las bandas por orden de energía. (b) A T > 0 K,

la agitación térmica de los electrones hace que algunos deellos puedan ocupar niveles de energía mayores. (c) Unsemiconductor, a T = 0 K, tiene una banda llena y otravacía con un agujero de energía relativamente pequeño. AT > 0 K, algunos electrones saltan de la banda llena a lavacía, posibilitando la conducción.

La conductividad de los semiconductores es normalmente intermedia entre la de los aislantes y la de los

conductores metálicos. Sin embargo, el criterio para distinguir entre un conductor metálico y un

semiconductor es que la conductividad del semiconductor aumenta con la temperatura (figura 4.12).

108

104

1

10 –4

10 –8

   C  o  n   d  u  c   t   i  v   i   d  a   d   (   S  c  m  –   1   )

T (K)10001 10 100

Conductor metálico

Semiconductor

Superconductor

Figura 4.12. La conductividad de un semiconductor

aumenta fuertemente con la temperatura, mientras que la deun conductor metálico disminuye.En un semiconductor, al aumentar la temperatura

aumenta el número de electrones que pasan de la bandallena a la vacía, y, por tanto, su capacidad conductora.

En un conductor metálico, hay un efecto contrario másfuerte que el anterior. Para que un electrón pueda moversesuavemente, es necesario que los átomos estén dispuestoslo más uniformemente posible. Al aumentar la temperatura,la estructura cristalina pierde uniformidad y el electrónpierde movilidad, por lo que la conductividad disminuye.

Un superconductor (ver debajo) se caracteriza por

conducir a bajas temperaturas sin resistencia eléctrica(conductividad infinita).

Semiconductores intrínsecos y extrínsecos. Un semiconductor intrínseco es aquel que lo es por sí 

mismo. Las propiedades semiconductoras de un material se pueden mejorar sustituyendo (dopando)

Universidad de Alcalá Tema 4: Estructura de la materia | 70

Page 70: Enlace Quimico Y Estructura de Materia

7/22/2019 Enlace Quimico Y Estructura de Materia

http://slidepdf.com/reader/full/enlace-quimico-y-estructura-de-materia 70/89

Page 71: Enlace Quimico Y Estructura de Materia

7/22/2019 Enlace Quimico Y Estructura de Materia

http://slidepdf.com/reader/full/enlace-quimico-y-estructura-de-materia 71/89

direccional. Por ello, los átomos de un metal tienden a rodearse del mayor número posible de otros

átomos, cristalizando a menudo en empaquetamientos compactos (número de coordinación = 12) o en

otras estructuras de altos números de coordinación (figura 4.15, y tabla 4.5). El número de coordinación

de una red es el número de vecinos más próximos que tiene un átomo o ion dado. Los planos de algunos

empaquetamientos compactos de metales no se apilan regularmente (ABAB... o ABCABC...) sino de

formas más complejas (p. ej. ABACBABABC...).Tabla 4.5. Tipos de red cristalina habituales en los metales

Tipo de red cristalina Número de Coordinaciónhexagonal (empaquetamiento hexagonal compacto, ABAB...) 12

cúbica centrada en las caras (empaquetamiento cúbico compacto, ABCABC...) 12cúbica centrada en el cuerpo 8

cúbica simple (muy poco usual) 6

Li13,094hcp<72ccc<PE

Be4,913

hcp<1250ccc<PE

Li Elemento13,094 Volumen molar (cm3 mol–1), a 298,15 Khcp<72 Rango de temperatura (K) de estabil idadccc<PE de las formas alotrópicas de cada metal

Na23,932hcp<36

ccc<PE

Mg14,082

hcp

Al10,061

ccp

K45,646

ccc

Ca26,439ccp<510ccc<PE

Sc15,134

hcp<1607ccc<PE

Ti10,697

hcp<1173ccc<PE

V8,377

ccc

Cr7,274

ccc

Mn7,42

cub<1368ccp<1408ccc<PE

Fe7,137

ccc<1183ccp<1663ccc<PE

Co6,712

hcp<661ccp<PE

Ni6,631

ccp

Cu7,156

ccp

Zn9,219

hcp

Ga11,865

orto

Rb56,200

ccc

Sr34,07

ccp<486hcp<878ccc<PE

Y20,017

hcp<1733ccc<PE

Zr14,106

hcp<1138ccc<PE

Nb10,895

ccc

Mo9,443

ccc

Tc8,644

hcp

Ru8,222

hcp

Rh8,334

ccp

Pd8,918

ccp

Ag10,335

ccp

Cd13,078

hcp

In15,851

fct

Sn16,391

diam<291tcc<PE

Cs

70,168ccc

Ba

38,399ccc

La-Lu Hf

13,526hcp<2268ccc<PE

Ta

10,919ccc

W

9,601ccc

Re

8,916hcp

Os

8,476hcp

Ir

8,572ccp

Pt

9,148ccp

Au

10,277ccp

Hg

14,152tcc<79romb<PE

Tl

17,322hcp<503ccc<PE

Pb

18,377ccp

Bi

21,442romb

Fr Ra41,337

ccc

Ac-Lr

La22,74

hcpd<613ccp<1141

ccc<PE

Ce20,83

ccp<95hcpd<263ccp<1003ccc<PE

Pr20,93

h c p d < 1 0 9 4ccc<PE

Nd20,71

h c p d < 1 1 3 5ccc<PE

Pm20,361

hcpd

Sm20,12

h c p d < 1 1 9 0ccc<PE

Eu29,160

ccc

Gd20,027

hcp<1535ccc<PE

Tb19,43

orto<220hcp<1589

ccc<PE

Dy19,123

hcp<1243ccp<PE

Ho18,869

hcp<1239ccc<PE

Er18,565

hcp<1190ccc<PE

Tm18,183

hcp<1277ccc<PE

Yb25,00

hcp<270ccp<1005ccc<PE

Lu17,891

hcp<1673ccc<PE

Ac22,694

ccp

Th19,922

ccp<1673ccc<PE

Pa15,278

tcc<1443ccc<PE

U12,572

orto<935tetr<1045ccc<PE

Np11,65

orto<553tetr<850ccc<PE

Pu12,12

mon<395

Am17,74

hcpd<923ccp<1350ccc<PE

Cm18,17

h c p d < 1 5 5 0ccp<PE

Bk 16,95

h c p d < 1 2 0 0ccp<PE

Cf16,50

hcpd<873ccp<998ccp<PE

Es

ccp

Fm Md No Lr

Empaquetamientos compactosccp = cúbico compacto (centrado en caras, ABC…)hcp = hexagonal compacto (ABAB…)hcpd = hexagonal compacto doble (ABAC…)

Otros empaquetamientostcc = tetragonal centrado en el cuerpotetr = tetragonal complejoorto = ortorrómbico centrado en las carasmon =: monoclínicoromb = romboédrico

Empaquetamientos cúbicos no compactosccc = cúbico centrado en el cuerpocub = cúbico simple

Figura 4.15. Estructuras cristalinas de los metales. Los colores indican el tipo de estructuracristalina del metal en su forma alotrópica estable a 298,15 K y 0,1 MPa.

Muchos metales presentan polimorfismo, es decir, existen bajo diferentes formas cristalinas, que setransforman unas en otras al cambiar la presión o la temperatura. En muchos casos las estructuras son

compactas a bajas temperaturas, pero se transforman en no compactas, generalmente cúbicas centradas

en el cuerpo, al subir la temperatura debido al incremento de las vibraciones de los átomos.

Universidad de Alcalá Tema 4: Estructura de la materia | 72

Page 72: Enlace Quimico Y Estructura de Materia

7/22/2019 Enlace Quimico Y Estructura de Materia

http://slidepdf.com/reader/full/enlace-quimico-y-estructura-de-materia 72/89

α  –Fecúbica centrada

en el cuerpo

906 °Cγ  –Fe

cúbica centradaen las caras

1401 °Cα  –Fe

cúbica centradaen el cuerpo

1530 °CFe(líquido)

A altas presionesβ  –Fe

hexagonal compact

4.6 Sólidos iónicos: enlace y estructura

 Enlace. Un modelo simple, aunque aproximado, de un sólido iónico es el de una disposición infinita de

cationes y aniones, que se mantienen unidos por fuerzas electrostáticas. En un modelo iónico puro, sesupone que los iones son esferas que no se deforman en presencia de otros iones de signo contrario.

Tipos de estructura. El enlace iónico es poco direccional, lo que explica que cada ion tienda a rodearse

del máximo número posible de contraiones, formando redes tridimensionales con un elevado número de

cationes y de aniones fuertemente unidos entre sí. Por tanto, los números de coordinación son altos (alre-

dedor de 6) y las estructuras compactas, aunque menos que las metálicas, debido a que el distinto tamaño

y la carga eléctrica de anión y catión limitan su empaquetamiento (tabla 4.6 y figura 4.16).

Tabla 4.6. Tipos de estructura y relación r + /r  – óptima para cada una de ellas

Tipo de estructura Número de Coordinación* N  + /N  – ** r < /r > óptimo*** FiguraBlenda de cinc (ZnS) (4,4) 1:1 0,22–0,41 4.14(c)Wurtzita (ZnS) (4,4) 1:1 0,22–0,41 4.14(d)Cloruro de sodio (NaCl) (6,6) 1:1 0,41–0,73 4.14(a)

Cloruro de cesio (CsCl) (8,8) 1:1 0,73–1,00 4.14(b)Rutilo (TiO2) (6,3) 1:2 0,41–0,73 4.14(f)

Fluorita (CaF2) (8,4) 1:2 0,73–1,00 4.14(e)

Antifluorita (ejemplo: K2O) (4,8) 2:1 0,73–1,00

* (Número de coordinación del catión, número de coordinación del anión).El número de coordinación de un ion es el número de iones de carga opuesta que tiene más próximos.

** Número de cationes/número de aniones*** Relación óptima entre el radio del ion más pequeño y el del más grande, calculada geométricamente. Habitualmente, el

radio mayor es el del anión, por lo que la relación anterior se convierte en r+ /r – .

Cl Cs(b)

(a) Cl Na

(f) O Ti

(e) F Ca

(d) ZnS

(c) ZnS

Figura 4.16. (a) La estructura tipo cloruro de sodio puede verse como unempaquetamiento cúbico compacto de voluminosos aniones Cl –, en elque los cationes Na+ ocupan los N agujeros octaédricos. Cada ion estárodeado de 6 contraiones (iones de carga opuesta). En una celdilla unidadhay 4 iones Na+ y 4 iones Cl – (tener en cuenta que un ion de un vértice secomparte entre 8 celdillas unidad, uno de un lado entre 4 y uno de unacara entre 2).(b) La estructura tipo cloruro de cesio tiene una celdilla unidad cúbica enla que un anión Cl – ocupa cada vértice y un catión de Cs+ el centro de la

celdilla (o viceversa). Cada ion está rodeado de 8 contraiones. En unaceldilla unidad hay 1 anión Cl – y un catión Cs+.(c) La estructura tipo blenda de cinc (ZnS) está basada en unempaquetamiento cúbico compacto de aniones S2–, en el que los cationesZn2+ ocupan la mitad de los 2 N agujeros tetraédricos. Cada ion estárodeado de 4 contraiones.(d) La estructura de la Wurtzita, otra forma polimórfica del ZnS, estábasada en un empaquetamiento hexagonal compacto de aniones. Loscationes ocupan la mitad de los 2 N agujeros tetraédricos. Cada ion estárodeado de 4 contraiones.(e) La fluorita (CaF2) está basada en un empaquetamiento cúbicocompacto de cationes, en el que los aniones ocupan todos los 2 N agujerostetraédricos (hay dos veces más de aniones que de cationes). Cada catión

está rodeado de 4 aniones y cada anión de 8 cationes. La estructuraantifluorita es idéntica a la fluorita, pero intercambiando cationes yaniones (Ejemplo, K2O).(f) El rutilo está basado en un empaquetamiento hexagonal compacto deaniones en el que los cationes ocupan la mitad de los  N agujerosoctaédricos (el número de cationes es la mitad del de aniones).

73 | Enlace químico y estructura de la materia Licenciatura en Química

Page 73: Enlace Quimico Y Estructura de Materia

7/22/2019 Enlace Quimico Y Estructura de Materia

http://slidepdf.com/reader/full/enlace-quimico-y-estructura-de-materia 73/89

 Radios iónicos. Cuando un sólido está formado por un sólo elemento, se toma la mitad de la separación

internuclear entre átomos adyacentes como una medida de radio. Sin embargo, en un sólido iónico, los

átomos adyacentes son diferentes, lo que plantea el problema de repartir la separación entre los iones

individuales. La forma más directa de resolver el problema es hace una suposición sobre el radio de un

ion y usar dicho valor para recopilar un conjunto de valores autoconsistentes para otros iones. El anión

O2– presenta la ventaja de encontrarse en combinación con un número elevado de elementos y de serrazonablemente duro (poco polarizable, su tamaño no varía mucho al cambiar el catión). Muchas escalas

de radios iónicos están basadas en r(O2–) = 1,40 Å.

 Racionalización de las estructuras. La estructura que adopta un sólido iónico puede racionalizarse si se

considera la obligación de mantener la electroneutralidad del sólido y la tendencia a maximizar las

atracciones entre iones de carga contraria y minimizar las repulsiones entre iones de la misma carga. El

primer requisito restringe las estructuras posibles para un compuesto iónico a aquellas que tengan la rela-

ción N +/ N  – adecuada (tabla 4.6). El segundo requisito sugiere que la estructura más adecuada es aquella

cuyo número de coordinación es el adecuado para la relación entre el radio del catión y del anión (figura

4.17 y tabla 4.6) De esta forma, se puede predecir la estructura de muchas sustancias. Sin embargo, exis-ten numerosas excepciones, que mayormente se pueden justificar por alejamientos del modelo iónico.

número de coordinación = 6relación de radios = 0,414

número de coordinación =relación de radios > 0,414

número de coordinación = 6relación de radios < 0,414

No favorableFigura 4.17. Un ion en un entorno octaédrico (N.C. = 6) está rodeado de 6 contraiones (por claridad, en el dibujo sólo se

muestran los 4 situados en un plano). Cuando la relación de radios es 0,414, el ion cabe justo en el agujero. Cuando esmenor, los iones de carga opuesta no están en contacto y los de carga igual se tocan: de acuerdo a argumentos

electrostáticos simples, el número de coordinación favorable es menor de 6. El número de coordinación de 6 es favorableentre 0,414 y 0,732 (para esta última relación de radios, es ya posible situar 8 contraiones alrededor del ion considerado).

4.7 La formación de sólidos iónicos: entalpía de red

 Entalpía de red. La estabilidad de una red iónica procede principalmente de la fuerte atracción entre

iones de carga opuesta y es medida por la entalpía de red  (∆ H U), que se define como la entalpía

correspondiente al proceso de ruptura de la red en los iones gaseosos, p. ej., KCl( s) → K+(g) + Cl –(g).

Ciclo de Born–Haber. La entalpía de formación (∆ H f ) de una sustancia es la variación de entalpía

producida en el proceso de formación de dicha sustancia a partir de sus elementos en su forma estable encondiciones normales (25 °C y 1 atm). El proceso de formación del cloruro de potasio es K(s) + 1/2

Cl2(g) → KCl(s). El ciclo de Born–Haber relaciona la formación de un sólido iónico como el cloruro de

potasio con otros procesos más elementales (figura 4.18).

El ciclo de Born–Haber permite calcular la entalpía de red o cualquier otra de las entalpías

participantes, a partir del resto (tabla 4.7). En el ejemplo de la figura 4.18, se cumple que:

∆ H f (KCl(s)) = ∆ H S(K) + 1/2 ∆ H D(Cl2(g)) + ∆ H EA(Cl) + ∆ H I(K) – ∆ H U(KCl)

Tabla 4.7. Entalpías de red a 25°C en kilojulios por mol  Haluros del grupo 1 y 11 del grupo 2 Óxidos Sulfuros

LiF 1046 LiCl 861 LiBr 818 LiI 759 BeCl2 3017 Li2O 2799 BeO 4443NaF 929 NaCl 787 NaBr 751 NaI 700 MgCl2 2524 Na2O 2481 MgO 3850 MgS 3406

KF 826 KCl 717 KBr 689 KI 645 CaCl2 2255 K2O 2238 CaO 3461 CaS 3119

AgF 971 AgCl 916 AgBr 903 AgI 887 SrCl2 2153 SrO 3283 SrS 2974

Universidad de Alcalá Tema 4: Estructura de la materia | 74

Page 74: Enlace Quimico Y Estructura de Materia

7/22/2019 Enlace Quimico Y Estructura de Materia

http://slidepdf.com/reader/full/enlace-quimico-y-estructura-de-materia 74/89

K(g) + Cl(g)

K+(g) + Cl –(g)

KCl(s)

 – ∆ H U = –717 kJ/mol

K(s) + 1/2 Cl2(g)

∆ H f = –437 kJ/mol

① ∆ H S(K)+ 1/2 ∆ H D(Cl2) = 89 +122 = 211 kJ/mol

➁ ∆ H I(K)+ ∆ H EA(Cl) = 418 + (–349) = 69 kJ/mol

➂211 kJ/mol

69 kJ/mol

Figura 4.18. El ciclo de Born–Haber para la formación de KClsólido a partir de potasio y cloro. La suma de las entalpías por elcamino largo (flechas huecas) es igual a la suma de las entalpíaspor el camino directo (flechas rellenas).

∆ H S = entalpía de sublimación∆ H D = entalpía de disociación∆ H I = entalpía de ionización∆ H EA = entalpía de electroafinidad∆ H U = entalpía de red∆ H f = entalpía de formación

① Formación de átomos gaseosos a partir de los elementos➁ Formación de los iones gaseosos a partir de los átomos gaseosos➂ Formación de la red iónica a partir de los iones gaseosos.

 Ecuación de Born–Landé. La ecuación de Born–Landé estima teóricamente la entalpía de red esperada

para un compuesto iónico, a partir de un modelo de enlace puramente iónico (recuadro 4.1).

 

∆ H estimadaN z ez e

d nU 

 A( ) = − −

+ −1

41

1

0 0πε 

 M  z+, z – = Cargas iónicas  M = Constante de Madelungn = Factor de Landé d 0 = Distancia equilibrio catión-anión

 N A = Constante de Avogadro e =Carga elemental (1,602 10 –19 C )

La constante de Madelung ( M ) es un índice corrector, calculado geométricamente (recuadro 4.2), que

toma en cuenta que cada ion en una red tiene cerca a más de un ion. Su valor depende del tipo de red

(tabla 4.8).Tabla 4.8. Valores de la constante de Madelung para algunos tipos de red iónica

Tipo de red  Blenda de cinc (ZnS) Cloruro de sodio Cloruro de cesio Fluorita (CaF2) Rutilo (TiO2) MM  M  M  1,63806 1,74756 1,76267 2,51939 2,408

El  factor de Landé  (n) refleja las repulsiones entre las esferas electrónicas de los iones, y puede ser

estimado a partir de su configuración electrónica externa (tabla 4.9). Para una red que contiene iones de

diferente configuración, se toma la media de los valores de sus iones. Así, para NaCl n = (7 + 9)/2 = 8.Tabla 4.9. Valores del factor de Landé para algunos ionesConfiguración He Ne Ar, Cu+ Kr, Ag+ Xe, Au+

n 5 7 9 10 12 Ejemplos Li+ F –, O2–, Na+ K+, Cl –, Ca2+ Br –, Rb+ I –, Cs+, Ba2+

Los valores de entalpía de red estimados mediante la ecuación de Born–Landé para sólidos muy iónicos,

concuerdan bastante bien con los valores experimentales del ciclo de Born–Haber. De hecho, podemosconsiderar el acuerdo entre la entalpía de red experimental y la teórica como una medida de la validez del

modelo iónico para una sustancia. Los dihaluros de la tabla 4.10 presentan valores de entalpía de red que

difieren significativamente de los calculados mediante la ecuación de Born–Landé, pues se ajustan peor a

un modelo iónico puro (cationes más polarizantes).Tabla 4.10. Comparación de las entalpías de red experimentales y teóricas de haluros, en kJ/mol Compuesto LiF NaF KF LiI NaI KI MgBr2 MgI2 MnBr2 MnI2

 Entalpía de red experimental  1046 929 826 759 700 645 2395 2315 2441 2382 Entalpía de red teórica 1032 901 790 709 672 622 2140 1985 2177 2010 Diferencia, en porcentaje 1,3% 3,0% 4,3% 6,6% 4,0% 3,6% 11% 14% 15% 16%

 Entalpía de red y puntos de fusión. En sólidos puramente iónicos, se esperaría que los puntos de fusiónaumentaran en el mismo sentido que la entalpía de red, es decir, al aumentar la carga de los iones y al

disminuir su tamaño. Aunque éste es el comportamiento observado en algunos casos (tabla 4.11), la

apreciable participación de enlace covalente en muchos compuestos iónicos hace que estos resultados no

75 | Enlace químico y estructura de la materia Licenciatura en Química

Page 75: Enlace Quimico Y Estructura de Materia

7/22/2019 Enlace Quimico Y Estructura de Materia

http://slidepdf.com/reader/full/enlace-quimico-y-estructura-de-materia 75/89

Recuadro 4.1. Deducción de la ecuación de Born-Landé.

En un modelo puramente iónico, se supone que un sólido iónico está formado por esferas cargadas indeformables. En talcaso la energía de atracción entre dos esferas de distinta carga viene dada por la expresión:

 E 

z ez e

d =+ −1

4 0πε  z –  z+

 z+, z – = Cargas iónicas (catión, anión)

ε 0 = permitividad del vacío (8,854 10 –12 C 2 m –1  J  –1 )

d 0 = distancia entre catión y anión

e = unidad elemental de carga (1,602 10 –19 C )

En un sólido iónico, hay más interacciones que en un par iónico. Así, un catión sodio en el cloruro sodio sufre laatracción de los seis cloruros que le rodean inmediatamente, pero las repulsiones de 12 cationes sodio situados máslejos, etc. (ver 4.6). La suma de todas esas atracciones-repulsiones se puede calcular geométricamente (recuadro 4.2) ysu resultado es la constante de Madelung ( M ).

 E  z ez e

d = + −1

4 0πε 

 M 

Si sólo existiera la atracción entre los iones, éstos se aproximarían infinitamente hasta fundirse. Si ésto no es así esporque la repulsión existente entre sus cortezas electrónicas acaba por detener la aproximación a una distancia en la quela energía se hace mínima. La energía de repulsión se puede expresar como

 E  B

d  R n

= 14 0πε 

donde B es una constante y n es el factor de Landé, que refleja la resistencia a acercarse estrechamente un ion al otro. Laenergía total queda como:

 E  z ez e

 B

 z ez e

 B

d n n= + = +

+ − + −1

41

41

40 0 0πε πε πε  

 M M (1)

En el equilibrio, los iones se colocan a una distancia d 0 en la que la energía se hace mínima:

dE 

dd 

 z ez e

nB

d n

= = − −

+ −+

01

4 0 02

01πε 

 M (2)

lo que físicamente equivale a decir que la fuerza de atracción y repulsión se hacen iguales. Despejando B en (2)

 B  z ez ed 

n

n= − + − − M  0 1

y sustituyendo en (1) ( E 0 es la energía en el equilibrio)

 E  z ez e

 z ez e

nd 

 B

 z ez e

d nn00 0 0 0 0 0

14

14

14

11

= −

= = −

+ − + − + −

πε πε πε  

 M M M 

Para expresar el resultado en forma de entalpía de red hay que cambiar el signo de la expresión y multiplicarla por elnúmero de Avogadro para contener 1 mol de sustancia:

∆ H estimada N z ez e

d nU 

 A( ) = − −

+ −1

41

1

0 0πε 

 M 

Se han propuesto otras ecuaciones que emplean funciones distintas de la aquí empleada para la modelizar la energía de

repulsión o que consideran otros factores como las atracciones de Van der Waals.Recuadro 4.2. Cálculo de la constante de Madelung

Calculemos la constante de Madelung para una red ficticia de una sola dimensión formada por igual número de cationesy aniones alternados y separados por la misma distancia d . La energía de atracción entre los iones de un par iónico es:

 E  z ez e

d = + −1

4 0πε  z –  z+

 z –  z+  z –  z+  z –  z+ z –  z+  z –  z+

Pero en la red cada ion es atraído por dos iones de carga contraria situados a la distancia d , repulsado por dos iones de lamisma carga situados a una distancia 2d , pero a la vez atraído por dos iones de carga contraria situados a 4d ... La energíade atracción culómbica es la suma de todas contribuciones:

 E 

 z ez e

 z ez e

 z ez e

 z ez e

 z ez e

d = − + − + = × × − + − +

+ − + − + − + − + −1

4

2 1

4

2

2

1

4

2

3

1

4

2

4

1

4 2 1

1

2

1

3

1

40 0 0 0 0πε πε πε πε πε  ... ...La suma de la serie entre paréntesis converge en ln 2, por lo que la energía de atracción en la red es 2 ln 2 = 1,386 vecesla del par iónico. Este es el valor de la constante de Madelung que, obsérvese, depende sólo de la geometría de la red.

 E  z ez e

d = ×+ −1

42

1 3860πε 

,

Universidad de Alcalá Tema 4: Estructura de la materia | 76

Page 76: Enlace Quimico Y Estructura de Materia

7/22/2019 Enlace Quimico Y Estructura de Materia

http://slidepdf.com/reader/full/enlace-quimico-y-estructura-de-materia 76/89

Tabla 4.11. Variación del radio del haluro y el punto de fusión para los haluros de sodioSustancia NaF NaCl NaBr NaI

 Radio del anión ( Å ) 1,36 1,81 1,95 2,16

 Punto de fusión ( °C ) 990 801 755 651

Tabla 4.12. Puntos de fusión para algunos halurosSustancia BeCl

2

CaCl2

HgCl2

NaBr MgBr2

AlBr3

LiBr NaBr KBr

 Punto de fusión ( °C ) 405 772 276 755 700 975 547 755 730

sean generalizables, tal como se observa en algunas de las series de puntos de fusión de la tabla 4.12, que

difieren de lo esperado para compuestos iónicos puros.

 Entalpía de red y solubilidad. Muchos compuestos iónicos son solubles en disolventes polares como el

agua. La entalpía de disolución (∆ H d) es la entalpía del proceso de disolución de un sólido iónico. Para el

cloruro de litio es la entalpía del proceso LiCl(s) → Li+(aq) + Cl –(aq), donde Li+(aq) y Cl –(aq) son los

iones hidratados. Se llama hidratación (para el agua) o solvatación (en general) al proceso por el cual un

ion se rodea de moléculas de disolvente, de forma que se produce una atracción mutua. Se llama entalpía

de hidratación (∆ H H) de un ion a la entalpía de este proceso, que para el catión litio es Li +(g) → Li+(aq).El proceso de disolución en agua de un sólido iónico se puede descomponer en dos etapas (figura

4.19). Para el LiCl, la entalpía de disolución se relaciona con las de hidratación y de red mediante la

ecuación ∆ H d(LiCl) = ∆ H H(Li+) + ∆ H H(Cl –) + ∆ H U(LiCl).

➁①∆ H H(Li+) + ∆ H H(Cl –) =

(–558) + (–340) =

 –898 kJ/mol

∆ H d = –37 kJ/mol

LiCl( s)

∆ H U =861 kJ/mol

Li+(aq) + Cl  –(aq)

Li+(g) + Cl –(g)

Figura 4.19. El ciclo de termoquímico para la disolución delLiCl sólido. Los compuestos iónicos se disuelven debido a quesus iones se hidratan.

∆ H H = entalpía de hidratación

∆ H U = entalpía de red∆ H d = entalpía de disolución① Ruptura de la red iónica para producir los iones gaseosos➁Hidratación de los iones gaseosos

El aumento de la carga y la disminución del radio iónico favorecen la hidratación de los iones (tabla

4.13), pero dificultan la ruptura de la red. Este efecto contradictorio hace difícil predecir la variación de

la entalpía de disolución y de la solubilidad de las sales iónicas (tabla 4.14).Tabla 4.13. Entalpías de hidratación en kilojulios por mol de algunos iones

 Ion Li+ Na+ K+ Cl– Br– I– Li+ Be2+ Al3+

 Entalpía de hidratación  –558 –444 –361 –340 –309 –296 –558 –1435 –2537

Tabla 4.14. Variación de la solubilidad de algunos haluros alcalinosCompuesto LiF LiCl LiBr NaCl KCl RbCl

 Entalpía de hidratación en kJ/mol   –1041 –898 –867 –783 –701 –675

 Entalpía de red en kJ/mol  1046 861 818 787 717 692

 Entalpía de disolución en kJ/mol  5 –37 –49 4 16 17

Solubilidad (mol/l) 0,06 14 10 5,4 4,2 6,0

 Estabilidad térmica de las combinaciones de cationes grandes – aniones grandes. Se ha observado que

los cationes grandes estabilizan a los aniones grandes (y viceversa). Por ejemplo, los carbonatos metáli-

cos son más estables térmicamente, frente a su descomposición en el óxido del metal y CO2, cuando el

catión es grande. La tabla 4.15 refleja el aumento de la temperatura y de la energía libre de la descom-posición con el tamaño del catión para varios carbonatos de metales del grupo 2. Este comportamiento no

se debe a la entropía de descomposición, que es muy parecida para todos los carbonatos ya que está

dominada por la formación de CO2(g), sino al aumento drástico de la entalpía de descomposición.

77 | Enlace químico y estructura de la materia Licenciatura en Química

Page 77: Enlace Quimico Y Estructura de Materia

7/22/2019 Enlace Quimico Y Estructura de Materia

http://slidepdf.com/reader/full/enlace-quimico-y-estructura-de-materia 77/89

Tabla 4.15. Datos termodinámicos para la reacción MCO 3(s) MO(s) + CO 2(g) a 25 °C

 M  Mg Ca Sr Ba

∆∆∆∆ H° (kJ mol  –1 ) 100,6 178,3 234,6 269,3

∆∆∆∆S° (J K  –1 mol  –1 ) 175,0 160,6 171,0 172,1

∆∆∆∆G° (kJ mol  –1 )* 48,3 178,3 234,6 269,3

Temp. descomposición (° C) 400 900 1290 1360

* ∆G° = ∆ H°  – T ∆S° . Los valores dados están calculados a 298 K (25 °C).

La explicación se encuentra en las entalpías de red de carbonato y óxido:

MCO3(s)∆ H° 

MO(s) + CO2(g)

M2–(g) + CO32–(g)

∆ H° U (MCO3)

M2–(g) + O2–(g) + CO2(g)

 –∆ H° U (MO)

∆ H° d (CO32–)

∆ H° = ∆ H° d (CO32–) + ∆ H° U (MCO3) – ∆ H° U (MO)

La entalpía de red de un óxido es mayor que la del carbonato correspondiente ya que la distancia

interiónica es más pequeña, y esto favorece la descomposición del carbonato para cualquier catión. Ahora

bien, si el catión es grande, la diferencia en la distancia interiónica y en la energía de red es relativamente

pequeña (figura 4.20), y la descomposición es menos favorable termodinámicamente.

CO32–

O2–

Pequeño %de cambio

CO32–

O2–

Gran %de cambio

Figura 4.20. Una muy exagerada representación de ladisminución en la distancia interiónica al descomponer unanión (por ejemplo, cuando CO3

2– descompone en O2–).

Cuando el catión es grande, la distancia y, por tanto, laenergía de red cambian relativamente poco. Cuando elcatión es pequeño, el cambio relativo en la distancia y laenergía de red es grande, lo que favorecetermodinámicamente la descomposición.

La diferencia entre la energía de red del óxido y la del carbonato aumenta con la carga del catión, lo que

explica que los carbonatos del grupo 2 sean menos estables que los del grupo 1.

4.8 Sólidos moleculares

Los sólidos moleculares están formados por moléculas covalentes discretas (p.ej. H2O, I2, etc) o por

átomos (p. ej., los gases nobles en estado sólido) unidos mediante fuerzas de Van de Waals.4.9 Sólidos covalentes

Los sólidos covalentes están formados por redes de átomos unidos por enlace covalente. Los números de

coordinación son bajos (en muchos casos 4), al estár limitados por el número de enlaces covalentes que

puede formar un átomo dado. Las redes covalentes pueden ser tridimensionales, bidimensionales (lámi-

nas) ó monodimensionales (cadenas), aunque sólo las primeras componen sólidos puramente covalentes.

Los sólidos covalentes tridimensionales tienen elevados puntos de fusión y ebullición por las fuerzas

extremadamente fuertes que los unen. En los bi– y monodimensionales, las láminas ó cadenas se atraen

por fuerzas débiles similares a las fuerzas de Van der Waals descritas para las moléculas (figura 4.21).

Silicatos y fosfatos. Fosfatos y fundamentalmente silicatos son ejemplos de sustancias que forman

sólidos covalentes, iónicos o moleculares de estructuras variadas y complejas. Todos ellos están

constituidos por unidades XO4 que comparten entre sí uno o más átomos de oxígeno. Algunos ejemplos

se dan a en la figura 4.22.

Universidad de Alcalá Tema 4: Estructura de la materia | 78

Page 78: Enlace Quimico Y Estructura de Materia

7/22/2019 Enlace Quimico Y Estructura de Materia

http://slidepdf.com/reader/full/enlace-quimico-y-estructura-de-materia 78/89

(a) (b)

1,54 Å

1,42Å

3,40 Å

Figura 4.21. El carbono cristalino.(a) Estructura del diamante. En el diamante el númerode coordinación del carbono es 4. Cada átomo estárodeado tetraédricamente por cuatro átomos equidis-tantes. La longitud de los enlaces C–C es 1,54 Å.(b) Estructura del grafito. Es la estructura más establedel carbono. Dentro de cada capa los enlaces C–C son

fuertes y la longitud de enlace es 1,42 Å. Los enlacesentre los carbonos de capas distintas son débiles y ladistancia es de 3,40 Å.

OP

O

PO

P

HO O

OHHO

O O

Si

Si

Si

Si

Si

SiO

O

O O

O

O

H3P3O9(anión cíclico)

Be3Al2(Si6O18)Mineral Berilo(anión cíclico)

Si O Si SiOSiO O Si O Si

Si

SiO

O

OSi

Si SiO

O O

O

Si

Si

Si

O

O

O

OSi

SiO

O

OSi

Si

Si

SiO

O

O

O

O

Si

Si

SiSi

O

O

O O

OSi

Si

SiSi

Si

SiO

O

O O

O

OPP

O O

O

O OO O

O

O

P

P

O

Si

Si Si

Si

O

O O

OO

Si

Si Si

Si

O

O O

OO

Si

Si

Si Si

Si

Si

O

O

O

O

OO

[Ca2Mg5(Si4O11)2(OH)2] xasbesto

(red monodimensional)

(Si2O52–) x

talco(red bidimensional)

P4O10(ciclo)

HO P

O

OH

OHPO

O

OH

HO Si

OH

OH

OHSiO

OH

OH

H6Si2O7 H6Si2O7

a)

b)

c)

LiAl(SiO3)2Mineral espodumena

(cadenas)Figura 4.22. a) Por comparto de

un vértice, se forman moléculas oiones dímeros.b) Al compartirse dos vértices, sepueden formar estructurasinfinitas en cadenas. Otraposibilidad es la formación deciclos, dando lugar a moléculasdiscretas. (En el anión Si6O18

12–,no se han dibujado los dosoxígenos terminales situadossobre cada silicio).c) Cuando se comparten tres

vértices, se pueden formarmoléculas discretas (P4O10),cadenas dobles (asbesto) o capas(talco). (En el asbesto y el talco,no se han dibujado los oxígenosterminales que hay sobre cadaátomo de silicio).d) Por compartimiento de cuatrovértices, se forman redescovalentes tridimensionales (p. ej.el cuarzo, SiO2, no dibujado)

4.10 Comparación entre las propiedades de los distintos tipos de sólidos

Tipo de sólido Iónico Metálico Covalente 3D Molecular  Unidad estructural  Ion Átomo Átomo Molécula

 Enlace entre unidades Enlace iónico Enlace metálico Enlace covalente Fuerzas de Van der Waals

 Dureza Duro Amplia gama Duro Blando Punto de fusión Alto Amplia gama Alto Bajo

(600 a 3000 °C) (–39 a 3400 °C) (1200 a 4000 °C) (–272 a 400 °C)

Conductividad  Aislante en sólido peroconductor fundido o en

disolución.

Conductor Aislante osemiconductor

Aislante

Generalmente se presenta en

Compuestos de los metalesy no metales

Metales de la mitadizquierda

No metales del centro No metales de la derecha

 Ejemplos KI, Na2CO3, LiH Na, Zn, bronce Diamante, Si, SiO2 O2, C6H6, H2O

79 | Enlace químico y estructura de la materia Licenciatura en Química

Page 79: Enlace Quimico Y Estructura de Materia

7/22/2019 Enlace Quimico Y Estructura de Materia

http://slidepdf.com/reader/full/enlace-quimico-y-estructura-de-materia 79/89

Bibliografía

Bibliografía complementaria

1 L. Smart, E. Moore, “Química del estado sólido”, Addison-Wesley Iberoamericana, Madrid, 1995, 301

páginas, ISBN 0-201-62582-2.

Seminarios

fuerzas intermoleculares

4.1 Lista los tipos de fuerzas intermoleculares que puede haber entre los pares de moléculas o átomos dados:

a) Cl2, b) Ar, c) HCl, d) HF, e) N2, f) H2O, g) CH4, h) CH2=CH2, i) I2.

4.2 ¿Qué molécula es más polar, BF3 ó PF3? ¿Qué molécula contiene los enlaces más polares? ¿Quien tendrá

el punto de fusión más elevado (piensa en quien tendrá mayores interacciones de Van der Waals)?

4.3 ¿Cuáles de las siguientes sustancias formarán probablemente enlaces de hidrógeno?

a) HF, b) NH3, c) CH4, d) CH3OH (metanol), e) CH3COOH (ácido acético), f) H3PO4, g) CH3 –O–CH3.

4.4 Di qué fuerzas intermoleculares hay que superar paraa) fundir el hielo, b) fundir I2 sólido, c) convertir NH3 líquido en NH3 vapor, d) eliminar el agua de

hidratación de MnCl2⋅4H2O

4.5 ¿Qué sustancia de cada par tendrá probablemente el punto de fusión más elevado?

a) HF y HCl, b)

C C

H

Cl Cl

H

y C C

H

Cl H

Cl, c) CH4 y SiH4, d) HCl y HBr.4.6 Explica por qué el punto de ebullición del H2S es más bajo que el del agua.

4.7 Las masas molares del bromo, Br2, y el cloruro de yodo son casi iguales, 159,8 g/mol y 162,4 g/mol,respectivamente. Explica el hecho de que la temperatura de ebullición normal del Br2 sea 59 °C inferior a

la del ICl, que es de 97 °C.

teoría de bandas

4.8 Aplicando la teoría de bandas explica los conceptos de conductor, semiconductor y aislante. ¿Afecta por

igual la temperatura a un conductor y a un semiconductor?

sólidos metálicos

4.9 La aleación intersticial WC tiene la estructura del cloruro de sodio. Descríbela en términos de los

agujeros de una estructura compacta.4.10 Señala de qué tipo son probablemente los siguientes semiconductores extrínsecos:

a) Ge dopado con As; b) Ge dopado con Ga.

estructura de los sólidos iónicos

4.11 ¿A qué se debe que los compuestos iónicos tengan elevados puntos de fusión?

4.12 En una estructura de tipo cloruro de sodio,

a) ¿Cuál es el número de coordinación del catión y del anión?

b) ¿Cuantos iones Na+ ocupan las posiciones de segunda línea alrededor de un ion Na+?

4.13 En una estructura de tipo cloruro de cesio,

a) ¿Cuál es el número de coordinación del catión y del anión?b) ¿Cuantos iones Cs+ ocupan las posiciones de segunda línea alrededor de un ion Cs+?

4.14 Señala el tipo de red cristalina al que pertenecen las estructuras iónicas a–c de la figura 4.16.

4.15 a) Sabiendo que los radios iónicos de Sr2+ y Se 2– son 1,14 y 1,86 Å, respectivamente, predice el tipo de

Universidad de Alcalá Tema 4: Estructura de la materia | 80

Page 80: Enlace Quimico Y Estructura de Materia

7/22/2019 Enlace Quimico Y Estructura de Materia

http://slidepdf.com/reader/full/enlace-quimico-y-estructura-de-materia 80/89

red en que cristalizará el SrSe. Idem para b) CsF (r+ = 1,81 Å, r – = 1,15 Å); c) MgS (r+ = 0,80 Å, r – =

1,70 Å); d) ZnBr2 (r+ = 0,74 Å, r – = 1,82 Å); e) KI (r+ = 1,52 Å, r – = 2,06 Å).

entalpía de red

4.16 ¿Cuáles de los siguientes pares de compuestos isoestructurales serán probablemente más estables frente a

la descomposición térmica? a) MgCO3, CaCO3 (la descomposición da MO + CO2); b) CsI3, N(CH3)4I3

(ambos compuestos contienen I3 –; la descomposición da MI + I2; el radio de N(CH3)4+ es mucho mayorque el de Cs+).

4.17 Presenta un argumento para dar cuenta del hecho de que, al quemarlo en oxígeno, el litio forma el óxido

Li2O, mientras que el sodio forma el peróxido Na2O2.

4.18 Justifica el hecho de que la diferencia existente entre las energías de red de LiI y NaI es menor que entre

LiF y NaF.

4.19 Explica el orden de entalpía de red de los siguientes compuestos:

a) ∆ H U(MgO) = 3850 kJ mol –1 y ∆ H U(BaO) = 3114 kJ mol –1.

b) ∆ H U(MgO) = 3850 kJ mol –1 y ∆ H U(MgS) = 3406 kJ mol –1.

4.20 Ordena los siguientes compuestos por su entalpía de red:a) KCl, b) LiCl, c) KBr, d) NaCl, e) KI.

4.21 Ordena los haluros de plata por su entalpía de red.

4.22 Explica la variación de los puntos de fusión en los siguientes compuestos iónicos:

a) NaF (1 261 K) y MgO (3 073 K), b) NaF (1 261 K), NaCl (1 074 K), NaBr (1 028 K) y NaI (924 K).

4.23 Observa los siguientes datos experimentales:Compuesto: Punto de fusión Solubilidad en aguaKCl 776 °C 34,7 g/100 ml (a 20°C)KBr 730 °C 53,5 g/100 ml (a 20°C)KI 686 °C 127,5 g/100 ml (a 20°C)

Relaciona el orden de puntos de fusión y de solubilidad encontrado con el previsto para la entalpía de red

por la ecuación de Born–Landé. De acuerdo a lo que sabes, ¿te hubiera sorprendido encontrar un orden

de solubilidad contrario?

4.24 Define: carga iónica, entalpía de red, entalpía de formación, entalpía de disociación, entalpía de

hidratación, entalpía de disolución.

sólidos covalentes

4.25 Compara las estructuras del diamante y del grafito (ver figura 4.21):

a) ¿Qué tipo de modelo (iónico, metálico, covalente ó molecular) describe mejor el enlace dentro de una

capa de la estructura del grafito?.b) ¿Qué tipo de modelo describe mejor el enlace entre capas en la estructura del grafito?

c) Explica por qué el grafito, al contrario que el diamante, es muy blando, mientras que, como el

diamante, tiene un punto de fusión muy alto.

4.26 Da una explicación estructural al hecho de que el cuarzo es duro, el asbesto fibroso y filamentoso, y la

mica tiene un aspecto laminar.

clases de sólidos

4.27 Clasifica como metálico, molecular, iónico o de red covalente a cada uno de los siguientes sólidos:

a) es no conductor como sólido, pero conduce como líquido.

b) se disuelve en agua para dar una disolución no conductora.c) funde por debajo de 100 °C en un líquido no conductor.

d) conduce la electricidad en estado sólido.

4.28 Para las siguientes sustancias en estado sólido, di la unidad estructural a partir de la cual se construye el

81 | Enlace químico y estructura de la materia Licenciatura en Química

Page 81: Enlace Quimico Y Estructura de Materia

7/22/2019 Enlace Quimico Y Estructura de Materia

http://slidepdf.com/reader/full/enlace-quimico-y-estructura-de-materia 81/89

sólido, el tipo de enlace dentro de la unidad y entre unidades, así como el tipo de sólido:

a) P4, b) C(diamante), c) KBr, d) TiCl4, e) I2, f) K, g) IBr, h) N2, i) H2O, j) Cu, k) CaCO3.

4.29 Distingue, entre las siguientes sustancias, las que serán conductoras de las que serán aislantes en estado

sólido:

a) Na, b) NaCl, c) TiCl4, d) Fe, e) CsCl2.

4.30 Clasifica cada una de las siguientes sustancias como sólido molecular, iónico, covalente o metálico:a) b) c) d) e) f) g)

Sustancia CeCl3 Ti TiCl4 NO2F B MoF6 Se8

 Punto de fusión (° C) 848 1675 –25 –175 2300 17,5 (406 torr) 217 Punto de ebullición (°C) 1727 3260 136 –45,9 2550 35 684Conductor eléctrico en sólido no sí no no no no maloConductor eléctrico en líquido sí sí no no no no malo

4.31 ¿Cuál puede ser la causa de las diferencias entre los puntos de fusión de las siguientes sustancias: SiO2

(1710 °C), NaCl (810 °C), Cl2 (–100 °C), H2O (O °C) y SF6 (–35 °C)?

Problemas

estructura cristalina

4.1 Muestra que la esfera mayor que puede caber en un agujero octaédrico de un empaquetamiento compacto

tiene un radio igual a 0,414 veces el de las esferas que forman la red. (Truco: Considera los cuatro

círculos obtenidos cortando por un plano que atraviese el agujero y el cuadrado que forman las esferas

que lo rodean).

sólidos metálicos

4.2 El aluminio cristaliza en una estructura de empaquetamiento compacto cúbico. Su radio metálico es 1,25

Å (1 Å = 10 –10 m) y su masa molar es 26, 98 g mol –1.

a) ¿Cuál es la longitud de una cara de la celdilla unidad?

b) ¿Cuál es la densidad del aluminio ( N A = 6,022 1023 mol –1)?

4.3 El oro cristaliza en una estructura de empaquetamiento compacto cúbico. Su densidad es 19,3 g/cm3 y su

masa molar es 196,97 g mol –1. Calcula su radio metálico ( N A = 6,022 1023 mol –1).

estructura de los sólidos iónicos

4.4 A partir de los siguientes datos sobre la longitud de una cara de la celdilla unidad de compuestos que

cristalizan en la estructura de tipo NaCl, determina el radio del catión: MgSe (5,45 Å), CaSe (5,91 Å),

SrSe (6,23 Å), BaSe (6,62 Å). (Para determinar el radio del Se2–, haz la suposición de que los iones Se2–

están en contacto en MgSe).entalpía de red, ciclo de Born–Haber

4.5 a) Calcula la entalpía de red del bromuro de potasio [∆ H U(KBr)] sabiendo que su entalpía de formación

[∆ H f (KBr(s))] vale –392 kJ mol –1, la entalpía de sublimación del potasio [∆ H S(K)] es 90 kJ mol –1, la

entalpía de disociación del bromo gaseoso [∆ H D(Br2(g))] es 193 kJ mol –1 y la de evaporación

[∆ H vap(Br(l))] es 31 kJ mol –1. La entalpía de ionización del potasio [∆ H I(K(g))] es 418 kJ mol –1 y la

afinidad electrónica del bromo [ EA(Br)] es 323 kJ mol –1. El bromo es líquido en condiciones normales.

b) Compara el valor obtenido en el apartado a) para la entalpía de red del bromuro de potasio con el

estimado mediante la fórmula de Born–Landé. El bromuro potásico cristaliza en una red similar a la del

cloruro de sodio [el valor de la constante de Madelung ( M ) para dicho tipo de red es 1,74756], el radio

iónico del bromuro es 1,95 Å (1 Å = 10 –10

m), el del potasio 1,33 Å, y el factor de Landé (n) para el parbromuro–potasio vale 9,5. El número de Avogadro ( N A) es 6,022 1023 mol –1 y la carga elemental (e)

1,602 10 –19 C. La permitividad del vacío (ε o) es igual a 8,854 10 –12 C2 m –1 J –1.

4.6 Calcula la afinidad electrónica del yodo (el yodo es sólido en condiciones normales), a partir de los

Universidad de Alcalá Tema 4: Estructura de la materia | 82

Page 82: Enlace Quimico Y Estructura de Materia

7/22/2019 Enlace Quimico Y Estructura de Materia

http://slidepdf.com/reader/full/enlace-quimico-y-estructura-de-materia 82/89

siguientes datos (en kJ mol –1): ∆ H f (KI(s)) = –327; ∆ H S(K(s)) = 90; ∆ H S(I2(s)) = 62; ∆ H D(I2(g)) = 151;

∆ H I(K(g)) = 418; ∆ H U(KI(s)) = 645.

4.7 Calcula la afinidad electrónica ( EA) del cloro, a partir de los siguientes datos (en kJ mol –1):

∆ H f (CaCl2(s)) = –794; ∆ H I1+I2(Ca(g) = 1734; ∆ H D(Cl2(g)) = 242; ∆ H S(Ca) = 171; ∆ H U(CaCl2) = 2239.

4.8 Calcula la entalpía de red (∆ H U) de red del Na2O, a partir de los siguientes datos (en kJ mol –1):

∆ H f (Na2O(s)) = –416; ∆ H D(O2(g)) = 496; EA(O(g)) = –690; ∆ H S(Na) = 100; ∆ H I(Na(g) = 494.4.9 Calcula la entalpía de ionización (∆ H I) para la plata, a partir de los siguientes datos (en kJ mol –1):

∆ H f (AgCl(s)) = –127; ∆ H S(Ag) = 284; EA(Cl(g)) = 349; ∆ H U(AgCl(s)) = 916; ∆ H D(Cl2(g)) = 242.

4.10 a) Estima, mediante la fórmula de Born–Landé, la entalpía de red para la sustancia ficticia NaCl2. Supón

una red tipo rutilo [el valor de la constante de Madelung ( M ) para dicho tipo de red es 2,408] y una

distancia Na–Cl de 2,53 Å (1 Å = 10 –10 m). El factor de Landé (n) para el par cloruro–sodio vale 8,0. El

número de Avogadro ( N A) es 6,022 1023 mol –1 y la carga elemental (e) 1,602 10 –19 C. La permitividad

del vacío (ε o) es igual a 8,854 10 –12 C2 m –1 J –1.

b) Considera la reacción hipotética Na(s) + Cl2(g) → NaCl

2(g), donde el producto obtenido contienen

iones Na2+ y Cl –. Estima la entalpía de formación para el dicloruro de sodio y di si es o no estable

[∆ H S(Na(s)) = 100 kJ mol –1, ∆ H D(Cl2(g)) = 242 kJ mol –1, ∆ H I1(Na(g)) = 494 kJ mol –1,

∆HI2(Na(g)) = 4560 kJ mol –1, EA(Cl) = 349 kJ mol –1].

4.11 Si los radios iónicos de Sr2+ y Se2– son 1,14 y 1,86 Å respectivamente (1Å = 10 –10 m), el SrSe cristaliza

en una red de tipo cloruro de sodio, y el estroncio y el selenio son sólidos en condiciones normales:

a) Estima el valor de la entalpía de red por el método de Born–Landé [ M  (NaCl) = 1,7476; n = 10;

 N A = 6,022 1023 mol –1, e =1,602 10 –19 C, ε o = 8,854 10 –12 C2 m –1 J –1].

b) Calcula el valor de la entalpía de red por el ciclo de Born–Haber [∆ H f (SrSe(s)) = –602 kJ mol –1;

 EA(Se(g)) = –425 kJ mol –1

; ∆HI1+I2(Sr(g)) = 1617 kJ mol –1

; ∆ H S(Sr(s)) = 207 kJ mol –1

;∆ H S(Se(s)) = 164 kJ mol –1].

4.12 El NiF2 cristaliza en una estructura de tipo fluorita ( M = 2,5194).

a) Estima el valor de la entalpía de red por el método de Born–Landé [d  = 2,0 Å, n = 8,0;

 N A = 6,022 1023 mol –1, e =1,602 10 –19 C, ε o = 8,854 10 –12 C2 m –1 J –1].

b) Calcula el valor de la entalpía de red por el ciclo de Born–Haber [∆ H f (NiF2(s)) = –785 kJ mol –1;

 EA(F(g)) = 328 kJ mol –1; ∆HI1+I2(Ni(g)) = 2480 kJ mol –1; ∆ H S(Ni(s)) = 355 kJ mol –1; ∆ H D(F2(g)) =158

kJ mol –1].

entalpía de red, entalpía de hidratación y entalpía de disolución

4.13 Calcula la entalpía de disolución del fluoruro de potasio, sabiendo que ∆ H U(KF(s)) = 798 kJ mol –1,∆ H H(K+) = –322 kJ mol –1 (H = hidratación o solvatación con agua) y ∆ H H(F –) = –506 kJ mol –1.

Soluciones a los seminarios4.1 Cl2 Ar HCl HF N2 H2O CH4 H2C=CH2 I2

 Dipolo–Dipolo si si si Fuerzas de London si si si si si si si si si Enlace de Hidrógeno si si

4.2 La molécula de BF3 (triangular plana) es no polar. La molécula de PF 3 (pirámide triangular) es polar.El BF3 tiene los enlaces más polares ya que la diferencia de electronegatividad entre sus átomos es mayor.El PF3 tiene mayor punto de fusión, ya que sus moléculas interaccionan mediante interacciones dipolo–dipolo además de

fuerzas de London.4.3 a), b), d), e), f). (tienen F, O, o N e hidrógenos con carga parcial positiva).4.4 a) b) c) d)

 Dipolo–Dipolo si si si (ion–dipolo) Fuerzas de London si si si si Enlace de Hidrógeno si si

83 | Enlace químico y estructura de la materia Licenciatura en Química

Page 83: Enlace Quimico Y Estructura de Materia

7/22/2019 Enlace Quimico Y Estructura de Materia

http://slidepdf.com/reader/full/enlace-quimico-y-estructura-de-materia 83/89

4.5 a) HF (enlace de hidrógeno); b) el primer isómero (es polar); c) SiH4 (fuerzas de London mayores); d) HBr (fuerzas deLondon mayores, pues su tamaño es mayor).

4.6 Porque no hay enlace de hidrógeno entre sus moléculas.4.7 Las moléculas de ICl son polares mientras que las de Br2 son apolares.4.8 Ver teoría.4.9 Empaquetamiento cúbico compacto de átomos de wolframio con átomos de carbono en los agujeros octaédricos.

4.10 a) tipo n; b) tipo p.4.11 Ver teoría4.12 a) (6,6); b) 12.4.13 a) (8,8); b) 6.4.14 a) cúbica centrada en las caras; b) cúbica simple; c) cúbica centrada en las caras.4.15 a) 1,14/1,86 = 0,61, cloruro de sodio (ver tabla 4.6); b) 0,66, cloruro de sodio; c) 0,47, cloruro de sodio; d) 0,41, rutilo; e) 0,74,

cloruro de cesio.4.16 Catión grande estabiliza a anión grande. a) CaCO3; b) N(CH3)4I3.4.17 El catión mayor (Na) estabiliza al anión grande (O2

2–) frente a su descomposición en O2– y oxígeno.4.18 Ver figura 4.16.4.19 Si consideramos un modelo 100% iónico, se debe cumplir la ecuación de Born-Landé que predice un aumento de la entalpía

de red al disminuir el radio de los iones, siempre que la carga iónica sea la misma. Por lo tanto, el orden a) es porque el radio

del Mg2+ es menor que el del Ba2+ y el b) porque el radio del O2– es menor que el del S2–.4.20 LiCl > NaCl > KCl > KBr > KI, ver seminario 2.4.4.21 AgF > AgCl > AgBr > AgI, ver seminario 2.4.4.22 Si consideramos un modelo 100% iónico, el aumento de la entalpía de red debe producir un aumento del punto de fusión. La

entalpía de red aumenta al disminuir el radio y al aumentar la carga iónica (ver seminario 2.4). a) La carga de los ionesaumenta al pasar de NaF a MgO. b) El tamaño del anión aumenta al ir de NaF a NaI.

4.23 La entalpía de red aumenta al ir de KCl a KI. Se observa una disminución del punto de fusión y un aumento de la solubilidadal disminuir la entalpía de red. La variación del punto de fusión se corresponde con lo discutido en 2.7. El aumento del radiode los aniones al ir de KCl a KI disminuye la entalpía de red, lo que favorece la solubilidad, pero también hace menosexotérmica la hidratación de los iones, lo que la dificulta. Por ello, un aumento del radio puede tanto aumentar como disminuirla solubilidad.

4.24 Ver teoría.

4.25 a) Covalente; b) molecular; c) Es blando porque una capa desliza con facilidad sobre la otra (están unidas por fuerzas de Vander Waals). Tiene alto punto de fusión porque para que se puedan mover con facilidad partículas pequeñas, es necesariovencer los enlaces covalentes.

4.26 El cuarzo es un sólido covalente tridimensional, el asbesto es monodimensional y la mica bidimensional.4.27 a) iónico; b) molecular; c) molecular; d) metálico.4.28 Unidad, enlace dentro de unidad, enlace entre unidades, tipo de sólido: a) P4, covalente, Van der Waals, sólido molecular; b)

C, ninguno, covalente, sólido covalente tridimensional; c) K+ y Br –, ninguno, iónico, sólido iónico; d) TiCl4, covalente, Vander Waals, sólido molecular; e) I2, covalente, Van der Waals, sólido molecular; f) K, ninguno, metálico, sólido metálico; g)IBr, covalente, Van der Waals, sólido molecular; h) N 2, covalente, Van der Waals, sólido molecular; i) H2O, covalente, Vander Waals, sólido molecular; j) Cu, ninguno, metálico, sólido metálico; k) Ca2+ y CO3

 –, ninguno en Ca2+ y covalente en CO3 –,

iónico, sólido iónico.4.29 Serán conductoras a) y d). El resto serán aislantes.4.30 a) iónico; b) metálico; c) molecular; d) molecular; e) covalente; f) molecular; g) molecular.4.31 SiO2 es un sólido covalente y NaCl es iónico, mientras que el resto son moleculares.

Soluciones a los problemas4.1

2r1

2r1 + 2r2

r2

r1

(2r1)2 + (2r1)2 = (2r1 + 2r2)2

8r12 = (2r1 + 2r2)2

(8)1/2r1 = 2r1 + 2r2

[(8)1/2 –2]r1 = 2r2

r2/r1 = [(8)1/2 –2]/2 = 0,4144.2 a) 3,54 Å, b) 4,05 g/cm3.4.3 1,44 Å.4.4 r(Se2–) = 1,93 Å; r(Ca2+) = 1,03 Å;

r(Sr2+) = 1,19 Å; r(Ba2+) = 1,38 Å.4.5 a) ∆ H U = 689 kJ mol –1; b) ∆ H U = 664 kJ mol –1.

4.6  EA = 296,5 kJ mol –1

.4.7  EA = 351 kJ mol –1.4.8 ∆ H U = 2541 kJ mol –1.4.9 733 kJ mol –1.

4.10 a) ∆ H U = 2314 kJ mol –1;,b) ∆ H f = 2384 kJ mol –1, no es estable.

Universidad de Alcalá Tema 4: Estructura de la materia | 84

Page 84: Enlace Quimico Y Estructura de Materia

7/22/2019 Enlace Quimico Y Estructura de Materia

http://slidepdf.com/reader/full/enlace-quimico-y-estructura-de-materia 84/89

4.11 a) 2913 kJ mol –1; b) 3015 kJ mol –1.4.12 a) 3061 kJ mol –1; b) 3122 kJ mol –1.4.13 ∆ H d = –30 kJ mol –1.

85 | Enlace químico y estructura de la materia Licenciatura en Química

Page 85: Enlace Quimico Y Estructura de Materia

7/22/2019 Enlace Quimico Y Estructura de Materia

http://slidepdf.com/reader/full/enlace-quimico-y-estructura-de-materia 85/89

© 2003, Ernesto de Jesús Alcañiz_________________________________________________________________________________________

5 Gases y líquidos5.1 La teoría cinético-molecular de los gases

5.2 Predicciones de la teoría cinético-molecular

5.3 Los gases reales: ecuación de Van der Waals

5.4 Propiedades de los líquidos_________________________________________________________________________________________

5.1 La teoría cinético-molecular de los gases

 Ley de los gases ideales. El comportamiento de los gases es aproximadamente descrito por la llamada ley

de los gases ideales, cuya expresión matemática es pV = nRT , donde R recibe el nombre de constante de

los gases ideales y vale 0,08205 litro atm K –1 mol –1. El comportamiento de un gas real sólo se aproxima

al descrito por dicha ley. Por conveniencia, se define gas ideal como el que la cumple exactamente en

todas las condiciones.

El volumen molar (V m) de un gas es el ocupado por un mol de dicho gas, V m = V /n. Las

condiciones normales (CN) en gases son 273,15 K (0 °C) de temperatura y 1 atm de presión. En esas

condiciones, el volumen molar se llama volumen molar normal y, para cualquier gas ideal, es de 22,414litros (tabla 5.1).

Tabla 5.1. Densidades y volúmenes molares de algunos gases

Gas H2 He Ne N2 O2 Ar CO2 NH3

 Densidad (g/litro) 0,090 0,178 0,900 1,250 1,429 1,784 1,977 0,771

 Masa molar (g/mol) 2,02 4,003 20,18 28,01 32,00 39,95 44,01 17,03

Volumen molar normal (l/mol) 22,428 22,426 22,425 22,404 22,394 22,393 22,256 22,094

Teoría cinético-molecular. La teoría cinético-molecular de los gases fue desarrollada por L. Boltzmann

(1844–1906) y J. C. Maxwell (1831–1879). Esta teoría explica satisfactoriamente el comportamiento de

los gases ideales a partir de las siguientes hipótesis:

1. Un gas está compuesto de un gran número de partículas pequeñas (moléculas) de tamaño despreciable

frente a las distancias entre ellas.

2. Las moléculas se mueven en movimiento rectilíneo, rápido, constante y casual. Las moléculas chocan

entre sí y con las paredes en choques elásticos (es decir, no hay fuerzas de atracción o repulsión entre

las moléculas, ni entre éstas y el recipiente, diferentes a las del choque).

3. Cada molécula tiene una energía cinética (velocidad) propia que no tiene que ser igual a la de las

restantes moléculas, pero la energía cinética promedio de todas las moléculas es proporcional a la

temperatura absoluta (figura 5.1). La temperatura es simplemente una consecuencia del movimiento

molecular.

Figura 5.1. Distribución de las energías cinéticas de

las moléculas de un gas, según la teoría cinético-

molecular. Las energías cinéticas promedio se indican

mediante líneas de trazos. Conforme aumenta la

temperatura, la energía cinética promedio de las

moléculas aumenta. A cualquier temperatura, una

porción de las moléculas puede tener velocidades muyaltas.

Según este modelo, la presión es el resultado de las colisiones de las moléculas con las paredes del

recipiente, a las que transfieren cantidad de movimiento. Si consideramos el problema de N partículas,

Page 86: Enlace Quimico Y Estructura de Materia

7/22/2019 Enlace Quimico Y Estructura de Materia

http://slidepdf.com/reader/full/enlace-quimico-y-estructura-de-materia 86/89

cada una de las cuales tiene una masa m, que están moviéndose en un cubo de volumen V (figura 5.2), a

un velocidad media v, puede deducirse fácilmente que la presión que ejercen es p = Nmv2/3V , o, escrito

de otra forma, pV =nN A2 E c/3, donde E c es la energía cinética media de cada partícula.

v xv y

v z a

Figura 5.2. Modelo de una partícula gaseosa en una caja cúbica.

a = arista de la caja.

V = volumen del cubo = a 2

v (1) = velocidad de la partícula 1

 E c (1) = energía cinética de la partícula 1

v = velocidad media de las partículas

 E c = energía cinética media de la partículas

Esta expresión es idéntica a la de la ecuación de los gases ideales, si suponemos que la relación postulada

en el punto 3 entre la energía cinética media por partícula del gas y la temperatura absoluta es

 E c = 3 RT /2 N A. Si consideramos la energía cinética de 1 mol de partículas, E c = 3 RT /2.

5.2 Predicciones de la teoría cinético-molecular

 Ley de las mezclas gaseosas. La formuló Dalton a partir de observaciones experimentales. “La presión

total de una mezcla de gases es igual a la suma de las presiones parciales de cada uno de los componen-

tes individuales de la mezcla gaseosa, definiendo presión parcial como la que produciría cada gas si

estuviera sólo en el recipiente”. Si tenemos una cantidad n1 de un gas 1, n2 de un gas 2… mezclados en

un recipiente de volumen total V y a una temperatura T , por definición llamaremos presión parcial de 1 a

 p1 = n1 RT/V , presión parcial de 2 a p2 = n2 RT/V … La ley de Dalton dice que la presión total p debe valer

 p = p1 + p2 +…. Este comportamiento es explicado por la teoría cinético-molecular (por el postulado 2).

Capacidad calorífica molar de los gases. La capacidad calorífica molar de un gas es la cantidad de

energía que hay que suministrar para aumentar una unidad la temperatura de un mol de sustancia. En ungas monoatómico, dicha energía se invierte únicamente en el movimiento de traslación de sus moléculas

y por tanto debe ser C v = ∆ E c = 3 R(T +1)/2 – 3 RT /2 = 3 R/2 = 12,472 J mol –1, lo que se confirma

experimentalmente ( R expresado en J K –1 mol –1 vale 8,3144). En moléculas poliatómicas, el calor debe

de invertirse también en los movimientos de rotación y vibración, y la deducción de su capacidad

calorífica molar es algo más compleja (tabla 5.2).

Tabla 5.2. Capacidades caloríficas molares de algunos gases, a volumen constante

Gas He Ne Ar H2 N2 O2 HCl CO2 CH4

Capacidad molar (J/mol K) 12,472 12,472 12,472 20,510 20,811 21,041 20,81 28,80 27,38

 Ley de la difusión de los gases. Graham (1805–1869) observó en 1846 que las velocidades de difusiónde dos gases a la misma presión y temperatura están en razón inversa de la raíz cuadrada de sus masas

molares:

v

v

 M 

 M 

difusión(1)

difusión(2)

=2

1

La teoría cinético-molecular permite comprender esta relación pues de ella se deduce que la velocidad

media de las partículas de un gas es inversamente proporcional a la raíz cuadrada de su masa molar.

v E 

m

 RT 

 M 

c= =

2 3

5.3 Los gases reales: ecuación de Van der Waals

Los gases reales se desvían del comportamiento ideal (figura 5.3) porque las moléculas reales tienen un

volumen no siempre despreciable y además se atraen entre sí con una fuerza no siempre despreciable.

Universidad de Alcalá Tema 5: Gases y líquidos | 87

Page 87: Enlace Quimico Y Estructura de Materia

7/22/2019 Enlace Quimico Y Estructura de Materia

http://slidepdf.com/reader/full/enlace-quimico-y-estructura-de-materia 87/89

Las atracciones moleculares son más relevantes cuando las moléculas se mueven lentamente, es decir, a

bajas temperaturas; cuando los gases se comprimen, los volúmenes moleculares se hacen importantes.

Por ello los gases se aproximan más al comportamiento ideal a temperaturas altas y presiones bajas.

gas ideal

 pV 

 RT 

1,3

1,1

1,0

0,9

 p, en atmósferas

(en

moles)

200 400 600

 –  1  0  0

   °  C

  0   ° C

 1 0 0  ° C

gas ideal pV  RT 

1,0

 p, en atmósferas

(en

moles)

20 40 60

H 2

N 2

O 2

CH4

80 100

0 °C

Figura 5.3. Desviación del comportamiento ideal de los gases reales. Para un mol de un gas ideal, la relación PV / RT debe ser

igual a 1, independientemente de la presión (línea a trazos). (a) Comportamiento de un mol de diferentes gases a la misma

temperatura. La desviación del comportameinto ideal se acentúa al aumentar la presión. (b) En cambio, al aumentar latemperatura, el comportamiento se acerca al de un gas ideal, como se observa en la gráfica para un mol de nitrógeno gas.

Se han propuesto varias ecuaciones de estado para los gases reales. Van der Waals modificó la ley de los

gases ideales en 1873, para tener en cuenta los dos factores anteriores. La ecuación de Van der Waals es

( p + n2a/V 2)(V  –nb) = nRT , donde a es una constante experimental relacionada con las atracciones

moleculares, y b está relacionada con el volumen molecular (tabla 5.3).

Tabla 5.3. Constantes de Van der Waals

Gas Hg He H2 H2O O2 N2 CO2

a (l2 atm mol –2) 8,09 0,0341 0,2444 5,464 1,360 1,390 3,592

b (cm3 mol –1) 17,0 23,70 26,61 30,49 31,83 39,12 42,67

5.4 Propiedades de los líquidos

Viscosidad. La fluidez de un líquido es tanto mayor cuanto menor es su viscosidad . La viscosidad

aumenta con las fuerzas intermoleculares.

Tensión superficial. Las moléculas en la superficie del líquido están menos atraídas por las fuerzas

intermoleculares, por lo que prefieren situarse en el interior (figura 5.4). La tensión superficial mide la

energía necesaria para ampliar la superficie de un líquido.

Figura 5.4. Diferencia entre las fuerzas que actúan sobre una molécula

dentro del líquido y las que actúan sobre una molécula en la superficie.Hay una fuerza resultante hacia el interior de la disolución sobre las

moléculas de la superficie. Esta fuerza es la responsable de que las gotas

de agua sean esféricas, ya que una esfera tiene un área menor que

cualquier otra forma del mismo volumen.

 Acción capilar. Las fuerzas que mantienen unido al líquido se denominan  fuerzas de cohesión. Las

fuerzas de atracción entre un líquido y otra superficie se denominan fuerzas de adhesión. Las diferencias

de magnitud entre ambas fuerzas son las responsables del menisco producido por un líquido en un

recipiente y de la acción capilar (figura 5.5).

88 | Enlace químico y estructura de la materia Licenciatura en Química

Page 88: Enlace Quimico Y Estructura de Materia

7/22/2019 Enlace Quimico Y Estructura de Materia

http://slidepdf.com/reader/full/enlace-quimico-y-estructura-de-materia 88/89

(a)

agua

(b)

Hg

(c)

h

Figura 5.5. a) Como el agua y el vidrio son polares, las fuerzas de

adhesión entre ambos son elevadas y mayores que las de cohesión del

líquido. Por ello, el agua asciende por los lados de un tubo de vidrio,

aumentando así la superficie de contacto. El menisco es cóncavo.

b) En cambio, en el mercurio las fuerzas de cohesión son mayores quelas de adhesión, por lo que se forma un menisco convexo.

c) La acción capilar puede observarse en un tubo de pequeño diámetro.

Cuando las fuerzas de adhesión son mayores que las de cohesión, ellíquido continúa ascendiendo por los lados del tubo hasta que se alcanza

el equilibrio entre las fuerzas de adhesión y el peso del líquido.

Bibliografía

Seminarios

teoría cinético-molecular de los gases

5.1 ¿Por qué los gases obedecen leyes más sencillas que los líquidos o los sólidos?.

5.2 ¿Qué sucedería si las moléculas de un gas permanecieran estáticas: aumentaría, seguiría igual o

descendería la presión creada por el gas?.

5.3 ¿Por qué las moléculas de oxígeno se mueven más lentamente que las de nitrógeno a igual temperatura?.

5.4 Los isótopos 235U y 238U se separan utilizando la diferencia de velocidad de difusión de sus

hexafluoruros gaseosos, 235UF6 y 238UF6. ¿Cuál de los dos compuestos se recoge antes?.

gases reales

5.5 ¿Qué explicación molecular puede darse a la desviación de los gases reales del comportamiento de los

gases ideales? ¿En qué condiciones se aproximan más los gases reales a los ideales?.

5.6 ¿Cuál de los siguientes gases tendrá un comportamiento más ideal en las mismas condiciones? H2, F2,

HF. ¿Cuál se desviará más del comportamiento ideal?5.7 ¿Qué factores aproximan la ecuación de Van der Waals a la de los gases ideales?

5.8 Se observó que la presión de una muestra de fluoruro de hidrógeno era inferior a lo esperado, y que

aumentaba más rápidamente de lo que predice la ley de los gases ideales. Propón una explicación.

5.9 Busca una justificación al hecho de que el valor del parámetro a de la ecuación de Van der Waals sea

mayor para H2O que para He.

Problemas

teoría cinético-molecular

5.1 a) Calcula la energía cinética para un mol de moléculas de un gas ideal que se encuentra a la temperatura

de 25 °C ( R = 8,3144 J K –1 mol –1)

b) ¿Cómo es esta energía en comparación con la de los enlaces químicos, que típicamente es del orden de

350 kJ mol –1? ¿Qué sucedería si ambas energías fueran de magnitudes más semejantes?.

5.2 Calcula la velocidad media de las moléculas de

a) oxígeno ( M r = 32,0), a 25 °C y a 500 °C [ R= 8,314 J mol –1 K –1, N A = 6,022 1023 moléculas mol –1]

b) hidrógeno ( M r = 2,0), a 25 °C y a 500 °C.

5.3 Un átomo de helio reta a una molécula de hexafluoruro de uranio, UF6, a una carrera. Para tratar de hacer

una competición justa, acuerdan correr cada uno a la temperatura de sus puntos de ebullición normales,

que son 4,18 K para el He y 56 °C para el UF6. ¿Por cuál apostarías como vencedor? [ R = 8,3144 J K –1

mol –1, M r(F) = 18,995, M r(He) = 4,0026, M r(U) = 238,03].

difusión de los gases

5.4 El NH3 y el HBr, ambos gaseosos, se difunden en sentidos opuestos, a lo largo de un tubo estrecho de

Universidad de Alcalá Tema 5: Gases y líquidos | 89

Page 89: Enlace Quimico Y Estructura de Materia

7/22/2019 Enlace Quimico Y Estructura de Materia

http://slidepdf.com/reader/full/enlace-quimico-y-estructura-de-materia 89/89

longitud l  (ver figura). ¿En qué parte del tubo se encontrarán para formar NH4Br? [ M r(Br) = 79,904,

 M r(H) = 1,008, M r(N) = 14,007].

 x 

NH4Br HBrNH3

5.5 Un pistón se desplaza en un cilindro de gas en el que hay un agujero pequeño, a través del cual se

produce la difusión del gas en el vacío. Cuando hay oxígeno ( M r = 32,0) en el cilindro, el tiempo

empleado por el pistón en recorrer cierta distancia es 38,3 s. Con otro gas en iguales condiciones, el

pistón se desplaza la misma distancia en 82,0 s. ¿Cuál es la masa relativa del segundo gas?.

gases reales

5.6 Calcula para un mol de oxígeno en condiciones normales:

a) el volumen ocupado considerando comportamiento ideal ( R = 0,0821 atm l K –1 mol –1).

b) la presión predicha por la ecuación de Van der Waals para el volumen de oxígeno calculado

anteriormente (a = 1,360 l2 atm mol –2, b = 31,83 10 –3 l mol –1).

c) El porcentaje de diferencia entre las presiones del gas ideal y de Van der Waals.

Soluciones a los seminarios5.1 Por que las interacciones entre sus partículas son muy débiles.

5.2 Sería nula.

5.3 A una temperatura dada, las moléculas de gases diferentes tienen la misma energía cinética media, por lo que se moverán más

rápido las que sean más ligeras.

5.4 Se recoge antes el 235UF6, ya que al ser sus moléculas más ligeras se difunden más rápidamente (Ley de Graham).

5.5 Porque no se puede despreciar totalmente ni el volumen ocupado por las moléculas ni las interacciones entre ellas. Los gases

reales se aproximarán más a los gases ideales a presiones bajas (volúmenes grandes) y temperaturas altas (mayor energía

cinética de las moléculas).

5.6 Comportamiento más ideal: H2 (menor tamaño y menos fuerzas intermoleculares). Comportamiento menos ideal: HF.5.7 Presiones bajas (volúmenes altos) y temperaturas altas.

5.8 Se aleja del comportamiento ideal.

5.9 Fuerzas intermoleculares más débiles en el helio.

Soluciones a los problemas5.1 a) 3,72 kJ mol –1; b) aproximadamente el 1% de la energía 5.3 v(He) = 161 m s –1, v(UF6) = 153 m s –1.

de enlace; se produciría la ruptura térmica de enlaces. 5.4 A 0,69 l de la entrada de NH3 y 0,31 l de la de HBr.

5.2 a) v(O2, 25 °C) = 482 m s –1, v(O2, 500 °C) = 776 m s –1; 5.5  M r = 147.

b) v(H2, 25 °C) = 1,92 103 m s –1, 5.6 a) 22,41 l; b) 0,9990 atm;

v(H2, 500 °C) = 3,10 103 m s –1. c) 0,1 % de diferencia entre ambas presiones.

90 | Enlace químico y estructura de la materia Licenciatura en Química